Jump to content

Wikipedia:Reference desk/Science: Difference between revisions

From Wikipedia, the free encyclopedia
Content deleted Content added
→‎Medroxyprogesterone versus medroxyprogesterone acetate: correction - not sure about MP's use in humans at all
Scsbot (talk | contribs)
edited by robot: archiving May 24
 
Line 1: Line 1:
[[Category:Non-talk pages that are automatically signed]]<noinclude>{{Wikipedia:Reference desk/header|WP:RD/S}}
<!--- Please DO NOT enter your question at the top here. Put it at the bottom of the page. An easy way to do this is by clicking the "new section" tab ---><noinclude>{{Wikipedia:Reference desk/header|WP:RD/S}}
[[Category:Wikipedia help forums]]
[[Category:Non-talk pages that are automatically signed]]
[[Category:Wikipedia resources for researchers]]
[[Category:Pages automatically checked for incorrect links]]
[[Category:Wikipedia resources for researchers]]
[[Category:Wikipedia help forums]]
[[Category:Wikipedia reference desk|Science]]
[[Category:Wikipedia help pages with dated sections]] </noinclude>


= May 26 =


== Why is there no monosmium octafluoride or moniridium nonafluoride or PtF10? ==
</noinclude>


There's crazy ways to make new forms like electrocuting extremely hot plasmas and forcing noble gases to bond or crushing between microdiamonds yet no one's figured out how to make OsF8 or IrF9 but OsO4 could be made with Paleolithic tech (poisonous though). [[User:Sagittarian Milky Way|Sagittarian Milky Way]] ([[User talk:Sagittarian Milky Way|talk]]) 16:58, 26 May 2024 (UTC)
{{Wikipedia:Reference_desk/Archives/Science/2010 June 28}}
:The reason will be that there is not enough space to pack that many fluorine atoms around the metal, at a close enough distance to have strong enough bonds. The bond in the hexafluoride would be much stronger, and to add more fluorine, it would have to push out those other fluorine atoms to squeeze new ones in. The energy gain by this would have to exceed that of breaking an F-F bond in F<sub>2</sub>. Perhaps atomic fluorine could assist in a rare gas matrix, but in warmer conditions F<sub>2</sub> would be produced. Perhaps a diamond anvil press could do something, but I have not heard of its use for these. [[User:Graeme Bartlett|Graeme Bartlett]] ([[User talk:Graeme Bartlett|talk]]) 21:35, 26 May 2024 (UTC)
::Under high pressure [https://onlinelibrary.wiley.com/doi/10.1002/anie.202002339 it should be possible]. OsF<sub>8</sub> and IrF<sub>8</sub> are predicted to be stabilised by 300 GPa, as the 6p orbitals on the metal lower and we have ligand-to-metal charge transfer from F 2p to M 6p. Also TcF<sub>7</sub> and CdF<sub>3</sub> should stabilise (the former interesting because [[rhenium heptafluoride|ReF<sub>7</sub>]] exists at normal conditions and this is one case where Tc and Re differ; the latter interesting for breaking the oxidation state barrier for stable group 12 elements). [[User:Double sharp|Double sharp]] ([[User talk:Double sharp|talk]]) 05:43, 27 May 2024 (UTC)
:Re Paleolithic OsO<sub>4</sub>: an interesting thought, but even the easy metals like Au seem to have been only recognised later per [[Element discovery]]. If we want to push it as early as possible, then I think you'd need an early discovery of Pt (accomplished by pre-Columbian South Americans), together with enough alchemical knowledge to get [[aqua regia]] to separate out an Os-Ir fraction. [[User:Double sharp|Double sharp]] ([[User talk:Double sharp|talk]]) 06:44, 27 May 2024 (UTC)
::Well from osmium, not completely de novo. [[User:Sagittarian Milky Way|Sagittarian Milky Way]] ([[User talk:Sagittarian Milky Way|talk]]) 14:27, 28 May 2024 (UTC)


{{ping|Graeme Bartlett}} That said, is there a similarly simple explanation for why Rh, Ir, and Pt can reach the VI oxidation state only in fluorides and not oxides? And why is Pd<sup>VI</sup> so elusive despite this oxidation state being well-established for all other PGMs? [[User:Double sharp|Double sharp]] ([[User talk:Double sharp|talk]]) 07:04, 27 May 2024 (UTC)
{{Wikipedia:Reference_desk/Archives/Science/2010 June 29}}
:I will add there is no Pd<sup>V</sup> either. Higher oxidation states get less stable as you move right in the periodic table from W to Pt or Mo to Pd. [[User:Graeme Bartlett|Graeme Bartlett]] ([[User talk:Graeme Bartlett|talk]]) 23:36, 28 May 2024 (UTC)
::Thanks. Reading around, maybe there is no simple answer exactly for why the threshold of impossibility is precisely where it is, so I shall have to be satisfied by this. :) [[User:Double sharp|Double sharp]] ([[User talk:Double sharp|talk]]) 09:48, 29 May 2024 (UTC)
:::Google Bard tells me it is about electronegativity of fluorine and polarizability of oxygen. Not to be trusted though. High oxidation compounds all seem to be isolated molecules, and not network solids, which would be topologically possible with oxygen. So it seems that more single bonds is more stable than a few double bonds, but up to the limit of about 6 due to not being able to pack in the atoms around the metal. [[User:Graeme Bartlett|Graeme Bartlett]] ([[User talk:Graeme Bartlett|talk]]) 11:46, 29 May 2024 (UTC)
::::This certainly makes sense. And in the 3d series, as expected the limiting coordination number seems even less in some cases: we have CrO<sub>3</sub> but only CrF<sub>5</sub>, and Mn<sub>2</sub>O<sub>7</sub> but only MnF<sub>4</sub>. [[User:Double sharp|Double sharp]] ([[User talk:Double sharp|talk]]) 15:24, 29 May 2024 (UTC)
::For discussion of Pd(V) and Pd(VI) inorganic and synthesis of a Pd(V) cluster, see [[doi:10.1073/pnas.0700450104]]. [[User:DMacks|DMacks]] ([[User talk:DMacks|talk]]) 17:19, 29 May 2024 (UTC)
:[[User:Graeme Bartlett|Graeme Bartlett]], we don't ever use the mono- prefix with the first word of a binary compound's name. CO2 is carbon dioxide, not monocarbon dioxide. [[User:Georgia guy|Georgia guy]] ([[User talk:Georgia guy|talk]]) 10:15, 29 May 2024 (UTC)
::But not applicable to Sagittarian Milky Way who likes "mono"; and there is [[monosodium phosphate]] or [[monosodium glutamate]] for non-binary compounds. [[User:Graeme Bartlett|Graeme Bartlett]] ([[User talk:Graeme Bartlett|talk]]) 11:46, 29 May 2024 (UTC)


For actual Ir(IX), there is [[iridium tetroxide|IrO<sub>4</sub><sup>+</sup>]], though this is not "bottlable" stuff. [https://cse.umn.edu/ctc/news/highest-oxidation-state-nature Possibly] Pt(X) will be possible too in PtO<sub>4</sub><sup>2+</sup>, but that one has not yet been experimentally confirmed. [[User:Double sharp|Double sharp]] ([[User talk:Double sharp|talk]]) 04:19, 31 May 2024 (UTC)
= June 30 =


== Two universes in same space ==
== Are sodium chloride crystals always square? ==


My heading may be mediocre considering Nasa's simplified definition "The universe is everything. It includes all of space, and all the matter and energy that space contains". So
Recently I've been making homemade potato salad. Never even knew how much I loved it until I made it fresh. Anyway, you put a ton of salt in the water where you boil the potatoes. So I left the salt laden water and went away for two days and when I got back I had salt crystals all over the bottom of the pan, big fat squares and smaller ones, all square crystals. I thought this was pretty neat. I just headed over to the articles and see a picture of a square crystal at the top as well as the structure, which looks conducive to squares, but I'm curious if they're always square like this?--[[Special:Contributions/141.155.157.4|141.155.157.4]] ([[User talk:141.155.157.4|talk]]) 00:04, 30 June 2010 (UTC)
*can there exist two (or more) 'universes' in the same space-time domain?
**If yes, can we say the space compromises the universe and not vice-versa?
**If no, does that mean all of space is part of 'universe', thus even if mass in form of galaxy, planets etc. is limited in mass and volume, still all of that infinite vacuum that surrounds it is part of 'our universe'?
*If there is something far away from our 'observable universe' which did not come out of big bang (considering big bang happened for sure), would that still be considered part of our universe?
You may answer any part you like. Thanks, [[User:ExclusiveEditor|<span style="background:Orange;color:White;padding:2px;">Exclusive</span><span style="background:black; color:White; padding:2px;">Editor</span>]] [[User talk:ExclusiveEditor|<sub>Notify Me!</sub>]] 20:04, 26 May 2024 (UTC)
:*Your "if no" option means that empty space is part of the Universe. Far away stuff would be part of the universe. [[User:Graeme Bartlett|Graeme Bartlett]] ([[User talk:Graeme Bartlett|talk]]) 21:39, 26 May 2024 (UTC)
:* You talk of an infinite vacuum that surrounds the universe. That would be absolutely empty space. But everything that exists, even absolutely empty space, is by definition part of the universe. So, either there's some sort of boundary, beyond which there is nothing, not even absolutely empty space - which means there is no "beyond"; or, there is no boundary and the universe is infinitely limitless. Take your pick. -- [[User:JackofOz|<span style="font-family: Papyrus;">Jack of Oz</span>]] [[User talk:JackofOz#top|<span style="font-size:85%; font-family: Verdana;"><sup>[pleasantries]</sup></span>]] 22:39, 26 May 2024 (UTC)
:*:Or it is a finite closed [[manifold]]. [[User:Tamfang|—Tamfang]] ([[User talk:Tamfang|talk]]) 21:25, 30 May 2024 (UTC)


:The universe, by definition, is everything. Or did you say that already? ←[[User:Baseball Bugs|Baseball Bugs]] <sup>''[[User talk:Baseball Bugs|What's up, Doc?]]''</sup> [[Special:Contributions/Baseball_Bugs|carrots]]→ 00:28, 27 May 2024 (UTC)
:Like other crystals, salt has preferred [[Cleavage (crystal)|cleavage planes]]. In the case of NaCl, the preferred planes are aligned at 90-degree angles (in all three axes). As such, there is a preference for square and rectangular shapes. In practice, impurities and the mechanics of forming and breaking crystal faces means that there will be imperfections. You can see [[:File:Halite(Salt)USGOV.jpg|such imperfections]] in the many images on our [[sodium chloride]] article. The atomic structure means that the ''alignment'' of faces will prefer 90-degree angles; it doesn't ''directly'' affect the aspect ratio of the crystals that form, so it is equally easy to form rectangles as squares; but because the crystal probably grew homogeneously and [[isotropy|isotropically]], a cubic structure is very common. [[User:Nimur|Nimur]] ([[User talk:Nimur|talk]]) 00:38, 30 June 2010 (UTC)
::The universe has (potentially) been demoted. See [[Multiverse]] ({{small|not a long song}}). [[User:Clarityfiend|Clarityfiend]] ([[User talk:Clarityfiend|talk]]) 06:49, 27 May 2024 (UTC)
:::Not really "potentially" at this point, just "hypothetically". ←[[User:Baseball Bugs|Baseball Bugs]] <sup>''[[User talk:Baseball Bugs|What's up, Doc?]]''</sup> [[Special:Contributions/Baseball_Bugs|carrots]]→ 08:01, 27 May 2024 (UTC)
::Fairly sure that "absolutely empty space" is, very much by definition, not a "thing" and hence not part of "everything" [[Special:Contributions/2A01:E0A:CBA:BC60:CF2:682A:5D96:ED9D|2A01:E0A:CBA:BC60:CF2:682A:5D96:ED9D]] ([[User talk:2A01:E0A:CBA:BC60:CF2:682A:5D96:ED9D|talk]]) 07:50, 28 May 2024 (UTC)
::: Mathematicians might disagree with you. Is the [[empty set]], for example, a "thing"? I'd say so. I'll concede that there is some controversy on the point. --[[User:Trovatore|Trovatore]] ([[User talk:Trovatore|talk]]) 07:58, 28 May 2024 (UTC)
:::Not sure that's right. What about [[scalar fields]]? [[User:Sean.hoyland|Sean.hoyland]] ([[User talk:Sean.hoyland|talk]]) 08:02, 28 May 2024 (UTC)
----
Q. Imagine big bang really happened, so by definition that the entire space (our space time domain to be exact) is part of 'our universe', and so that will mean something far away '''which did not come out of our big bang''' is still part of 'our universe'; thus meaning that '''big bang did not necessarily give birth to 'our universe'''' but just '''''a part of 'our universe''''''. Also that means 'expansion of universe' would just mean expansion of ''''<u>our</u> part of <s>our</s> universe'''' which is near us and came thru our big bang (and not all universe)? [[User:ExclusiveEditor|<span style="background:Orange;color:White;padding:2px;">Exclusive</span><span style="background:black; color:White; padding:2px;">Editor</span>]] [[User talk:ExclusiveEditor|<sub>Notify Me!</sub>]] 07:45, 27 May 2024 (UTC)
:Where is the question in that? Our definition of Universe includes the far away stuff. The definition does not depend on how it was created. You may also be interested in the idea of an inter-penetrating alternate universe that uses the same space. For example a [[Dark matter]] world, or perhaps a [[Counter-Earth]], six months out of phase. I can imagine it as a [[clathrate]]. [[User:Graeme Bartlett|Graeme Bartlett]] ([[User talk:Graeme Bartlett|talk]]) 07:53, 27 May 2024 (UTC)


:The question is more of terminologies and what words means, than physics itself. But clarifying this helps as to know if the definitions are standardized or still vague. [[User:ExclusiveEditor|<span style="background:Orange;color:White;padding:2px;">Exclusive</span><span style="background:black; color:White; padding:2px;">Editor</span>]] [[User talk:ExclusiveEditor|<sub>Notify Me!</sub>]] 07:50, 27 May 2024 (UTC)
==Drugs==
::What evidence, if any, supports your premise? ←[[User:Baseball Bugs|Baseball Bugs]] <sup>''[[User talk:Baseball Bugs|What's up, Doc?]]''</sup> [[Special:Contributions/Baseball_Bugs|carrots]]→ 07:52, 27 May 2024 (UTC)
Why are [[narcotics|drugs]] so costly. I think heroin costs as much as Rs 1 crore (that's 10 million) per kg [[User:Jon Ascton|<span style="border:1px solid DarkGreen;padding:1px;"><font style="color:White;background:DarkGreen" size="0">&nbsp;Jon Ascton&nbsp;</font></span>]]&nbsp;[[User talk:Jon Ascton|<font color="DarkGreen" size = "0">(talk)</font>]] 03:16, 30 June 2010 (UTC)
::Like many words, people do not always use the term ''universe'' to mean the same thing. Usually you can understand from the context which sense is meant. When people say "'''the''' universe", they usually mean the totality of space and everything in it that we know to exist or can reasonably expect to exist based on what we know. So the spatial extension of the universe is the totality of space; there is no room for another universe.
::Mathematically, you can have several spaces that are in no way connected to each other, so one can imagine there are other universes that occupy other spaces that we have no way of knowing about: we cannot reach them, and voyagers or information from these other universes cannot reach us. Therefore the hypothesis that other universes exist is [[unfalsifiable]] and falls outside the realm of [[Scientific method|science]]. It is a plot device in [[science fiction]], but then there are portals connecting the universes, so from a definitional point of view one might say they are all part of a universe that is somehow compartmentalized.
::Another sense of the term ''universe'', given by the Merriam–Webster dictionary as sense 1c(3), is: "an aggregate of stars comparable to the Milky Way galaxy".<sup>[https://www.merriam-webster.com/dictionary/universe]</sup> That is a complicated way of saying "a [[galaxy]]". Used in this sense, there are maybe a trillion such universes in the [[observable universe]]. They share the space and may [[Interacting galaxy|collide]]. &nbsp;--[[User talk:Lambiam#top|Lambiam]] 09:46, 27 May 2024 (UTC)
:::That sounds like a throwback to when the Milky Way was thought to be "everything". I'm also reminded of a time when I had a short discussion with a fundamentalist Christian who claimed that God was "outside of the universe." I said that the universe, by definition, is everything. So if God exists, then He must be part of the universe. The guy conceded. ←[[User:Baseball Bugs|Baseball Bugs]] <sup>''[[User talk:Baseball Bugs|What's up, Doc?]]''</sup> [[Special:Contributions/Baseball_Bugs|carrots]]→ 21:11, 27 May 2024 (UTC)
:::: I agree with that guy. If you accept the notion of an omnipotent God, then He must be able to create stuff that is not a part of Himself. Just because we "define" the Universe to conform with our limited conceptual ability does not alter that. -- [[User:JackofOz|<span style="font-family: Papyrus;">Jack of Oz</span>]] [[User talk:JackofOz#top|<span style="font-size:85%; font-family: Verdana;"><sup>[pleasantries]</sup></span>]] 21:53, 27 May 2024 (UTC)
:::::{{small|God being omnipotent, It is also able to create Itself. &nbsp;--[[User talk:Lambiam#top|Lambiam]] 09:17, 28 May 2024 (UTC)}}
:</br>
:::: Perhaps he conceded too easily. It's a fairly standard theological view that God exists beyond space and time, and while you personally may use "the universe" to mean "everything that exists", I think it's probably more usual to imagine the universe as being restricted to that which exists in spacetime.
:::: <small>That said, and this is of course a digression, the view you report does not strike me as exactly orthodox ''Christian'' theology, which usually emphasizes God's dual role as both [[immanent]] and [[transcendent]]. If you view God as transcendent only and not at all as immanent, then it seems like you might be a [[deist]] or something. --[[User:Trovatore|Trovatore]] ([[User talk:Trovatore|talk]]) 22:01, 27 May 2024 (UTC) </small>


::You seem to be picturing the 'Big Bang' as an explosion of matter/energy ''into'' a pre-existing infinite space, which still surrounds the (expanding) 'universe' of matter and energy that we are in (so that there could be other 'Big Bang bubbles' somewhere else in that infinite space): this is a fundamental misunderstanding of current astronomical theories. According to mainstream astronomy, The 'Big Bang' was a (rapid) expansion of ''space and time themselves'', more properly understood as the single entity [[Spacetime]], and of the energy (some of which became matter) within them/it. There is, geometrically, nothing 'outside' Spacetime because there is no outside. This is, of course, hard to visualise and grasp. {The poster formerly known as 87.81.230.195} [[Special:Contributions/94.2.67.173|94.2.67.173]] ([[User talk:94.2.67.173|talk]]) 21:15, 27 May 2024 (UTC)
:I don't imagine that the heroin industry has a substantial R&D budget. [[User:Thegoodlocust|TheGoodLocust]] ([[User talk:Thegoodlocust|talk]]) 03:28, 30 June 2010 (UTC)
:::As said by Hank Green (and expanded by Elle Cordova) "there was no up, there was no down, there was no side to side". Search youtube for "big bang poem". --[[User:Khajidha]] ([[User talk:Khajidha|talk]]) ([[Special:Contributions/Khajidha|contributions]]) 11:46, 29 May 2024 (UTC)


:Heroin is highly addictive, which means people are willing to pay such large amounts for it. The costs and risks involved with smuggling and dealing illegal drugs (if you risk many years in prison, you're only going to want to take that risk if you can expect massive profits - it's the risk/return trade-off that is discussed a few sections above) mean it isn't viable to sell it any cheaper. The combination of those two facts means that it is sold at very high prices. (If the first weren't true, it wouldn't be sold. If the second weren't true, it would be sold much cheaper.) --[[User:Tango|Tango]] ([[User talk:Tango|talk]]) 03:54, 30 June 2010 (UTC)


= May 29 =
:Also, since drug dealing is already illegal, dealers tend to have no qualms about using illegal business practices to get rid of competition and charge monopoly rates. Since it's so hard to quit drugs, they can get away with making these rates very, very high. — [[User:DanielLC|DanielLC]] 05:12, 30 June 2010 (UTC)
::Hardly. Drugs are an incredibly competitive market. Certainly they have a low elasticity, but that balances against an incredibly fluid market, unregulated (notwithstanding, of course, the ''massive'' resources spent prohibiting them). Insofar as dealers have monopoly rates it's a function of how able they are to repel competitive dealers, which largely requires illegal activity that attracts much more attention than the actual drug selling does. In short, drug prices are high because interdiction and law enforcement efforts make them that way. That doesn't mean though that they're not a competitive market. [[User:Shadowjams|Shadowjams]] ([[User talk:Shadowjams|talk]]) 08:04, 30 June 2010 (UTC)
:The book [[Freakonomics]] has a chapter devoted to the economics of crack. Its pretty good read on the subject. --[[User:Jayron32|<font style="color:#000099">Jayron</font>]]'''''[[User talk:Jayron32|<font style="color:#009900">32</font>]]''''' 03:45, 2 July 2010 (UTC)


== Elderly digestion ==
== why does my sink solution turn cloudy when I add baking powder to it? ==


Older people have more distended intestines due to loss of muscle tone. It also means food is pushed along more slowly along the digestive tract. Does this mean that per ounce of ingested food, elderly people will extract more nutrients?
I mean dissolved sodium bicarbonate should be transparent... right? Or does some of it end up not dissolving? [[User:John Riemann Soong|John Riemann Soong]] ([[User talk:John Riemann Soong|talk]]) 03:42, 30 June 2010 (UTC)


P.S. I had HUGE chunk of dishwater. Surely that's enough water to dissolve a tablespoon or two of sodium bicarbonate? Would the alkaline conditions precipitate any ions commonly found in food? [[User:John Riemann Soong|John Riemann Soong]] ([[User talk:John Riemann Soong|talk]]) 03:46, 30 June 2010 (UTC)
Of course, if this causes them to eat less, it may not mean more calories absorbed. [[User:Imagine Reason|Imagine Reason]] ([[User talk:Imagine Reason|talk]]) 09:24, 29 May 2024 (UTC)
:I don't know if any of the premises of your argument are correct. But consider this; for about half a billion years our ancestors have had a digestive system. Extracting every usable bit of nutrients from our food has been under strong selection the whole time, because starvation kills and food is limited. Why then would a malfunctioning elderly system do better? <span style="font-family: Cambria;"> [[User:Abductive|<span style="color: teal;">'''Abductive'''</span>]] ([[User talk:Abductive|reasoning]])</span> 18:54, 29 May 2024 (UTC)
::There is an evolutionary pressure to extract every usable bit of nutrients from our food. There's also an evolutionary pressure to keep the power-to-weight ratio of the digestive system high. Wasting some nutrients to keep the digestive system light may be benificial. I'm not suggesting I disagree with your conclusion. [[User:PiusImpavidus|PiusImpavidus]] ([[User talk:PiusImpavidus|talk]]) 19:54, 29 May 2024 (UTC)
:::The human digestive system is far from perfect in extracting calories. [[User:Imagine Reason|Imagine Reason]] ([[User talk:Imagine Reason|talk]]) 21:52, 31 May 2024 (UTC)


== Elvish astronomy ==
:Do you live in a hard-water area? If your tap water has a high level of calcium or magnesium ions, then those ions could react with the baking soda to form insoluble carbonates, which could make the water cloudy. FWIW [[Special:Contributions/67.170.215.166|67.170.215.166]] ([[User talk:67.170.215.166|talk]]) 09:22, 30 June 2010 (UTC)


Considering [https://physics.stackexchange.com/questions/122785/could-legolas-actually-see-that-far Legolas' canonical feats], what would be his naked-eye [[limiting magnitude]], assuming seeing conditions in which the average human would get 6.0?
(edit conflict)
:In ionic solutions, the ions are fully disassociated, i.e. if you dissolve in water the salts sodium chloride and calcium iodide, you also get sodium iodide and calcium chloride. What else was in the water? If there was a soluble calcium salt (say [[calcium chloride]]), then the insoluble [[calcium carbonate]] will be produced, which will [[precipitate]] out of solution. (Calcium bicarbonate decomposes to calcium carbonate). Do you live in a [[hard water]] area? Hard water contains calcium salts. [[User:Csmiller|CS Miller]] ([[User talk:Csmiller|talk]]) 09:43, 30 June 2010 (UTC)


(The main reason for this question is that in ''Morgoth's Ring'' an Elvish name for Neptune is given. Though I suppose they might've been using palantíri as telescopes.) [[User:Double sharp|Double sharp]] ([[User talk:Double sharp|talk]]) 09:36, 29 May 2024 (UTC)
== reason of raining ==


:Perhaps elves have [[eagle eye]]s; Tolkein is silent on the issue. [[User:Alansplodge|Alansplodge]] ([[User talk:Alansplodge|talk]]) 11:15, 29 May 2024 (UTC)
What is the reason of raining ? <span style="font-size: smaller;" class="autosigned">—Preceding [[Wikipedia:Signatures|unsigned]] comment added by [[Special:Contributions/203.194.98.218|203.194.98.218]] ([[User talk:203.194.98.218|talk]]) 04:02, 30 June 2010 (UTC)</span><!-- Template:UnsignedIP --> <!--Autosigned by SineBot-->
::He does kind of imply at least that the naked-eye limiting magnitude is greater for Elves than for Men. In ''The Nature of Middle-Earth'', "Dark and Light" it is written: {{tq|The Quendian imagination of the shape of Arda and of the visible Heaven (Menel) above it, was due to the acute minds of a people endowed with sight far keener than the human norm.}} Though this is for the Round World version, in which the Sun and Moon already exist from the beginning. As for quantitative figures, I guess we're stuck with the data point of Legolas counting 105 horsemen from a distance of 24 km.
:See [[rain]] for an explanation. [[User:Looie496|Looie496]] ([[User talk:Looie496|talk]]) 04:58, 30 June 2010 (UTC)
::Actually, it occurs to me that (perhaps more interestingly than limiting magnitudes), Elves really ''ought'' to be able to resolve the Galilean moons of Jupiter. This provides an alternate solution to the longitude problem, at least if you take the Round World versions. :) [[User:Double sharp|Double sharp]] ([[User talk:Double sharp|talk]]) 15:20, 29 May 2024 (UTC)
:::The Galilean moons would be a cinch for an elf. I had a (human) friend who could do this (he was tested on it several times). Not being particularly interested in astronomy, he only found out in his adult years that this was not usual for most people. {The poster formerly known as 87.81.230.195} [[Special:Contributions/94.2.67.173|94.2.67.173]] ([[User talk:94.2.67.173|talk]]) 19:42, 29 May 2024 (UTC)
::::Hmm, probably also the crescent of Venus should be resolvable for them. This perhaps has implications on the shapes of the Silmarils. (Although in the Round World Version, Venus already exists beforehand, and its identification with Eärendil is said to be mythologising. From the same essay I quoted: {{tq|Certain stars (no doubt those we call planets) and among them especially Venus, which they called ''Elmō'' (and later mythologically ''Eärendil''), they early observed were “wayward” and altered their places with regard to the “farstars” (fixed stars). These they called companions of the Sun and thought them quite small heavenly bodies – derived from the Sun.}}) [[User:Double sharp|Double sharp]] ([[User talk:Double sharp|talk]]) 02:31, 30 May 2024 (UTC)
:The limiting magnitude can be increased with a sharper view. The sharper the view, the smaller the area of the detector (retina) on which the light falls. The background light is fixed per unit of surface area of the detector, so with the signal on a smaller area, less background competes in this area, increasing signal-to-noise. A sharper view can also help to take a faint object out of the glare of a nearby bright object; relevant to see the Galilean moons.
:The other way to increase the limiting magnitude is by increasing sensitivity. No matter how sharp your eyes are, you need a couple of photons before you can see anything. The more photons you detect, the lower the relative [[Poisson noise]]. The sensitivity can be increased by (A) a better detector, detecting a larger fraction of the incoming photons; (B) larger aperture, i.e. a bigger pupil; (C) increasing integration time. Many nocturnal animals (and elves may be somewhat nocturnal) have better, more sensitive detectors, although at a price. The [[tapetum lucidum]] found in many animals reduces resolution somewhat; some species sacrificed colour vision for better low-light vision. Bigger eyes help to see better, but although Tolkien often writes that Elven eyes are keen and fair, he never writes (AFAIK) that they're big. Maybe elves can at will increase the integration time of their eyes. For humans it's fixed at several centiseconds, but if elves can boost it to a full second (they would largely loose the ability to detect motion), seeing Neptune shouldn't be too hard. Still takes a lot of dedication and patience to find out which of those tens of thousands of faint stars slowly moves, but patience is something you should have if you live forever. Ents might disagree. [[User:PiusImpavidus|PiusImpavidus]] ([[User talk:PiusImpavidus|talk]]) 20:55, 29 May 2024 (UTC)
::Why would Ents disagree? They have loads of patience (though it is unclear if they are immortal or just extremely long-lived). [[User:Clarityfiend|Clarityfiend]] ([[User talk:Clarityfiend|talk]]) 22:23, 29 May 2024 (UTC)
:::Ents might disagree with the statement that elves have patience. [[User:PiusImpavidus|PiusImpavidus]] ([[User talk:PiusImpavidus|talk]]) 11:07, 30 May 2024 (UTC)
::Thanks for the details! I guess I'm personally more inclined now towards explaining the Neptune thing by an Elvish invention of the telescope.
::Or maybe the Valar told them where to look, noting that Neptune's magnitude is actually brighter than the most extreme reports of naked-eye viewing of stars. It would be a lot easier to find Neptune if you already know where it is, than to find which of those myriad faint stars is slowly moving. In NoME, ''Elvish Reincarnation'' implies that the Eldar were informed about [[isotope]]s by the Valar, so this isn't unreasonable in-universe. (Though finding that passage makes me amused by the thought of seeing Galadriel's NMR spectra.) [[User:Double sharp|Double sharp]] ([[User talk:Double sharp|talk]]) 04:23, 30 May 2024 (UTC)
::In canon Men are sometimes mistaken for Elves, so there must not be any gross difference in eye size. [[User:Tamfang|—Tamfang]] ([[User talk:Tamfang|talk]]) 21:37, 30 May 2024 (UTC)
:::Assuming a comparable physiology, an Elven retina may pack more sensitive photoreceptor cells, while the lens may have better optical qualities. &nbsp;--[[User talk:Lambiam#top|Lambiam]] 06:08, 31 May 2024 (UTC)
I wouldn't take it for granted that the Solar System outside of Middle Earth is the same as the real one. According to ''[[The Silmarillion]]'' Earth (or "Arda") was explicitly created by God and smaller deities with their omnipotent powers, and so were the peoples of that world (Elves, Men, Dwarves, and surely Hobbits and Ents, too). And the part that would drive mad the astronomers reading that book, the Sun and the Moon were also created by those beings... ages ''after'' the creation of Arda, and ''after'' life existed on that world. [[User:Cambalachero|Cambalachero]] ([[User talk:Cambalachero|talk]]) 13:00, 31 May 2024 (UTC)
:Indeed. But Tolkien planned in his later years to make an altered [[Round World Version of Tolkien's legendarium|Round World Version]] in which the Sun and Moon exist from the beginning of Arda. In that case the Two Trees simply preserve their light as it was before they were tainted by Melkor. As I quoted above, these late rewrites imply that the planets in Tolkien's world are the same ones that we have. :) [[User:Double sharp|Double sharp]] ([[User talk:Double sharp|talk]]) 11:05, 1 June 2024 (UTC)
::Hmm, curious where you heard about these supposed intended revisions to create a lore more consistent with actual astral bodies. I'm a little skeptical without seeing a source on this, just because it doesn't seem to add up with other well-established details. Bear in mind that every element of the legendarium that bears light on the creation and cosmology of Eä was published after J.R.R. Tolkien's death. Christopher Tolkien finished ''The Silmarillion'' by trying to faithfully patch together the content (and fill in gaps himself) using an express desire to have the final work reflect what his father, the original author, would have intended. Nobody would have been in a better position than the younger Tolkien (who made the completion of the legendarium a substantial part of his own life's work and had the fullest access to the existing materials) to know what the elder intended in this respect, and he would have had every opportunity to cause the final work to reflect it, as he did with countless other details. {{pb}} Further, thematically it just doesn't seem to fit: the trees are so fundamental to the cosmonogy of the legendarium, as well as its evolution and eschatology. The trees are part of the more gnostically "pure" version of the world after it was sung into existence out of the higher pleroma by the music of Ainur. The fall of the trees, though instigated by the machinations of Melkor and Ungoliant, are thematically (and arguably psuedo-naturally) the result of an inevitable and unarrestable trend of the world trending away from the direct influence of the Valar and towards a world more defined by physicality and all the ills that come with it. {{pb}} All of this being the result of Melkor's discordant notes, which Eru permitted to remain a part of the song of creation. First the Valar leave Middle Earth and retire to Valinor, and as time wears on, begin making less and less in the way of even indirect influence over it. Next, the Trees are destroyed and the First Age begins. Later the world is reshapped such that Valinor is not even entirely on the same physical plane as the rest of Arda, and reachable only via the Straight Path. The elves diminish and go into the West, returning ultimately to Valinor, the magic and grace of their realms failing and converting them into mundane lands. Meanwhile the light of the trees persists in the silmarils and the phial of Galadriel, bitter sweet echoes of a purer but irretrievable age. Magic fades and Illuvatar's second (and less ethereal) children, humans, inherit the world. {{pb}} You see what I mean? The idea of the world's inclination to a state defined increasingly more by a base, more purely physical state and away from the direct influence of the spiritual animus that gave birth to it is baked into the narrative and the lore, from start to finish. And the trees are the ultimate symbol of the starting point (or at least the start after the music was finished and the Ainur descended into Arda). Besides, the sun and the moon predating the round world just doesn't make much sense: a flat Arda wouldn't be able to rotate on is axis. Mind you, not to imply that there's in way to get the legendarium's cosmology to work with actual astrophyics. Which Tolkien very well knew: all indications are that this was a part of the point. But introducing the sun and the moon before the First Age just makes the discontinuities more obvious and intrusive.{{pb}}All that said, very curious to see where this comes from originally. Certainly there is no shortage of matters that Tolkien went back and forth on over his decades of revisions of the relevant works, nor issues where his son had to make best-guess efforts in choosing among the disparate versions of events. But personally, I tend to doubt that Tolkien seriously considered this particular shake-up. Tonally and in terms of continuity, it just doesn't add up. -sighs a sigh of glutted satisfaction, having sucked out all of the fun the subject matter and wrecked it as surely as a giant spider sucking the magic out a world tree and poisoning it.- ''[[User:Snow Rise|<b style="color:#19a0fd;">S</b><b style="color:#66c0fd">n</b><b style="color:#99d5fe;">o</b><b style="color:#b2dffe;">w</b><b style="color:#B27EB2;">Rise</b>]][[User talk:Snow Rise|<sup><b style="color:#d4143a"> let's rap</b></sup>]]'' 02:51, 3 June 2024 (UTC)
:::{{ping|Snow Rise}} There is a good summary [https://realelvish.net/2024/04/14/it-always-had-been-a-vast-globe/ here], though the primary sources (JRRT's texts) are still in ''[[Morgoth's Ring]]'' (ed. Christopher Tolkien) and ''[[The Nature of Middle-earth]]'' (ed. Carl Hostetter). A scholarly paper about it is [https://dc.swosu.edu/mythlore/vol42/iss1/10/ here].
:::A slightly shorter summary: pre-ROTK, JRRT produced a version of the ''Ainulindalë'' with the changes to the cosmology. Melkor seizes part of the Earth to make the Moon for his stronghold, before the Valar cast him out of it and cleanse it. For the time being, it was only an experiment.
:::But in the late 1950s, JRRT came to believe that the making of the Sun and Moon was too "astronomically absurd" to write in an age when most people believe that the Earth is spherical and is more or less like an island in space. So he came up with a new concept: the "Flat World Versions" are traditions that were handed down by the Númenoreans and then in Arnor and Gondor, that are inescapably blended and confused with Mannish myths. The Elves had their own lore from Valinor that was astronomically and geologically in accord with what we know instead.
:::JRRT describes the new cosmology across some essays published in the aforementioned collections and in an interview. The Sun and Moon were created together with the Earth, and originally they had the Primeval Light, and what Melkor did instead was corrupt them. Middle-earth was then twilit, because Morgoth darkened the Earth with clouds, such that the stars and moon were invisible and the Sun was only a dim twilight (something like the real Venus without the greenhouse effect, I might add). The significance of the Two Trees is that only ''there'' was the Primeval Light preserved, and Varda domed Valinor over to keep Morgoth's corruption out and have it only be lit by the stars. The world was also round ''from the beginning'', but you could not circumnavigate it before the drowning of Númenor because Aman would block the way. The Númenorean Catastrophe removes the inhabitants of Aman from the physical world, though the landmass remains and becomes America after significant geographical upheaval. Thus JRRT writes in these notes {{tq|Aman and Eressëa would be the memory of the Valar and Elves of the former land.}}
:::Crucially, in 1966 ''The Hobbit'' was slightly edited in accordance with this new revision: where once read {{tq|In the Wide World the Wood-elves lingered in the twilight before the raising of the Sun and Moon}} (completely correct per the Flat World version), there now read {{tq|In the Wide World the Wood-elves lingered in the twilight of our Sun and Moon}}. JRRT's late texts published and edited by Christopher in ''[[The Peoples of Middle-earth]]'' sometimes also imply this: the new description of Fëanor's burning of the ships specifies that it was done "in the night", and that "In the morning the host was mustered", which makes no sense in the old cosmology because the Sun wouldn't yet have first risen; and Thingol's throne room of Menelrond is supposed to be based on the domes of Varda (which don't exist before the changes). The heraldry JRRT drew in the 1960s for the House of Finwë is a "Winged Sun", which also makes more sense in the new cosmology because Finwë otherwise could not have seen a Sun that only rose after he died. So the evidence is clear that the elder Tolkien really seriously intended this change.
:::As for why it was not adopted by Christopher: Christopher's commentary on these texts indicates that he seems to have thought it a bad idea on the part of his father. I think Christopher made a sensible call, as going on with the intended changes would require much more editorial intervention than leaving things as they were: it does not seem that JRRT ever ''finished'' the planned rewrites. But from my perspective as a fan, the whole idea is fascinating and allows the amusing conceit of trying to figure out what was really going on behind the scenes, which is why I asked the initial question about how astronomically plausible the reworking was in one aspect (the visibility of Neptune). [[User:Double sharp|Double sharp]] ([[User talk:Double sharp|talk]]) 03:45, 3 June 2024 (UTC)
::::Fascinating! Just when I thought the paratextual narrative of this particular piece of literature couldn't get more complicated! Personally I like the ultimate approach adopted (even beyond the concern you rightly point out of it requiring more intervention--I still think I prefer the narrative irrespective of whether you hypothesize a scenario in which J.R.R. could make the adjustments himself). The Age of the Trees being treated as literal (or at least facially literal from the perspective of the Silmarillion/legendarium's narrative; all of the books are presented to varying degrees as possibly unreliable ancient historical text, after all) just feels very at home with both the broader cosmogony as well as the thematics of the overall work. But then, I have revisited those books so many times over the years (at least compared to my once, ''maybe'' twice and done policy for most literature: soooo many things to read!) that I may have some bias! ''[[User:Snow Rise|<b style="color:#19a0fd;">S</b><b style="color:#66c0fd">n</b><b style="color:#99d5fe;">o</b><b style="color:#b2dffe;">w</b><b style="color:#B27EB2;">Rise</b>]][[User talk:Snow Rise|<sup><b style="color:#d4143a"> let's rap</b></sup>]]'' 16:54, 3 June 2024 (UTC)
:::::{{ping|Snow Rise}} I kind of think of JRRT's problem as being that in seeking increased verisimilitude, he'd gone from wanting to write a mythology to wanting to write a science fiction novel. Only, instead of starting a new science fiction novel and leaving the mythology as it was in its almost-complete state, he ended up trying to turn his mythology into a science fiction novel, with predictable results. But I suppose the whole narrative was too dear to his heart for him to come up with another like it, just like the Silmarilli were to Fëanor. Which is understandable: I share your obsession, and how much greater must it have been for the author? :)
:::::With that said: even in this take, the Age of the Trees would still be literal in the sense that there were luminescent trees in Valinor that got killed by a giant spider. They would only not be literal in the sense that the Sun would still be shining (in its form post-tainting by Melkor) over Middle-earth. Only in the northwest (over Beleriand) is there a volcanic winter due to the fumes of Thangorodrim – and from that, we can see where the idea of the Sun first rising later comes from as an in-universe myth. [[User:Double sharp|Double sharp]] ([[User talk:Double sharp|talk]]) 10:42, 4 June 2024 (UTC)


== V-tails ==
= May 30 =


== Volume of honey in a bee nest ==
Why are [[V-tail]]s so stealthy, since when top of the aircraft faces the radar it forms a right angle or [[corner reflector]]? --[[User:The High Fin Sperm Whale|The]] [[User talk:The High Fin Sperm Whale|High]] [[Special:Contributions/The High Fin Sperm Whale|Fin]] [[Special:EmailUser/The High Fin Sperm Whale|Sperm]] [[User:The High Fin Sperm Whale/Sandbox|Whale]] 05:05, 30 June 2010 (UTC)


:Are they terribly stealthy? Anyway, how often do you think the top of an aircraft is going to be facing radar? AFAIK we don't have satellite based radar. [[User:Thegoodlocust|TheGoodLocust]] ([[User talk:Thegoodlocust|talk]]) 05:38, 30 June 2010 (UTC)
What is the average volume of honey in a bee nest in the wild? I was able to find information on the average volume of a bee nest, but I know that not all of that volume is honey, of course. Thank you! [[User:HeyArtemis|HeyArtemis]] ([[User talk:HeyArtemis|talk]]) 07:49, 30 May 2024 (UTC)
:[[Apis mellifera|It]] [[Apis dorsata|varies]] [[Apis cerana|alot]]. [[Honey#Production]] has a number for ''Apis mellifera''. [[User:Sean.hoyland|Sean.hoyland]] ([[User talk:Sean.hoyland|talk]]) 08:39, 30 May 2024 (UTC)


== Does light decay? ==
::AFAIK V-tails reduce RCS when viewed ''from the side'' by avoiding the right-angle reflector formed by the conventional rudder/tailplane combination. BTW, in most V-tailed stealth aircraft (e.g. [[F-117]], [[F-22]], [[YF-23]]) the [[ruddervators]] are mounted at an angle ''other'' than 90 degrees. FWiW [[Special:Contributions/67.170.215.166|67.170.215.166]] ([[User talk:67.170.215.166|talk]]) 09:28, 30 June 2010 (UTC)


Let's say that an object, such as a star, emits a beam of light and it moves across the space. It goes at the speed of light and, unless it reaches an opaque object, it would keep going... for how much time? Forever? Or is there a point when light would simply dimish and disappear? [[User:Cambalachero|Cambalachero]] ([[User talk:Cambalachero|talk]]) 19:53, 30 May 2024 (UTC)
:::Yes, the critical point here is that ''from certain directions'', the reflections are attenuated. Designers of these aircraft "estimate" which view angles are most likely during a standard flight / sortie during the high-level system-design, when things like "preferred combat sortie flight altitude," "cruising speed," and "maximum allowable proximity to hostile RADAR" are still subject to re-design. As these parameters solidify, flight profiles emerge that indicate that, for example, the aircraft might always be viewed from the side at an angle of, say, 15 to 45 degrees elevation; then the designers then attempt to make such a viewing angle the ''most'' stealthy (attenuated reflection). Pilots and mission planners are instructed to design flight profiles to keep the aircraft within its "best-performance" spec. In practice, because aircraft specs are hard to change once they are built, and combat flight profiles are subject to operational needs, the aircraft may be used during combat in a "sub-optimal" stealth regime; but it's still ''probably'' more stealthy than an aircraft that wasn't designed carefully with RADAR reflectivity in mind. What this means, on average, is that combat flight profiles that get the aircraft closer to hostile zones will result in statistically fewer detections; and thus, statistically fewer downed aircraft. Historically, especially after the fall of the USSR, the development of [[stealth aircraft]] went hand-in-hand with complete and total [[air superiority]] during most combat operations; so these kinds of design considerations are subject to fierce debate in terms of how economically effective they are. But if we were fighting an air-war on the scale of, say, World War II, when [[Anti-aircraft_warfare#World_War_II|anti-aircraft fire could down 10% of the planes on any given mission]], we would be fighting tooth-and-nail to improve those statistics. Every fraction of a dB of attenuation corresponds to a few extra moments of non-detection, and thus a few extra moments of tactical advantage. <small>I could [[WP:SOAP|ruminate extensively]] on the futility of these sorts of optimizations with respect to modern warfare, but I won't...</small> [[User:Nimur|Nimur]] ([[User talk:Nimur|talk]]) 17:20, 30 June 2010 (UTC)
: No.
: It may react with matter, if it encounters any. A flux of many photons will spread out to a larger volume and so the intensity (number of photons passing through an area) will diminish in accord with the [[inverse square law]]. But light passing through a vacuum does not 'decay' or have a limit on its range.
: BTW, this theory that light can only travel a few thousand miles before 'running out' is part of flat earther canon for some models, as an explanation of how nighttime happens. But then they're flat earthers. [[User:Andy Dingley|Andy Dingley]] ([[User talk:Andy Dingley|talk]]) 20:11, 30 May 2024 (UTC)
::The idea of [[tired light]] used to be somewhat popular but has been entirely discarded by now. --[[User:Wrongfilter|Wrongfilter]] ([[User talk:Wrongfilter|talk]]) 20:22, 30 May 2024 (UTC)
:::I'm afraid it is a zombie idea.<sup>[https://www.uottawa.ca/about-us/media/news/new-research-suggests-our-universe-has-no-dark-matter]</sup> &nbsp;--[[User talk:Lambiam#top|Lambiam]] 05:58, 31 May 2024 (UTC)
:I guess the apparent reality that things in motion continue in motion forever by default, if unperturbed, seems a bit unnatural when you live in a macroscopic world. [[User:Sean.hoyland|Sean.hoyland]] ([[User talk:Sean.hoyland|talk]]) 07:32, 31 May 2024 (UTC)
::Correct. If that wasn't the case, we wouldn't be able to see stars that are too far away. We are currently seeing stars so far away that they aren't just stars. They are galaxy or similar objects so far away that they show up as one little blob of light. The limit is not how far light can travel before pooping out. It is how old the stars are. For example, a 100 year old star that is 90 light years away would not be visible becuase the original light hasn't reached us yet (ignoring the complication of direction of relative movement). So, the argument is that the night sky should be flooded with light from all directions. And, it is. It only looks black in a relative sense. [[Special:Contributions/12.116.29.106|12.116.29.106]] ([[User talk:12.116.29.106|talk]]) 14:51, 4 June 2024 (UTC)
:::I imagine it would be much worse than not being able to see stars. If excitations of quantum fields got tired we would be in big trouble. [[User:Sean.hoyland|Sean.hoyland]] ([[User talk:Sean.hoyland|talk]]) 15:05, 4 June 2024 (UTC)


= May 31 =
== Chromosome Location ==
==Is redshift calculated differently for different spectra?==
I ask because I came across an article, [[TXS 1545-234]], in the course of regular gnoming. The article claims this radio galaxy to be one of the most distant known objects, but its redshift is only around {{math|z}} = 2.754 (which I take to be measured from radio emissions). It does appear in [https://ned.ipac.caltech.edu/level5/Sept08/Miley/Miley9.html this source] {{nowrap|({{para|date}}}} at least 2006), and I'm wondering whence the claim of such great distance paired with such pedestrian redshift. I'm not able to understand our article [[Redshift]]. Also, if anyone has any ideas about how to de-orphan the article linked, please do have at. [[User:Folly Mox|Folly Mox]] ([[User talk:Folly Mox|talk]]) 11:31, 31 May 2024 (UTC) {{small|{{ins|edited 11:52, 31 May 2024 (UTC)}}}}
:It is certainly not one of the most distant galaxies known. The author of the article, {{ping|Galaxybeing}}, should explain why they think it is. --[[User:Wrongfilter|Wrongfilter]] ([[User talk:Wrongfilter|talk]]) 12:12, 31 May 2024 (UTC)
::They appear to have made a similar claim at [[Special:Permalink/1226518102|MRC 0406-244]] ({{math|z}} = 2.44) although thankfully not at another recent creation, [[Special:Permalink/1226388188|QSO J0100-2708]] ({{math|z}} = 3.52). [[User:Folly Mox|Folly Mox]] ([[User talk:Folly Mox|talk]]) 12:57, 31 May 2024 (UTC)
:The redshift, a [[dimensionless quantity]], is the same for the whole spectrum emitted by an object. Compared to TXS 1545-234, [[JADES-GS-z13-0]] is thought to be more than three times as far away from us. &nbsp;--[[User talk:Lambiam#top|Lambiam]] 12:37, 31 May 2024 (UTC)
::Thanks both for your answers, and confirming my suspicion that the claim was merely incorrect. [[User:Folly Mox|Folly Mox]] ([[User talk:Folly Mox|talk]]) 13:00, 31 May 2024 (UTC)
:A redshift of 2.754 is pretty far away, but stating it's one of the most distant known objects is overstating it a bit. Distances to far-away galaxies are normally expressed in redshift, as redshift is directly observable, in contrast to distance, which depends on a model of the expansion of the universe. But when using redshift as a distance measure, one has to keep in mind that it's highly non-linear. Also, distance is a bit of a strange concept when dealing with these cosmologically distant objects. Are we talking about the distance today, or at the time the light was emitted, or the distance travelled by the light? That last at least has some relevance as it translates to the time that the light has travelled and therefore when it was emitted. JADES-GS-z13-0 may be several times farther away today than TXS 1545-234, but most of that is thanks to the expansion of the universe after the light was emitted. When the universe was young and small (although still infinite), it expanded fast in absolute numbers (percentage per year). In light travel distance, the difference isn't so much.
:These objects can be studied to learn more about the early universe. For that, knowing the distance to us is not so important; we want to know about the distance (or time) to the Big Bang. At some point, distances (times) to the Big Bang are known more accurately than distances (times) to us. In any case, the redshift tells us immediately that the universe expanded by a factor of five between the times when the light of JADES-GS-z13-0 was emitted and when the light of TXS 1545-234 was emitted. That puts JADES-GS-z13-0 a lot closer to the Big Bang, although only a small fraction farther from us. [[User:PiusImpavidus|PiusImpavidus]] ([[User talk:PiusImpavidus|talk]]) 11:05, 1 June 2024 (UTC)
::On a tangent but an amusing use (abuse?) of "red shift": chemists use red shift generically to mean "moves to lower energy". Even in the IR part of the spectrum, the term red shift would be used to describe the shift of a band to lower E, say 2000 to 1950 cm-1. This language is of course strange because, formally speaking, a shift toward red for an IR band would mean a shift to higher E. Just sayin'.--[[User:Smokefoot|Smokefoot]] ([[User talk:Smokefoot|talk]]) 17:11, 1 June 2024 (UTC)
:::Radioastronomers apply the same "abusive" terminology. We see this linguistic phenomenon also in uses of the verb "to dial", as in the advise to "<u>dial</u> 9-1-1 for any emergency" given to users of smartphones with touch screens. (Using the original rotary sense of the verb in connection with the casual parlance of [https://www.nbcnews.com/politics/politics-news/rudy-giuliani-butt-dials-nbc-reporter-heard-discussing-need-cash-n1071901 "butt dial"] results in the unfortunate mental image of Giuliani twerking.) Other examples are referring to cotton bed sheets as "linens", or (in the US) to stainless-steel knives, forks and spoons as "silverware", and the computer-graphics terminology calling a screen region a "canvas". I think there is a learned term for this phenomenon if the sense of a word getting abstracted from the physical embodiment after which it was originally named. &nbsp;--[[User talk:Lambiam#top|Lambiam]] 05:29, 2 June 2024 (UTC)
:::[[File:JADES-GS-z13-0.png|thumb|JADES-GS-z13-0]]
:::Astronomers talk about redshift if it goes to longer wavelengths, in radio, IR, UV etc., and about blueshift if it goes to shorter wavelengths, in radio, IR, UV etc. Things can also redshift past red. See how red that distant object in this picture is? It's a feature coming from UV, shifted to IR. The [[Lyman-alpha]] absorption line is at 121.6&nbsp;nm, here broadened into a [[Gunn–Peterson trough]], redshifted to 1.6&nbsp;μm, between the F150W and F200W filters of the camera in JWST. It's how they make a first estimate of the redshift, based on broadband images. A precise number follows later from spectroscopy, but takes far more observing time, so this is only done for the most promising targets. [[User:PiusImpavidus|PiusImpavidus]] ([[User talk:PiusImpavidus|talk]]) 10:18, 2 June 2024 (UTC)


== Physics problem ==
Which chromosome isresponsible for the DNA that codes for our brain and memory, whether someone is naturally better at Math or better at art? Is it all encoded on one chromosome or is spread out among multiple chromosomes? Also, how is it posible to know which part of a chromosome or the genetic material encoded on say chromosome 6 for example has the DNA for brown eyes, how do scientists know this? <span style="font-size: smaller;" class="autosigned">—Preceding [[Wikipedia:Signatures|unsigned]] comment added by [[Special:Contributions/71.143.241.161|71.143.241.161]] ([[User talk:71.143.241.161|talk]]) 07:24, 30 June 2010 (UTC)</span><!-- Template:UnsignedIP --> <!--Autosigned by SineBot-->


A shop sign is made of a panel that protrudes slightly from the wall on which it is hung, forming an angle of 5° with it. It is 0.74 m tall and has a mass of 8.9 kg. The upper side of the panel is attached to the wall by two cables, one from the right side and one from the left side. Find the tension of the 2 cables [[Special:Contributions/78.211.54.11|78.211.54.11]] ([[User talk:78.211.54.11|talk]]) 19:40, 31 May 2024 (UTC)
:No one knows what codes for brain and memory, and they don't know what codes for brown eyes. At best all we can say is that a bunch of people with a similar trait all had this piece of DNA in common. But often you'll have people with that trait that don't have the gene, and sometimes people with the gene still don't have the trait. It's rare to actually be able to "read" the DNA and understand what it does. Usually all you can say is that this piece seems to be common to these traits, without understanding any of the details - and without even understanding what other DNA might have an effect too. [[User:Ariel.|Ariel.]] ([[User talk:Ariel.|talk]]) 11:54, 30 June 2010 (UTC)


:<small>Original in Italian, now edited by IP to English.</small>Translates as: "A shop sign consists of a panel that protrudes slightly from the wall on which it hangs, forming an angle of 5° with it. It is 0.74 m high and has a mass of 8.9 kg. The upper end is attached to the wall by two cables, one on the right side and one on the left side. Find the tension in the 2 cables."
::No [[DNA]] ''directly'' codes for traits, features, aptitudes, or any such things. Bits of DNA (usually called [[genes]]) generally code (directly or via the intermediary of [[RNA]]) for the production or [[Expression (genetics)|expression]] of [[proteins]], which then interact in often extremely complicated ways (including promoting or blocking the actions and/or effects of other DNA or proteins). The end results of all these RNA and protein expressions (including, crucially, their timing) and interactions may usually, often or sometimes (depending on the context of all the other interactions going on) result in the physical manifestation of certain particular traits, features, etc of the human body and brain: saying that "gene x codes for feature y" is a frequently used shorthand expression, but glosses over the complexities and can be misleading.
:IP editor: as it says at the top of this page, we don't answer homework questions and what we do answer should preferably be asked in English. [[User:Michael D. Turnbull|Mike Turnbull]] ([[User talk:Michael D. Turnbull|talk]]) 19:46, 31 May 2024 (UTC)
::Sometimes a functioning gene may not be contiguous: that is, it may be split into two (or more?) physically unconnected parts. Sometimes a part of a gene "for x" may also interact with some or all of another gene to result in "y", so the same short stretch of DNA may be expressing two (or more?) different proteins at the same or different times; also, the same protein may do two (or more?) different things at different times, or at the same time in different places, depending on the other proteins present there.
::{{small|Asking homework questions also causes tension. ←[[User:Baseball Bugs|Baseball Bugs]] <sup>''[[User talk:Baseball Bugs|What's up, Doc?]]''</sup> [[Special:Contributions/Baseball_Bugs|carrots]]→ 01:15, 1 June 2024 (UTC)}}
::Things like higher mental properties and aptitudes are ''very'' difficult to correlate with specific bits, or combinations of bits, of DNA - the concept of [[Emergence|emergent properties]] may be useful to consider in this regard.
::Doesn't the tension depend on the angle the cables (idealized as straight line segments) make with the panel? &nbsp;--[[User talk:Lambiam#top|Lambiam]] 02:53, 1 June 2024 (UTC)
::Chromosomes are merely a way of packaging all the genes into manageable 'chunks' that are less fragile, less unwieldy, and easier to copy than if they were all in one 'chunk' - a single chromosome, which the earliest life forms with fewer genes presumably started out with. It's possible for a single chromosome to split into two, or two to merge into one, without this preventing successful reproduction by the individual concerned with an 'unmodified' mate. For example, Chimpanzees and Bonobos have 48 chromosomes instead of Humans' 46 because around 1 million years ago (after the split some 4-8 million years ago between the Human and C/B populations) two chromosomes in a human individual joined up to form one larger one which, after spreading through the human population, is now our [[Chromosome 2 (human)|Chromosome 2]]. Similarly, a human family has recently been identified in which 2 more chromosomes have merged giving a complement of only 44, though all (or nearly all) of the genes involved are still present, just differently packaged. [Sorry, can't re-find the reference for this. Anyone?] Migration of individual genes from one chromosome to another is, I believe, not at all uncommon, and minor losses (or duplications) of one or several genes in a chromosome are pretty common.
:::Correct. The answer to the question as asked is a curve of tension vs cable length or vertical location. [[User:Greglocock|Greglocock]] ([[User talk:Greglocock|talk]]) 22:56, 1 June 2024 (UTC)
::The approximate position or near-exact identity of a particular gene "for" x has in the past been worked out by looking at people with and without x, and with and without various bits of chromosomes, and finding the correlations. Now that [[gene sequencing]] has, very recently, become much easier and cheaper and the entire human genome (that is, those of a handful of individual humans) [[Human Genome Project|has been sequenced]], this field of knowledge is moving more rapidly than laypersons (like myself) can keep up with. [[Special:Contributions/87.81.230.195|87.81.230.195]] ([[User talk:87.81.230.195|talk]]) 14:05, 30 June 2010 (UTC)
[[File:The hanging sign for 'Fieldys' public house - geograph.org.uk - 5284520.jpg|thumb|left|Is it like this? [[User:Graeme Bartlett|Graeme Bartlett]] ([[User talk:Graeme Bartlett|talk]]) 22:42, 1 June 2024 (UTC)]][[File:Tension of cable in hanging sign.svg|thumb|or is it like this? [[User:Graeme Bartlett|Graeme Bartlett]] ([[User talk:Graeme Bartlett|talk]]) 22:22, 1 June 2024 (UTC)]]
{{clear}}
[[File:Statica-balk1.png|thumb|100px|like this but not equilateral]]
In my interpretation of the problem – which is not necessarily the intended one – the lower edge of the panel is attached to the wall and the panel can rotate along that edge, like in the "or is it like this?" diagram, which has an exaggerated thickness for the panel. Unlike that diagram, the panel does not stick out at a 90° angle but is standing almost upright. Also, the cables are attached to the upper edge. So it is more like the situation here to the right, but instead of a bottom 60° angle we have a 5° angle. Not enough info has been given to determine the other angles. &nbsp;--[[User talk:Lambiam#top|Lambiam]] 04:45, 2 June 2024 (UTC)<br>P.S. Another way to frame the problem could have been in terms of a [[:File:Drawbridge.gif|drawbridge]]. &nbsp;--[[User talk:Lambiam#top|Lambiam]] 04:53, 2 June 2024 (UTC)
:<small> The answer is 42 (glorps). I'll leave it to you to figure out the conversion factor to mks. [[User:Clarityfiend|Clarityfiend]] ([[User talk:Clarityfiend|talk]]) 12:06, 3 June 2024 (UTC) </small>
<small><PRE> |
|'.
| '.
| '.
| '.
| '.
| ::
| ::
|< 5deg >::
| ::
| ::
| ::.
| :: .
| :: .
| :: .
| :: .
|:: V
|* 8.9kg


Equate turning moments at *
:There are a couple of misconceptions in the original question. First, [[memory]] is not encoded in the DNA, it is encoded in the connections ([[synapses]]) between [[neuron]]s within various parts of the [[brain]]. The field of [[neurophysiology]] is all about the chemical and electrical signals that mediate memory formation. There might be a role for [[epigenetics|epigenetic mechanisms]] in memory formation, but this is still an active area of research and not definitive. Formation of the brain itself is an extraordinarily complicated process that is guided by thousands of genes scattered across all chromosomes, acting in different cell types at different times during development. The field of [[developmental neuroscience]] aims to understand this process from anatomic, cellular, and molecular points of view. Whether a person is skilled at math or art (or both!) is a result of [[multifactorial inheritance]] and cannot be pinned to a single gene, and is equally dependent on complex interactions between their genetic makeup and their environment. See [[nature versus nurture]] for a general discussion, although I think this is a false dichotomy -- everything is about genes AND environment. The methods used for identifying genes associated with human characteristics (eye color, disease risk, complex traits) are quite diverse and depend on the specific question being asked. The OP should start with [[human genetics]] as a basic introduction to the topic. --- [[User:Medical geneticist|Medical geneticist]] ([[User talk:Medical geneticist|talk]]) 14:53, 30 June 2010 (UTC)
clockwise = anticlockwise
8.9 kg x sin(5 deg) x 0.74m / 2 = t/2 x 0.74m
where t = tension in each of 2 cables
= 8.9 sin(5 deg)
= 0.775686... kg</PRE> [[User:Philvoids|Philvoids]] ([[User talk:Philvoids|talk]]) 13:16, 3 June 2024 (UTC)</small>


:Shouldn't there be a sine or cosine factor in the anticlockwise term? Imagine the cables being attached to the wall very close to the <tt>*</tt>. Consider dU/dL, where U is potential energy of the panel and L is the length of the cables. &nbsp;--[[User talk:Lambiam#top|Lambiam]] 15:27, 3 June 2024 (UTC)
::Although not accurately shown in the ASCII sketch my calculation presumes that the two cables are at right angles to the sign, are parallel and are fastened to the wall separately at two points. This minimises the cable lengths and tensions. [[User:Philvoids|Philvoids]] ([[User talk:Philvoids|talk]]) 10:28, 4 June 2024 (UTC)
:::The homework problem did not specify where the cables are attached or that the tension should be minimized, making it unsolvable for the average student. For an old-fashioned drawbridge over a castle moat, as in the animation, a better choice is to place the attachment point of the cables at a distance from the hinge equal to the length of the bridge, making the triangle isosceles. Otherwise, the tension grows without bounds as the bridge near a vertical position. &nbsp;--[[User talk:Lambiam#top|Lambiam]] 15:37, 4 June 2024 (UTC)
::::I could not find an ASCII symbol for a hinge but installing one at the base of the sign is a well thought mechanical improvement that can stop the thing flapping about in a wind. This reference desk cannot grant permission to construct a drawbridge with walkway at this location and the shop owner of the wall would likely protest at your plan to cut a hole for passage. Unless he is a herder of sheep or other small animals and himself less than 74cm tall and sees some advantage. The OP doesn't ask for the cable tension when the sign/bridge is vertical but we can say there are two cases: ''Case #1:'' The hinge is broken or absent. t = 8.9/2 = 4.45 kg plus distributed weight of the cable itself; ''Case #2:'' The hinge takes the weight, t = 8.9 sin (0) so both cable lengths and tensions are zero. In that case it would be simpler just to nail the sign to the wall. Or put out a call to [[Banksy]] who can save us the cost of the sign, usually upset someone and greatly increase the value of the wall. [[User:Philvoids|Philvoids]] ([[User talk:Philvoids|talk]]) 18:39, 4 June 2024 (UTC)
:::::For all we know, there is already an opening behind the sign for letting the proprietor's homing pigeons in. If they are equipped with an RFID chip, the sign can be let down automatically on their arrival. &nbsp;--[[User talk:Lambiam#top|Lambiam]] 05:39, 5 June 2024 (UTC)
::::::<small>Pigeon RFIDs can be compromised by malicious [[Yinpterochiroptera]] and [[Yangochiroptera]]. Members of this notorious "YinYangBat Gang" equipped with ultrasound-to-UHF converters can mount [[Spoofing attack]]s on the shared ID frequency. An ad hoc [[Aposematism|aposematic]] remedy such as a wall painting of a cat with the sign text "I EAT BATS" does not alone give security. To this end all pigeons must be urged to register a pass-coo that is less obvious than the too common "Coo Coo". [[User:Philvoids|Philvoids]] ([[User talk:Philvoids|talk]]) 10:28, 5 June 2024 (UTC)</small>


= June 2 =
Putting aside environment for a moment, what are some of the methods used now or in the past that scientists and researchers use in order to identify genes associated with human characteristics? Does one of those methods include comparing the genes of people with a similiar characteristic or trait? And also, when they identify those genes are they able to trace them back to where and which chromosome they came from? I ask because, I once read that the location on the chromosome and which specific chromosome determines what that DNA will turn into or become, though I am not sure how true that is? <span style="font-size: smaller;" class="autosigned">—Preceding [[Wikipedia:Signatures|unsigned]] comment added by [[Special:Contributions/71.143.241.161|71.143.241.161]] ([[User talk:71.143.241.161|talk]]) 17:29, 30 June 2010 (UTC)</span><!-- Template:UnsignedIP --> <!--Autosigned by SineBot-->
::<small>I've tried to answer part of this question below, but I'm having trouble understanding your last question. Can you clarify what you mean by "the location on the chromosome and which specific chromosome determines what the DNA will turn into or become" ? --- [[User:Medical geneticist|Medical geneticist]] ([[User talk:Medical geneticist|talk]]) 19:18, 30 June 2010 (UTC) </small>


== Why females produce androgens ==
:Concerning the original question: ''All'' of the chromosomes are involved in generating the structure of the brain. There are 23 pairs of chromosomes, each containing several thousand protein-coding genes. The great majority of genes are expressed in the brain, and a large fraction are expressed only in the brain. In quantitative terms, most of the information in our genome is directed toward shaping our brains. (This is not to deny the contribution of experience and environment -- I am only asserting that the contribution of genes is very large, not that other contributions are small.) [[User:Looie496|Looie496]] ([[User talk:Looie496|talk]]) 17:38, 30 June 2010 (UTC)


If human embryons of both sexes start off from a female blueprint and given that females lack the male Y chromosome, how it came that women also produce [[androgens]] (even if in small quantity), with related limb and facial hair? [[Special:Contributions/212.180.235.46|212.180.235.46]] ([[User talk:212.180.235.46|talk]]) 19:59, 2 June 2024 (UTC)
:As I said above, there are lots of methods that can be used to determine the function of genes. A classic method is called [[linkage analysis]] where one basically tries to identify markers that segregate with a particular trait following a particular [[Human_genetics#Genetic_differences_and_inheritance_patterns|inheritance pattern]] in a family. For the study of complex traits, the [[genome-wide association study]] or GWAS (unfortunately, that article could be much better) has become the dominant method of identifying genes associated with human characteristics. There have been hundreds or thousands of GWAS studies performed in the last few years, all building on the completion of the human genome and [[Hap map]] projects. One way to perform a GWAS is to compare people with a certain characteristic (disease, trait, etc) to people without that characteristic by looking at the presence of [[single nucleotide polymorphism]] (SNP) markers, which are genetic differences that are present in the general population at a certain frequency. Using fancy statistics you can determine which markers are more often present in people with the trait versus those without, thus generating an "association" between a marker and the trait of interest. By definition, we know exactly where these markers are located along each chromosome (otherwise they wouldn't be that useful) and what genes are nearby. However, association does not equate to causality and there are lots of challenges to interpreting the biological significance of any given association. --- [[User:Medical geneticist|Medical geneticist]] ([[User talk:Medical geneticist|talk]]) 19:15, 30 June 2010 (UTC)
:The article you linked says that the ovaries also produce androgens. ←[[User:Baseball Bugs|Baseball Bugs]] <sup>''[[User talk:Baseball Bugs|What's up, Doc?]]''</sup> [[Special:Contributions/Baseball_Bugs|carrots]]→ 22:33, 2 June 2024 (UTC)
:Biological systems weren't built by any sort of logical designer. They in no way resemble a computer program, a computer, or, for that matter, anything else in the universe. In the case of androgens, the article mentions that androgens are the precursors to estrogens. Males need estrogens too, btw. All these are [[steroids]], which are fundamental to life and are derived from [[cholesterol]]. <span style="font-family: Cambria;"> [[User:Abductive|<span style="color: teal;">'''Abductive'''</span>]] ([[User talk:Abductive|reasoning]])</span> 23:18, 2 June 2024 (UTC)
::"Biological systems weren't built by any sort of logical designer." Which is why so-called "[[intelligent design]]" is just nonsense. The "design" is emphatically not "intelligent". --[[User:Khajidha]] ([[User talk:Khajidha|talk]]) ([[Special:Contributions/Khajidha|contributions]]) 12:07, 3 June 2024 (UTC)
:::I'm not sure biology and computing are as far apart as they used to be now that systems essentially create the gigantic opaque function that transforms input to output themselves in response to their environment/what they have seen and the objectives. Generative adversarial networks for example seem a bit closer to biology than systems used to be. [[User:Sean.hoyland|Sean.hoyland]] ([[User talk:Sean.hoyland|talk]]) 13:06, 3 June 2024 (UTC)
::::Not in a billion years. <span style="font-family: Cambria;"> [[User:Abductive|<span style="color: teal;">'''Abductive'''</span>]] ([[User talk:Abductive|reasoning]])</span> 17:51, 3 June 2024 (UTC)
:::::The technology for [[synthetic biology]], still in its infancy, is advancing with large strides. Whether you like it or not, sooner rather than later it will become possible to design and create complete viable and functioning biological organisms. &nbsp;--[[User talk:Lambiam#top|Lambiam]] 06:40, 4 June 2024 (UTC)
::::::Further, the idea that it is so complicated that nobody understands what it does because it thinks for itself is a farce. No matter what is being used for the computing hardware, be it electronic or biological, the mechanism of operation is very well understood by the engineers who developed it. It just sounds cool to say that it is beyond comprehension. It doesn't sound cool to say that the engineers understand it very well and could trace input through to the output if they wanted to, but simply don't care to do because they have other things to work on. [[Special:Contributions/12.116.29.106|12.116.29.106]] ([[User talk:12.116.29.106|talk]]) 14:42, 4 June 2024 (UTC)
:::::::Tracing the input through to the output is IMO not a helpful concept. Not only do we not know why the most advanced chess or go playing programs make certain surprising moves, but it is not even clear what it means to "understand" why they did this. The computing platform performs a calculation with a certain outcome. The engineers can perform the same calculation by hand, or using abaci, and if they make no mistake they may arrive at the same result in a few billion years: 42. But can they say more than that the answer is 42 because this is the consequence of the rules applied to the input? They knew that already. If someone wants to know ''why'' it is the consequence, they can tell them to repeat the calculation. &nbsp;--[[User talk:Lambiam#top|Lambiam]] 15:51, 4 June 2024 (UTC)
:::::::IP, LLMs for example involve activations in very high dimensional spaces. Trying to map those activation patterns to things we can understand, like concepts etc., is the whole field of interpretability, and it is in its infancy (and safety critical). Engineers are still far from understanding why input A to a model is transformed into output B. If this is something that interests you, have a look at the work being done in Anthropic's lab. [[User:Sean.hoyland|Sean.hoyland]] ([[User talk:Sean.hoyland|talk]]) 17:16, 4 June 2024 (UTC)
::::::::Overgrown Markov Chain models are pretty much useless in any technical field. BTDT [[User:Greglocock|Greglocock]] ([[User talk:Greglocock|talk]]) 04:48, 5 June 2024 (UTC)
:::::::::I guess that's one of the reasons why "Attention Is All You Need" turned out to be such a great title for a paper. [[User:Sean.hoyland|Sean.hoyland]] ([[User talk:Sean.hoyland|talk]]) 08:39, 5 June 2024 (UTC)


= June 5 =
::Even if [[talent]]s have a genetic basis, I don't think it's possible to determine that any single gene is responsible for what the individual is good at. An example of environmental factors taking precedence over genetic predispositions is music making a child more intelligent, as it has been shown that playing a musical instrument increases can make a person smarter in subjects such as mathematics, while limiting a child's exposure to music has been demostrated to have the opposite effect. ~<font color="blue">[[User:AstroHurricane001/A|A]][[User:AstroHurricane001|H]][[User:AstroHurricane001/D|1]]</font><sup>([[User:AstroHurricane001/T|T]][[Special:Contributions/AstroHurricane001|C]][[User:AstroHurricane001/U|U]])</sup> 21:36, 2 July 2010 (UTC)


==Prostheses==
== Telescope Eyepiece ==


what are the matrials uesed in implantable artificial prostheses, such as artificial hearts and small-diameter blood vessels; in the engineering of living tissues ? -- 196.153.184.240
In old German astronomy texts the ocular is described as : mit 20 Linien Oeffnung ( 20 lines opening) . Is there a possibility to translate this into a unit used today? Thanks.--[[User:Stone|Stone]] ([[User talk:Stone|talk]]) 07:28, 30 June 2010 (UTC)
:The length unit [[Line (length)|line]] (German: [[w:de:Linie (Einheit)|Linie]]) seems to be referenced here. Best see both of these articles. HTH. --[[User:Ouro|Ouro]] <small>([[User_talk:Ouro|blah blah]])</small> 08:47, 30 June 2010 (UTC)
::The line was a unit of length of about 1.9–2.2&nbsp;mm throughout most of Europe. I've not been able to find a specific reference to the German Linie, but it's a fair assumption that it was about the same length. That makes your occular 38–44&nbsp;mm: does that sound reasonable? [[User:Physchim62|Physchim62]] [[User talk:Physchim62|(talk)]] 09:14, 30 June 2010 (UTC)
:::Perfect thanks! There was no German line. The French line was in use for most of the technical stuff like telescopes. The best point to look for is the [[w:de:Pariser_Linie]] given with 2,256 mm. [[Paris inch]] is another place to look for. Why do we have no article on the Paris line? --[[User:Stone|Stone]] ([[User talk:Stone|talk]]) 09:22, 30 June 2010 (UTC)
::::[[Ligne]]. Although I think I might merge it into [[Line (length)]] along with the other translations. [[User:Physchim62|Physchim62]] [[User talk:Physchim62|(talk)]] 09:48, 30 June 2010 (UTC)
:::::Be bold! --[[User:Ouro|Ouro]] <small>([[User_talk:Ouro|blah blah]])</small> 10:56, 30 June 2010 (UTC)
:::::I understand from the german text, that these two units are similar, but not the same. --[[User:Eingangskontrolle|Eingangskontrolle]] ([[User talk:Eingangskontrolle|talk]]) 19:11, 1 July 2010 (UTC)


: Heart valves usually made from [[pyrolytic carbon]]. https://www.ncbi.nlm.nih.gov/pmc/articles/PMC10034107/#:~:text=Mechanical%20valves%2C%20usually%20made%20from,stroke%20(3%2C%204). [[Special:Contributions/41.23.55.195|41.23.55.195]] ([[User talk:41.23.55.195|talk]]) 06:13, 5 June 2024 (UTC)
== Make more time ==


:Many different materials are used depending on requirements. For details see [[prosthesis]] and the many links therefrom.[[User:Shantavira|Shantavira]]|[[User talk:Shantavira|<sup>feed me</sup>]] 15:30, 5 June 2024 (UTC)
I've been asked at work to make more time for a project, but I'm not sure I have the necessary supplies. What's time made of? [[Special:Contributions/67.188.234.85|67.188.234.85]] ([[User talk:67.188.234.85|talk]]) 08:36, 30 June 2010 (UTC)
:[http://www.businessballs.com/stories.htm#rocks_bucket_funny_story Rocks and buckets.] --[[User:TammyMoet|TammyMoet]] ([[User talk:TammyMoet|talk]]) 08:41, 30 June 2010 (UTC)
:: A song text says: ''time is made from honey slow and sweet''. Worth a try.--[[User:Stone|Stone]] ([[User talk:Stone|talk]]) 08:41, 30 June 2010 (UTC)
::: Yeah, that's what the song ''says''. But only the fools know what it means. --[[User:Trovatore|Trovatore]] ([[User talk:Trovatore|talk]]) 08:54, 30 June 2010 (UTC)
::::<small>Next you'll be sent down to the supply depot for a "long weight". [[User:Csmiller|CS Miller]] ([[User talk:Csmiller|talk]]) 09:47, 30 June 2010 (UTC)</small>
:::::<small>Don't forget to pick up some "elbow grease" while you're there. And don't come back without it! [[User:Physchim62|Physchim62]] [[User talk:Physchim62|(talk)]] 12:56, 30 June 2010 (UTC)</small>
::::::<small>Making more time is easy. What you do is you find some midnight oil and ignite it. I suppose that the additional time is a waste product of this process. [[User:Googlemeister|Googlemeister]] ([[User talk:Googlemeister|talk]]) 12:59, 30 June 2010 (UTC) </small>
:::::::<small>I had some midnight oil, but it solidified when it got cold out. So I burned it at both ends and that worked out just fine. [[User:Matt Deres|Matt Deres]] ([[User talk:Matt Deres|talk]]) 13:40, 30 June 2010 (UTC) </small>
:If you keep it up, the <nowiki><small></nowiki> tag will break sooner or later. --[[User:Chemicalinterest|Chemicalinterest]] ([[User talk:Chemicalinterest|talk]]) 14:46, 30 June 2010 (UTC)
::<small>Did you intend that to be a reply to 67.188? [[Special:Contributions/86.164.57.20|86.164.57.20]] ([[User talk:86.164.57.20|talk]]) 15:51, 30 June 2010 (UTC)</small>
::<small>Or your [[typeface|face]] will get stuck that way.... ''[[User:Paul Stansifer|Paul]] ([[User talk:Paul Stansifer|Stansifer]])'' 17:41, 30 June 2010 (UTC)</small>


= June 6 =
{{hat|Gibberish}}
: yjr nrdy esu yp ,slr ,ptr yo,r gpt s nifhry od yp ,slr yjr pggovr ,ptr ;olr s jp,r/ d[rmf snpiy yjtrr yjpidsmf fp;;std pm pggovr gocyitrd yjsy ,slr oy ,ptr ;olr s jp,r. smf upi eo;; drr yjsy rbrtupmr pm yjr [tpkrvy jsd ,ptr yo,r yp eptl pm oy \'- [[Special:Contributions/85.181.144.74|85.181.144.74]] ([[User talk:85.181.144.74|talk]]) 18:25, 30 June 2010 (UTC) <small>my hands were on the wrong key, can you fix that?</small>
::<small>/ayewkt rgR QIYKS Gcw vwwb niew qiergqgukw ud tiy gS Nsw HIJW/. /KUJWM /U/;N deTWS JBIR, bs nTVW QIEJ IB RGW WZOKbRUIBM UD TIY;EW YAUBF RGW AOxwvE, [[Special:Contributions/86.164.57.20|86.164.57.20]] ([[User talk:86.164.57.20|talk]]) 19:32, 30 June 2010 (UTC)</small>
{{hab}}


== Health risk of [[taurine]] consumption ==
:: the best way to make more time for a budget is to make the office more like a home. spend about three thousand dollars on office fixtures that make it more like a home. and you will see that everyone on the project has more time to work on it ] Translation. --[[User:Chemicalinterest|Chemicalinterest]] ([[User talk:Chemicalinterest|talk]]) 19:32, 30 June 2010 (UTC)


Realistically, how worried should people be about the health risks of consuming taurine as a supplement? I only ask because half the literature says they find it helpful and beneficial at some unknown dosage, while the other half says it is potentially carcinogenic and could contribute to colon cancer. As a layperson, I find this very confusing. Some of the literature says it could be simply a matter of dosage, but nobody seems to know what the safe or harmful limits are. Can anyone offer some risk analysis devoid of emotion? Should we avoid anything with taurine in it, or not worry at all about it? [[User:Viriditas|Viriditas]] ([[User talk:Viriditas|talk]]) 01:27, 6 June 2024 (UTC)
:::In the June 2010 Scientific American article ''"Is Time an Illusion?",'' we learn that, ''"The universe may be timeless, but if you can imagine breaking it into pieces, some of the pieces can serve as clocks for the others."'' Hopefully that will help. [[User:Bus stop|Bus stop]] ([[User talk:Bus stop|talk]]) 19:42, 30 June 2010 (UTC)


:As far as Wikipedia is concerned, [[WP:MEDRS]] applies. That means that the minimum quality level for [[WP:RS]] making medical claims are systematic reviews indexed for MEDLINE (there are some exceptions from this indexation, but generally speaking MEDLINE is the gold standard). [[User:tgeorgescu|tgeorgescu]] ([[User talk:tgeorgescu|talk]]) 03:29, 6 June 2024 (UTC)
:Simple: you spend less time doing other things!--[[User:RampantHomo|RampantHomo]] ([[User talk:RampantHomo|talk]]) 01:17, 1 July 2010 (UTC)
:Our article states that there is no good clinical evidence that taurine supplements provide any benefit to human health, Why pay for useless supplements? The human body naturally produces <u>a large amount</u> of taurine, far more than one can reasonably take in as a supplement. There is increasing evidence that taurine actually plays a role in preventing cancer.<sup>[https://link.springer.com/chapter/10.1007/978-3-030-93337-1_11]</sup> Any carcinogenicity of supplements can only be due to their being fake, or a lack of quality control in their production. &nbsp;--[[User talk:Lambiam#top|Lambiam]] 07:23, 6 June 2024 (UTC)
::The underlying issue is that large doses of taurine are added to energy drinks. Nobody seems to know why. A current study is looking at an association between energy drink consumption and the rise in colon cancer in young adults.[https://www.cancer.gov/research/participate/clinical-trials-search/v?id=NCI-2024-02154&r=1] [[User:Viriditas|Viriditas]] ([[User talk:Viriditas|talk]]) 08:23, 6 June 2024 (UTC)
:::Even disregarding any potential carcinogenic risks, there are enough studies that show damaging health effects of high consumption levels of energy drinks.<sup>[https://www.ncbi.nlm.nih.gov/pmc/articles/PMC4682602/][https://www.kxan.com/news/energy-drinks-show-damaging-effects-on-young-adults-multiple-studies-show/]</sup> &nbsp;--[[User talk:Lambiam#top|Lambiam]] 18:28, 6 June 2024 (UTC)
::::¼ liter of my favorite energy drink has 80 mg caffeine, while adults usually consume up to 400 mg caffeine per day. And I use the energy drink totally without sugar. [[User:tgeorgescu|tgeorgescu]] ([[User talk:tgeorgescu|talk]]) 18:59, 6 June 2024 (UTC)
:::::The energy drink thing may be related; most energy drinks contain [[vitamin B12]]. There is a concern that (mega)dosing B12 in excess of daily requirements carries with it a slight increase in risk of cancer due to B12 containing cobalt, which is both a heavy metal and has a trace of radioactive [[cobalt-60]]. <span style="font-family: Cambria;"> [[User:Abductive|<span style="color: teal;">'''Abductive'''</span>]] ([[User talk:Abductive|reasoning]])</span> 20:25, 6 June 2024 (UTC)
::::::Thank you, that's helpful. I just read the comments by the epidemiologist on that topic, and while my reading might be flawed, they seemed to indicate that dosage and tobacco smoking played a significant role in the risk. [[User:Viriditas|Viriditas]] ([[User talk:Viriditas|talk]]) 22:02, 6 June 2024 (UTC)
:::::::Real skepticism cuts both ways. Taurine is in human breast milk, evidence of benefit of at least one energy drink exceeding risk. :-) But our article has imho excessive, even [[Georg Kreisel|dubious doubt]] in that section on its conditional essentiality or benefit for infants, and thus its common use in [[Infant formula|another energy drink]]. Doubt that appears to stem from OR or opinion rather than the source, which says e.g. "Thus the new data provide further support for the view that taurine is a conditionally essential nutrient for the preterm infant" & that ethical considerations seem to prevent further research.[[User:John Z|John Z]] ([[User talk:John Z|talk]]) 03:57, 7 June 2024 (UTC)


I've also heard that you can spend all your time making money, or you can spend all your money making time. --[[User:Trovatore|Trovatore]] ([[User talk:Trovatore|talk]]) 01:24, 1 July 2010 (UTC)


= June 8 =
Sheesh. [[Time (magazine)|Time]] is made of paper, ink, and staples. What else? --Anonymous, 03:36 UTC, July 1, 2010.

:You can extend the time you have using [[time dilation]]. Either move slower than everyone else, move further from a gravity well, or some combination thereof. Also, make sure not to spend more than a few milliseconds doing this, or you'll waste more time than you save. — [[User:DanielLC|DanielLC]] 07:56, 2 July 2010 (UTC)

Time may be partially or completely made of [[Paul Simon]]. He wrote "Time, time, time, see what's become of me." If there's not enough Paul left (he was getting kind of small the last time I looked), he may be able to suggest other aging musicians who could help. (I don't think Crosby has much to do these days.) [[User:Twang|Twang]] ([[User talk:Twang|talk]]) 18:23, 2 July 2010 (UTC)

:You don't actually have to ''make'' more [[time]] (ie. in a schedule, or physically creating time which is impossible) to make it feel like you have more time. See [[time perception]] and [[perception of time]]. I'm sure there are techniques to make it feel like you had more time than you did considering all the stuff you've gotten done and I've done that, however I don't have the capacity to elucidate them, but compare [[apathy]] and [[flow (psychology)|flow]]. ~<font color="blue">[[User:AstroHurricane001/A|A]][[User:AstroHurricane001|H]][[User:AstroHurricane001/D|1]]</font><sup>([[User:AstroHurricane001/T|T]][[Special:Contributions/AstroHurricane001|C]][[User:AstroHurricane001/U|U]])</sup> 18:35, 2 July 2010 (UTC)

== Fabry-Perot filter ==

What's a MEMS-based tunable Fabry-Perot filter? There isn't an article on it. --[[Special:Contributions/76.77.139.243|76.77.139.243]] ([[User talk:76.77.139.243|talk]]) 13:15, 30 June 2010 (UTC)
:There is, it's just called a [[Fabry–Pérot interferometer]]! [[User:Physchim62|Physchim62]] [[User talk:Physchim62|(talk)]] 13:21, 30 June 2010 (UTC)
::And you may want to read up on [[MEMS]] as well. "Tunable" suggests that the wavelength that the filter passes can be adjusted, probably because the MEMS is capable of adjusting the spacing of the components on demand. -- [[User:Coneslayer|Coneslayer]] ([[User talk:Coneslayer|talk]]) 13:25, 30 June 2010 (UTC)

== Why do cars go slower in reverse than they do going forward? ==

^Topic says it all. [[Special:Contributions/148.168.127.10|148.168.127.10]] ([[User talk:148.168.127.10|talk]]) 14:13, 30 June 2010 (UTC)
:In general, because it is anticipated that you'll only travel for very short distances in a backwards direction; because visibility is restricted, and because steering works differently, both of which make longer distances a more unlikely proposition. And then because having only a single reverse gear calls for less machinery in the gearbox department, equalling lower cost. --[[User:Tagishsimon|Tagishsimon]] [[User_talk:Tagishsimon|(talk)]] 14:17, 30 June 2010 (UTC)

:Taking a slightly different tack: as Tagishsimon notes, cars have only a [[Transmission_(mechanics)#Automotive_basics|single reverse gear]]. It's comparable, roughly, to first gear going forward, so a car goes about as fast in reverse as it does going forward in first gear. The car doesn't go faster in reverse because there are no additional gears as there are for driving forwards. Now apply the rest of Tagishsimon's answer -- there are no further reverse gears because of cost, weight, lack of need, safety, etc. But the fundamental mechanical limitation is the single gear ratio. &mdash; [[User talk:Lomn|Lomn]] 14:49, 30 June 2010 (UTC)

::Actually most cars can go pretty fast in reverse gear, much faster than is safe anyway. I've always thought they should have a speed limiter in reverse gear. Maybe they do these days, it's a while since I tried accelerating hard in reverse.--[[User:Shantavira|Shantavira]]|[[User talk:Shantavira|<sup>feed me</sup>]] 15:04, 30 June 2010 (UTC)
:::Yes, you can go quite fast, in the same way that you can go quite fast forwards in 1st gear, but the car doesn't like it! (And neither does your fuel consumption.) --[[User:Tango|Tango]] ([[User talk:Tango|talk]]) 16:33, 30 June 2010 (UTC)

:Some cars do go as fast in reverse as forwards...the [[Bond Minicar]], for example doesn't have a proper reverse gear - so to drive backwards, you stop the engine - flip a switch that makes the starter motor run backwards and start the engine running in the opposite direction! There is one such family of cars, the [[DAF Daffodil]] and it's successors that used an early continuously variable transmission called "[[Variomatic]]" - fans of the Daffodil who restore these cars have races where everyone drives in reverse! The Dutch have a regular series of races where everyone drives in reverse - and the Daffodils had to go into a class of their own because they can go 100+ mph backwards! [http://www.youtube.com/watch?v=S7ipFApsFec Here is a YouTube video of one race]...but I don't think it's the Daffodil class because they typically race at '-60 mph'!

:Also, there are indeed cars with speed limiters in reverse. The older BMW [[MINI ONE]]'s (which also had continuously variable transmissions) have to be speed-limited in reverse in order to avoid shredding their wierd rubber drive belts. It's in the nature of any normally-gearless transmission system that in order to go backwards, you have to have a way to use auxilliary gears to get into reverse (often built into the differential casing) - and that naturally results in a car that goes as fast in reverse as forwards. Since that's obviously dangerous, using a speed limiter is a smart idea.

: My (stick-shift) MINI Cooper'S can do close to 45mph in 1st gear - and the 1st and reverse gear ratios are almost identical - so it clearly '''could''' go 45mph in reverse. I don't intend to find out though!

: [[User:SteveBaker|SteveBaker]] ([[User talk:SteveBaker|talk]]) 19:42, 30 June 2010 (UTC)
::Steering while driving in reverse at high speed is not nearly as stable as when going forward at the same speed so experimentation as to how fast your car can go in reverse is probably not a fantastic idea. [[User:Googlemeister|Googlemeister]] ([[User talk:Googlemeister|talk]]) 20:09, 30 June 2010 (UTC)
:::See [[Steering#Rear_wheel_steering]]. '''<font face="Arial">[[User:Acroterion|<font color="black">Acroterion</font>]] <sub><small>[[User talk:Acroterion|<font color="gray">(talk)</font>]]</small></sub></font>''' 20:30, 30 June 2010 (UTC)

I recall seeing an English film in which a couple were out driving in the early days of the automobile and their car did not have sufficient power to make it up a steep hill -- it kept stalling, even in 1st. They switched places and the woman backed it up the hill successfully because the reverse gear was geared lower than 1st gear. Does anyone here know (1) what film that was from and (2) how common it is (or was) to have the reverse gear geared considerable lower than 1st? -- [[Special:Contributions/60.49.38.251|60.49.38.251]] ([[User talk:60.49.38.251|talk]]) 11:05, 1 July 2010 (UTC)

:In winter it's common to drive a [[Front-wheel drive]] car in reverse up a slippery slope to maximise the traction of the engine weight over the front wheels. [[User:Cuddlyable3|Cuddlyable3]] ([[User talk:Cuddlyable3|talk]]) 11:24, 1 July 2010 (UTC)
:No idea about the film, but lots of old (and newer) cars have reverse gears at higher ratios compared to the rest, resulting in more torque and less top speed for the gear. It only stands to reason, for all the explanations mentioned, that you would want the reverse gear to allow only slow, careful motion. Our own article on [[gear ratio]]s proclaims this, at least for the 2004 Chevy Corvette. --[[Special:Contributions/144.191.148.3|144.191.148.3]] ([[User talk:144.191.148.3|talk]]) 19:07, 1 July 2010 (UTC)

== Acidity of warm water ==
since
# Is warm water more or less acidic than cold water?
# What happens when a carbonated beverage becomes warm?

--[[User:478jjjz|478jjjz]] ([[User talk:478jjjz|talk]]) 16:40, 30 June 2010 (UTC)

:Pure water is neutral at any temperature, as far as I know (I can't think of any reason for it not to be). A carbonated beverage contains [[carbonic acid]], so is slightly acidic. I don't know what happens to it when it changes temperature, though. --[[User:Tango|Tango]] ([[User talk:Tango|talk]]) 16:44, 30 June 2010 (UTC)

::In practicality, water will have the same pH, whatever the temperature is. If there is a lot of carbon dioxide (such as you blowing breath through the water), then it will be more acidic when cold since more carbon dioxide dissolves in cold water.

::Gases are less soluble in high temperatures than in low temperatures. So the equilibrium below would go toward the gas side. --[[User:Chemicalinterest|Chemicalinterest]] ([[User talk:Chemicalinterest|talk]]) 18:06, 30 June 2010 (UTC)

::: A solution is neutral when pH = pOH. This may or may not be 7, depending on the temperature. [[User:John Riemann Soong|John Riemann Soong]] ([[User talk:John Riemann Soong|talk]]) 19:03, 30 June 2010 (UTC)
::: I believe the pH of completely neutral boiling liquid water (no carbon dioxide) is 6.15. Now if you turn the pressure up and it boils at a higher temperature, you can increase the self-dissociation constant considerably. [[User:John Riemann Soong|John Riemann Soong]] ([[User talk:John Riemann Soong|talk]]) 19:14, 30 June 2010 (UTC)

CO<sub>2</sub> (g) + H<sub>2</sub>O (l) {{eqm}} H<sub>2</sub>CO<sub>3</sub> (aq)

What is the role of temperature in the above reaction?--[[User:478jjjz|478jjjz]] ([[User talk:478jjjz|talk]]) 16:49, 30 June 2010 (UTC)

:Primarily because of [[entropy]], when you warm a solution, otherwise solid [[solute]]s become more soluble, whereas gaseous solutes become less soluble. Warm water can hold less dissolved gasses (such as CO<sub>2</sub>) than cold water. When you boil water, this is the origin of the initial bubbles you see when the water is still below the boiling point. It's not the water turning into gas, it's the dissolved gasses coming out of solution as the water warms. -- [[Special:Contributions/140.142.20.229|140.142.20.229]] ([[User talk:140.142.20.229|talk]]) 17:16, 30 June 2010 (UTC)

::Several things I should add: While absolutely pure water contains equal concentrations of [[hydronium]] and [[hydroxide]] ions, the absolute concentrations of them varies slightly with temperature (see [[autoionization constant for water]]). Also water, even pure water, rapidly picks up a small amount of carbon dioxide from the air, pushing the pH into the acid range. Finally, in addition to the temperature dependence on solubility, there is a temperature dependence on [[acid dissociation constant]]s. I'm not sure what it is for the carbonic acid <==> bicarbonate transition, but for the bicarbonate <==> carbonate transition, it's enough to cause a 0.1 pH units or so difference per 10 °C change near room temperature. -- [[Special:Contributions/140.142.20.229|140.142.20.229]] ([[User talk:140.142.20.229|talk]]) 17:30, 30 June 2010 (UTC)

:This one smacks of homework. We won't do homework, as a rule of thumb, unless you're stuck. The second one is an unusual question though, so I'll bite: gases are less soluble at higher temperatures, so the drink will become fizzier as you warm it. [http://202.129.0.134/courses/716/174temppres.html This Elmhurst College] page has a bit more on it. [[User:Brammers|Brammers]] ([[User_talk:Brammers|talk]]/[[Special:Contributions/Brammers|c]]) 17:42, 30 June 2010 (UTC)

::Gases are less soluble in hot liquids, so the equilibrium would go to the gas side. In cold temperatures, it will go away from the gas side. See [[Le Chatelier's principle]] for an explanation of equilibrium. --[[User:Chemicalinterest|Chemicalinterest]] ([[User talk:Chemicalinterest|talk]]) 18:06, 30 June 2010 (UTC)
:::The above is true, and is true for '''''all''''' gases. It's an [[entropy]] effect. [[User:Physchim62|Physchim62]] [[User talk:Physchim62|(talk)]] 12:40, 1 July 2010 (UTC)
The pH of water varies with temperature, and this can occasionally have important chemical effects (I personally once spent three months studying one of them, but it never got to the point where it could be published). We say that water is neutral if it has equal concentrations of H<sup>+</sup> and OH<sup>−</sup>, but these are different chemical species with different chemistry. If you heat pure water, you will increase the concentration of H<sup>+</sup> (and of OH<sup>−</sup> at the same time), so you will increase the rate of any reaction which depends on H<sup>+</sup> for its [[rate-determining step]]. [[User:Physchim62|Physchim62]] [[User talk:Physchim62|(talk)]] 12:40, 1 July 2010 (UTC)
::::For a related topic on a larger scale, see [[ocean acidification]]. The oceans ''release'' more carbon dioxide from carbonic acid and are able to absorb ''less'' carbon dioxide from the atmosphere at a warmer temperature, but high atmospheric carbon dioxide concentrations that would likely be responsible for the warming result in ''more'' carbon dioxide going to the oceans and therefore ''more'' carbonic acid production and ''increased'' acidification. ~<font color="blue">[[User:AstroHurricane001/A|A]][[User:AstroHurricane001|H]][[User:AstroHurricane001/D|1]]</font><sup>([[User:AstroHurricane001/T|T]][[Special:Contributions/AstroHurricane001|C]][[User:AstroHurricane001/U|U]])</sup> 18:28, 2 July 2010 (UTC)

== I tried to sanitise my potato peeler by sonicating it in acetone ==

When I dipped it in acetone it seemed fine...then I turned the sonicator on and the solution turned all black! When I removed the peeler from solution half of the lacquer/varnish had been dissolved.

question: is the peeler still safe to use for food? (I brought it from home to peel plant/vegetable tissue for the lab, but still...)

Also, why would acetone do this, as opposed to methanol, ethanol or isopropanol? Or heavy-duty glassware detergent?(Nothing happened with those agents when I used them as sonicating solvents.) [[User:John Riemann Soong|John Riemann Soong]] ([[User talk:John Riemann Soong|talk]]) 19:21, 30 June 2010 (UTC)

:Acetone is a more powerful solvent. I don't think that it is unsafe. Wash away the dissolved residue and let the acetone evaporate. --[[User:Chemicalinterest|Chemicalinterest]] ([[User talk:Chemicalinterest|talk]]) 19:23, 30 June 2010 (UTC)
:Because acetone is a strong solvent for paints and plastics and is used to strip some kinds of paints and lacquers? It's commonly used as nail polish remover and to remove uncured urethane, and to my knowledge, it's not a particularly good disinfectant (or at least not better than), compared to, say, ethanol or isoproanol, and more toxic to people than either, at least until it's all evaporated. '''<font face="Arial">[[User:Acroterion|<font color="black">Acroterion</font>]] <sub><small>[[User talk:Acroterion|<font color="gray">(talk)</font>]]</small></sub></font>''' 19:27, 30 June 2010 (UTC)
::Actually acetone is not very toxic at all. It's a great solvent, but not a particularly good disinfectant (possibly because it's not very toxic). [[User:Ariel.|Ariel.]] ([[User talk:Ariel.|talk]]) 05:41, 1 July 2010 (UTC)
:::Yes, for short-term exposure, even to high concentrations, it's not too bad (and I rather like the odor), but occupational exposure to acetone-based paints and finishes can do harm. Urethane floor finishes once exclusively employed an acetone solvent/carrier, and I was acquainted with a number of floor finishers who were pretty well addled. '''<font face="Arial">[[User:Acroterion|<font color="black">Acroterion</font>]] <sub><small>[[User talk:Acroterion|<font color="gray">(talk)</font>]]</small></sub></font>''' 12:22, 1 July 2010 (UTC)

:Acetone does it because it lacks OH groups, which means that dissolution of non-OH containing solids such as lacquer is not unfavourable as it is in MeOH , EtOH , water/detergent etc.. but it is polar, which usually helps.
:Other non hydrogen bonding solvents will do the same eg DCM, CCl4, MeOAc [[Special:Contributions/83.100.252.42|83.100.252.42]] ([[User talk:83.100.252.42|talk]]) 21:37, 30 June 2010 (UTC)
:: Is carbon tetrachloride isn't polar though....isn't it exceptionally oily? How it dissolve lacquer when hexanes wouldn't?
:: Also lacquer I think is composed of solvent-like polymers with lots of esters or carbonyl functionalities...do esters and nonprotic carbonyls generally prefer to bond to carbonyl carbons electrostatically rather than through hydrogen bonding? I suppose it's because carbonyl oxygens are weaker hydrogen bond acceptors? [[User:John Riemann Soong|John Riemann Soong]] ([[User talk:John Riemann Soong|talk]]) 22:02, 30 June 2010 (UTC)
:::1. CCl4 is much more polarisable. (and a fundamentally bigger molecule that hexane)
:::2. a. Yes your description of lacquers sounds right.
:::2. b. No. The idea related to the hydrogen bonds is that MeOH etc can form 2 hydrogen bonds with MeOH, but only one with C=O , that's why hydrogen bonding solvents show reduced dissolving power compared to other polar solvents for molecules that are not hydrogen bond donor/acceptors.
:::Non-protic polar molecules tend to dissolve ... generally .. the polar factor is usually small compared to the entropy effect. eg acetone and hexane mix. The energy of dipole dipole interactions is going to be fairly limited compared to the thermal energy in a liquid at r.t - any interactions may be frozen out in the solid though. A hydrogen bond to C=O is stronger than a dipole dipole interaction. [[Special:Contributions/83.100.252.42|83.100.252.42]] ([[User talk:83.100.252.42|talk]]) 22:35, 30 June 2010 (UTC)
:Next time sonicate the peeler in water with detergent! [[User:Graeme Bartlett|Graeme Bartlett]] ([[User talk:Graeme Bartlett|talk]]) 12:10, 1 July 2010 (UTC)
::Or use a brush.. I remember your question about cleaning a knife with a gas flame.. what next?! . "''I used [[laser ablation]] to remove grease spots from a tea towel and I'm worried it's affected the fabric...''" [[User:Sf5xeplus|Sf5xeplus]] ([[User talk:Sf5xeplus|talk]]) 12:52, 1 July 2010 (UTC)
::: Ewwww brushes. I was trying to kill microbes and remove any [[biofilm]]s, not generate them. 15:50, 1 July 2010 (UTC)

== Parasite? ==

[[File:Parasite.JPG|250px|right]]
Does anybody happen to be able to identify this (I suppose) parasite? (Sorry for the out-of-focus image.) Some context: several of this white pods appeared, together with whitish spiderweb-like filaments, on a lemon plant kept in a vase on a balcony in an apartment building in Rome (Italy). The largest pod is about 1 centimeter long. Thanks! [[::User:Goochelaar|<span style='font-family: "Palatino"'>Goochelaar</span>]]&nbsp;([[::User talk:Goochelaar|talk]]) 19:44, 30 June 2010 (UTC)

: It is a kind of a [[scale insect]], probably [[Margarodidae]] family. --[[User:Dr Dima|Dr Dima]] ([[User talk:Dr Dima|talk]]) 20:44, 30 June 2010 (UTC)

::It really looks like the [[cottony cushion scale]].--[[User:Stone|Stone]] ([[User talk:Stone|talk]]) 21:38, 30 June 2010 (UTC)

:::Thanks to both! It looks like it indeed. My lemon is not happy, but at least I have learnt something today. [[::User:Goochelaar|<span style='font-family: "Palatino"'>Goochelaar</span>]]&nbsp;([[::User talk:Goochelaar|talk]]) 21:55, 30 June 2010 (UTC)

::::You are welcome. The easy way to deal with those guys is simply to remove them using a cotton ball soaked in vodka or dilute ethanol. Don't soak the cotton ball too much, and don't let the alcohol drip on the soil or roots. You will need to repeat this several times, every other week or so. There are species of scale insects that would color the cotton ball bright red, this is normal (it's not blood, it's [[carmine]]). --[[User:Dr Dima|Dr Dima]] ([[User talk:Dr Dima|talk]]) 22:51, 30 June 2010 (UTC)
:::::<small>A lemon tree? Carmine? The OP might be able to make a [[Campari]] and Lemonade --[[User:Frumpo|Frumpo]] ([[User talk:Frumpo|talk]]) 08:05, 1 July 2010 (UTC)</small>

== how to remove cuticle from a leaf? ==

Is there any way of removing the cuticle of a leaf while keeping most of the cells underneath it alive? It's okay if some of them are injured. A surfactant + sonication, maybe? [[User:John Riemann Soong|John Riemann Soong]] ([[User talk:John Riemann Soong|talk]]) 19:50, 30 June 2010 (UTC)

: [http://www3.interscience.wiley.com/journal/119543386/abstract this] paper says that the plants they chose allow the leaf epidermis to be easily (?) removed with a dissecting needle and forceps. You will probably need a good [[stereo microscope]] and a steady pair of hands :) --[[User:Dr Dima|Dr Dima]] ([[User talk:Dr Dima|talk]]) 21:01, 30 June 2010 (UTC)

:: Interesting..... uhh I'll try it out tomorrow. Maybe there's a way to "weaken" the cuticle? Btw, if I dip the leaves in alcohol or something it won't kill the cells inside, should it? Maybe a few seconds of sonication then attempt to pry off the epidermis? (Btw, I assume the bottom of the leaf is covered with a water-impermeable substance too ... just less waxy...) [[User:John Riemann Soong|John Riemann Soong]] ([[User talk:John Riemann Soong|talk]]) 21:13, 30 June 2010 (UTC)

== Extinct Volcanoes in North America and Asia. ==

Please give me the names and information about extinct Volcanoes in Canada, Alaska, The United States, Australia, and New Zealand. Also, any other outside Reference sources of Information I can consult. --[[Special:Contributions/24.193.198.15|24.193.198.15]] ([[User talk:24.193.198.15|talk]]) 20:58, 30 June 2010 (UTC)
:You can find the ones we have articles on in [[:Category:Extinct volcanoes]] [[Special:Contributions/83.100.252.42|83.100.252.42]] ([[User talk:83.100.252.42|talk]]) 21:05, 30 June 2010 (UTC)
::You may also be interested in [[Large igneous province]], [[supervolcano]] and [[extinct volcano]]. ~<font color="blue">[[User:AstroHurricane001/A|A]][[User:AstroHurricane001|H]][[User:AstroHurricane001/D|1]]</font><sup>([[User:AstroHurricane001/T|T]][[Special:Contributions/AstroHurricane001|C]][[User:AstroHurricane001/U|U]])</sup> 18:17, 2 July 2010 (UTC)

== which type of plant cells (and species) tends to have the biggest / most permissive [[plasmodesmata]] ==

Are onion root cells a good choice, or are there other types I should look at? What are "root tip cells" btw? Could I get them from store-bought onions? [[User:John Riemann Soong|John Riemann Soong]] ([[User talk:John Riemann Soong|talk]]) 21:11, 30 June 2010 (UTC)

== Do any nonhuman animals possess a conscience? ==

Or is it impossible to determine with certainty, as with love, mentioned above? --[[Special:Contributions/95.148.107.189|95.148.107.189]] ([[User talk:95.148.107.189|talk]]) 21:52, 30 June 2010 (UTC)

:As with love, how do we define it? There are definitely experiments that show that many other animals understand what we might call "fairness"—getting angry when they don't get equal treatment for other animals they consider their equals. See [http://www.npr.org/templates/story/story.php?storyId=97944783 this story from NPR] from a couple years back. Is that a "conscience"? Can we speak with any certainty about human emotions in non-humans? Do we trust [[Koko (gorilla)|Koko]] when she signs "mad" at the three kittens who have lost their mittens? --[[User:Mr.98|Mr.98]] ([[User talk:Mr.98|talk]]) 22:34, 30 June 2010 (UTC)

: '''(ec)''' You must be more specific in what you ask. Conscience means (1) ability to know good and evil, that is, to tell a moral right from moral wrong; and (2) an ability to feel remorse. AFAIK this is not directly related to the ability to feel affection or love. There exist definitions of [[conscience]], [[consciousness]], [[sentience]], [[sapience]], and [[self-awareness]]; knowing them will allow your question to be formulated more precisely (but not necessarily to be answered more fully, as the answers to most questions of that kind are not yet known). Too many a flame war has been fought due to inconsistent terminology on this subject; you would not want to be on the receiving end of most of it. Please restate your question more specifically. --[[User:Dr Dima|Dr Dima]] ([[User talk:Dr Dima|talk]]) 22:37, 30 June 2010 (UTC)

::I'm not sure any of those terms have "precise definitions". In fact their articles are largely about the endless philosophical debates about what they might mean ("sapience" is the exception, because it foists the definition off onto "wisdom" and "sound judgment"). The plain fact of it is that these terms mean a lot to us as emotional/intellectual beings but applying them in a precise, "scientific" manner seems fairly impossible, because nobody can really agree on what is "precisely" meant by them (and any one "precise" definition ends up excluding other meanings that are seen as important).--[[User:Mr.98|Mr.98]] ([[User talk:Mr.98|talk]]) 00:08, 1 July 2010 (UTC)

:::I said "definitions", I never said "precise definitions". By putting "precise definitions" in quotes you incorrectly cite the previous post. --[[User:Dr Dima|Dr Dima]] ([[User talk:Dr Dima|talk]]) 02:13, 1 July 2010 (UTC)

:If you are very interested in animal intelligence, can I point you towards the books of [[Temple Grandin]], especially ''[[Animals in Translation]]''? They are a delight to read and extremely enlightening. If I can hazard a personal opinion, I think she does a good job of asking the "right" kind of questions about animal intelligence, and avoiding the pointless ones (which are really just proxies for "do animals have souls?" and other metaphysical dead-ends). --[[User:Mr.98|Mr.98]] ([[User talk:Mr.98|talk]]) 00:11, 1 July 2010 (UTC)

::If animals have any sense of [[morality]], it must be very different from the human sense, and even that has a lot of intervariability. However, we cannot "test" to see if animals have a self-awareness, as brain scans fail to "detect" any material consciousness and humans assume that we (the [[self]]) have a consciousness but this delves into the realm of [[qualia]]. You might also like to try the thought experiment involving robots, as we cannot currently by definition create a "consciousness" in a robot just as we cannot test for a consciousness in either robots, other humans or animals. ~<font color="blue">[[User:AstroHurricane001/A|A]][[User:AstroHurricane001|H]][[User:AstroHurricane001/D|1]]</font><sup>([[User:AstroHurricane001/T|T]][[Special:Contributions/AstroHurricane001|C]][[User:AstroHurricane001/U|U]])</sup> 18:13, 2 July 2010 (UTC)

== [[Neutron star]] vs. [[quark star]] vs. [[black hole]] ==

1) According to the article [[neutron star]], the star doesn't collapse further because of the [[Pauli exclusion principle]]. But the article also clearly indicates that a [[quark star]] or [[black hole]] will collapse further. Does this mean that the Pauli exclusion principle only applies up to the point at which the [http://www2.kutl.kyushu-u.ac.jp/seminar/MicroWorld3_E/3Part3_E/3P32_E/nuclear_force_E.htm strong repulsive force] that keeps the nucleus from imploding applies?

2) I'm still miffed by the whole black hole paradox. As person A approaches the black hole, his clock will continue to go slower and slower until it appears to the outside world that it stops (i.e., it's [[asymptotic]]). But as person A approaches the black hole, time doesn't stop for person A. What the heck is going on here? [[User:Magog the Ogre|Magog the Ogre]] ([[User talk:Magog the Ogre|talk]]) 22:31, 30 June 2010 (UTC)

:For (1), yes. And for (2), that's just what happens. It's like in special relativity, where a clock moving relative to your frame of reference will go slower than one that is stationary, except that it's caused by the gravitational field rather than the relative motion of reference frames. [[User:ConMan|Confusing Manifestation]]<small>([[User talk:ConMan|Say hi!]])</small> 23:44, 30 June 2010 (UTC)
Are we to assume that guy A just gets torn apart right at the event horizon then? Because right at the edge, time will be going infinitely fast. So we're assuming the black hole falls apart via [[Hawking radiation]] in person B's time, which would be infinitely quick by person A's time? If so, that means that person A would get ripped apart before falling within the event horizon... I think... [[User:Magog the Ogre|Magog the Ogre]] ([[User talk:Magog the Ogre|talk]]) 01:11, 1 July 2010 (UTC)

::No, the person A (Shall we call her Alice?) does not get torn apart at the horizon (assuming that the black hole is big enough that the horizon is not too close to the center of the black hole where alice will be torn apart by the singularity. See [[spaghetification]]). Also even though from the point of view of B (Shall we call him Bob?) Alice does halt to a stop as she approaches the horizon, from the point of view of Alice nothing special happens at the horizon and she does not see Bob spedding up to infinite speed. [[User:Dauto|Dauto]] ([[User talk:Dauto|talk]]) 03:31, 1 July 2010 (UTC)
:::I think you meant [[spaghettification]]. [[User:Csmiller|CS Miller]] ([[User talk:Csmiller|talk]]) 06:41, 1 July 2010 (UTC)
::::Yes, indeed. Thank you. [[User:Dauto|Dauto]] ([[User talk:Dauto|talk]]) 17:07, 1 July 2010 (UTC)
Yes, but this is a contradiction. Bob stops seeing Alice move, her time stops. So, um, when she sees Bob, doesn't she see him quicker and quicker? That's what Stephen Hawking maintains anyway. It violates the principle that one person's reality does not conflict with another's (I can't remember the name). Grr, this is the second or third time I've brought this up on here, is there some internet forum where physicists hang out? [[User:Magog the Ogre|Magog the Ogre]] ([[User talk:Magog the Ogre|talk]]) 00:43, 2 July 2010 (UTC)
:::::You're thinking of [[Leonard Susskind]] and his book "The Black Hole War". But it's not really a paradox (even though it seems like one); as long as the two can never get together and "compare clocks", there's no problem. From Alice's point of view, she passes through the event horizon just fine, but gets spaghettified eventually. From Bob's point of view, Alice sits near the Event horizon forever. But Alice can't come back and ask Bob what happens after what happens after (to him) an "infinite" amount of time because she's stuck in the gravity well of a black hole. Bob sees one thing, and Alice sees another, but as paradoxical as it sounds, it ends up being OK. [[User:Buddy431|Buddy431]] ([[User talk:Buddy431|talk]]) 02:37, 2 July 2010 (UTC)
:Alice and Bob see different things but without contradiction (That's why the theory is called relativity).For instance, from the point of view of Alice there is no Hawking radiation, while from the point of view of Bob there is a surface just above the horizon (called the extended horizon) of incredibaly high temperature from which the Hawking radiation comes from. Which one is right? Both are. To reiterate what I said earlier, Alice '''doesn't''' see Bob speeding up to infinite speed. The idea that Bob and Alice have clocks that tick at different rates and that if one sees the other one faster than the latter must see the former slower is too simplistic and plainly wrong. It is wrong even within special relativity where when two observers pass each other both of them observe the other's clock to tick more slowly than their own. [[User:Dauto|Dauto]] ([[User talk:Dauto|talk]]) 04:34, 2 July 2010 (UTC)
::A related concept is the [[Relativity of simultaneity]], which is a function of these same "paradoxes" (being paradoxes only in the ''classical physics'' or ''intuitive'' sense). It is actually impossible to determine which of two events can be said to occur "absolutely" first. For any observable pair of events which are seperated by any arbritrary space, there will always exist some frame of reference where one occurs first, and a different frame of reference where the other occurs first. This is very similar to the idea that from Alice's frame of reference, she passes through the event horizon at a steady rate; and notices no observable change to herself at the event horizon in terms of her motion, but Bob observes her stop moving entirely. That's the core principle of the term "relativity"; there '''is no universal reference frame''' from which measurements can be made in an absolute manner. All reference frames are relative to each other. --[[User:Jayron32|<font style="color:#000099">Jayron</font>]]'''''[[User talk:Jayron32|<font style="color:#009900">32</font>]]''''' 04:51, 2 July 2010 (UTC)

== Homogeneous Continuous-time Markov chains - analytical expressions for the finite time transition probabilities? ==

This looks like a question much better suited to the [[Wikipedia:Reference desk/Mathematics]]. [[User:Dolphin51|<font color="green">''Dolphin''</font>]] ''([[User talk:Dolphin51|<font color="blue">t</font>]])'' 00:15, 1 July 2010 (UTC)

:I have transferred this question to the Mathematics Reference Desk. See [[WP:RD/MA#Homogeneous Continuous-time Markov chains - analytical expressions for the finite time transition probabilities?|HERE]]. [[User:Dolphin51|<font color="green">''Dolphin''</font>]] ''([[User talk:Dolphin51|<font color="blue">t</font>]])'' 00:30, 1 July 2010 (UTC)

= July 1 =

== Iron ==

Does the body use iron(II) or iron(III)? --[[Special:Contributions/75.25.103.109|75.25.103.109]] ([[User talk:75.25.103.109|talk]]) 00:18, 1 July 2010 (UTC)
:<s>Both..</s><sub>oops</sub> Fe2+ probably = See [[Hemoglobin]] - especiallly [[Hemoglobin#Iron's_oxidation_state_in_oxyhemoglobin]] . [[User:Sf5xeplus|Sf5xeplus]] ([[User talk:Sf5xeplus|talk]]) 00:21, 1 July 2010 (UTC)
::Both. Most iron containing proteins are (nominally) iron(II), but iron is transported around the body (by [[transferrin]]) and stored (by [[ferritin]]) as iron(III). [[User:Physchim62|Physchim62]] [[User talk:Physchim62|(talk)]] 10:24, 1 July 2010 (UTC)

== Expiration of medicine ==

What does it mean for medicine to expire? Assuming we are dealing with a pharmaceutical that doesn't become harmful as it ages, how much of the active ingredients can be lost before it is considered expired? <span style="font-size: smaller;" class="autosigned">—Preceding [[Wikipedia:Signatures|unsigned]] comment added by [[Special:Contributions/173.49.77.61|173.49.77.61]] ([[User talk:173.49.77.61|talk]]) 00:22, 1 July 2010 (UTC)</span><!-- Template:UnsignedIP --> <!--Autosigned by SineBot-->
:Usually drugs just become less effective as they get older. Some could become harmful though - don't risk it! --[[User:Tango|Tango]] ([[User talk:Tango|talk]]) 00:27, 1 July 2010 (UTC)
::Why do they get less effective? --[[User:RampantHomo|RampantHomo]] ([[User talk:RampantHomo|talk]]) 01:14, 1 July 2010 (UTC)
:::Various reasons. Oxidation, decay due to exposure to light (that's why they are often in dark brown bottles), etc.. --[[User:Tango|Tango]] ([[User talk:Tango|talk]]) 02:23, 1 July 2010 (UTC)

:How about a reference here at the Reference Desk? The first google hit on ''why does medicine expire'' yields [http://www.medscape.com/viewarticle/460159 this Medscape article], in which a medical doctor concludes that almost all drugs are fine to take after the expiration date, which he says is merely the last date at which the pharmaceutical company asserts the drug is still effective. On the other hand, he cites one example in which a guy may have damaged his kidneys by taking expired [[tetracycline]]. [[User:Comet Tuttle|Comet Tuttle]] ([[User talk:Comet Tuttle|talk]]) 03:01, 1 July 2010 (UTC)
::As an example, I recall from my chemistry class that [[Hydrogen peroxide]] molecules (H<sub>2</sub>0<sub>2</sub>) will drop the extra Oxygen atom at somepoint, and as this happens to collectively more Hydrogen Peroxide molecules, the contents of the bottle become more and more water, and less and less hydrogen peroxide, thereby significantly reducing the effectiveness over time. <font color="009900"><b>Falconus</b></font><sup>[[User:Falconus|<font color="000000"><b>p</b></font>]] [[User talk:Falconus|<font color="000000"><b>t</b></font>]] [[Special:Contributions/Falconus|<font color="000000"><b>c</b></font>]]</sup> 03:34, 1 July 2010 (UTC)
:::Hydrogen peroxide is a special case - it's very unstable. Most (pretty much all) medicines are far far more stable. Concentrated hydrogen peroxide is unstable enough to be used as rocket fuel. [[User:Ariel.|Ariel.]] ([[User talk:Ariel.|talk]]) 10:32, 1 July 2010 (UTC)
::::Important distinction: it's important that a rocket fuel is "high energy" (thermodynamically unstable) not that it rapidly/spontaneously decomposes (somewhat unrelated kinetic issue, which is sometimes even a negative quality to have). Lots of excellent rocket fuels are easy to handle and store until they are made to react with a large release of energy. Some fairly low-energy molecules are labile enough that they can change slightly under mild conditions, leading to loss of intended activity. [[User:DMacks|DMacks]] ([[User talk:DMacks|talk]]) 14:17, 1 July 2010 (UTC)
:::::I should have said "unstable enough to be used as a [[monopropellant]] rocket fuel". [[User:Ariel.|Ariel.]] ([[User talk:Ariel.|talk]]) 14:52, 1 July 2010 (UTC)
::::::If the drug is ''not'' harmful after it expires, it may even still work due to the [[placebo effect]]. ~<font color="blue">[[User:AstroHurricane001/A|A]][[User:AstroHurricane001|H]][[User:AstroHurricane001/D|1]]</font><sup>([[User:AstroHurricane001/T|T]][[Special:Contributions/AstroHurricane001|C]][[User:AstroHurricane001/U|U]])</sup> 18:06, 2 July 2010 (UTC)

==Help identifying this flower==
[[File:UnknownBlueFlower HSVLevels.jpg|thumb|right|300px]]
I would like to know the species of this flower so I can add the image to the correct article, but I haven't got a clue! The background info for this photo is:
*In the UK in Oxfordshire
*Taken in March
*Found in open cut grass
Thanks in advance - '''[[User:Zephyris|Zephyris]]''''' <sub>[[User talk:Zephyris|Talk]]</sub>'' 00:47, 1 July 2010 (UTC)

:Is it not [[speedwell]] ([[Veronica (plant)]] of some variety)? It looks like the thing I've always called speedwell, and if I had to guess from the articles I'd say it was [[Veronica persica]]. But hopefully a botanist will help you with the more specific species. [[Special:Contributions/86.164.57.20|86.164.57.20]] ([[User talk:86.164.57.20|talk]]) 01:40, 1 July 2010 (UTC)
::Looking it up in Complete British Wild Flowers by Paul Sterry, which has photos of many kinds of Speedwell, then it could be Veronica persica, polita, or agrestis. V. persica - reddish stems, "white on the lower lip of the corolla". Cannot see a reddish stem, but the other part matches. The photo shown in that Wikipedia article is perhaps the wrong species therefore. [[Special:Contributions/92.28.244.45|92.28.244.45]] ([[User talk:92.28.244.45|talk]]) 09:57, 1 July 2010 (UTC)
::Thanks guys, thats brilliant. From those descriptions it must be ''V. polita'', I will replace the image on that page with this one. - '''[[User:Zephyris|Zephyris]]''''' <sub>[[User talk:Zephyris|Talk]]</sub>'' 13:13, 1 July 2010 (UTC)
:::I thought I had typed V. persica above, not polita. Anyway I have now corrected it, and the desrciption corresponds to V. persica, and the V. polita article photo needs to be changed back. [[Special:Contributions/92.28.247.183|92.28.247.183]] ([[User talk:92.28.247.183|talk]]) 19:15, 2 July 2010 (UTC)

== Neodymium magnets ==

Do the neodymium and iron in neodymium magnets (Nd<sub>2</sub>Fe<sub>14</sub>B) oxidize over time? --[[Special:Contributions/75.25.103.109|75.25.103.109]] ([[User talk:75.25.103.109|talk]]) 01:02, 1 July 2010 (UTC)
:Not sure but they are almost always coated (ie Ni plated)...
:"'' Neo magnets or rare earth magnets have poor resistance to corrosion and should have a coating or plating applied''" [http://www.duramag.com/neodymium.html] . Not sure of the details.[[User:Sf5xeplus|Sf5xeplus]] ([[User talk:Sf5xeplus|talk]]) 01:31, 1 July 2010 (UTC)

:"''Sintered Nd2Fe14B tends to be vulnerable to corrosion. In particular, corrosion along grain boundaries may cause deterioration of a sintered magnet. This problem is addressed in many commercial products by providing a protective coating. Nickel plating or two layered copper nickel plating is used as a standard method, although plating with other metals or polymer and lacquer protective coatings are also in use''" from [[Neodymium magnet]] .. link from that article http://www.journalamme.org/papers_vol20/1369S.pdf [[User:Sf5xeplus|Sf5xeplus]] ([[User talk:Sf5xeplus|talk]]) 01:33, 1 July 2010 (UTC)

::From first hand experience: Yes they rust! I have several of those magnets ripped out of hard disks. The glue they use to fasten it to the holder is very very good so I torn of the nickel layer and now the magnet is not protected at that point. After several weeks this area get redish coloured and if you touch it you have rust on your fingers. The material is sintered and has a large surface area making it vulnerable for corrosion along the former grains.--[[User:Stone|Stone]] ([[User talk:Stone|talk]]) 05:05, 1 July 2010 (UTC)

== Weight loss ==

I was watching a comedy/sitcom TV series and there is a woman who is a bit...*ahem*... "un-thin" but she is not at all overweight or fat. She came up with this insane regimen to lose weight, eg eating nothing but 1 salad and coffee 3 days of the week and eating normally the other 4 but running 3 miles and biking 10. This made me think: how would a person who is of a somewhat higher but not immediately health-threatening weight lose weight? Like what kind of everyday changes should they make. This is on a hypothetical level of course, not for medical advice, since I noticed every diet seems to be geared only to the REALLY overweight people.<span style="font-size: smaller;" class="autosigned">—Preceding [[Wikipedia:Signatures|unsigned]] comment added by [[Special:Contributions/ 68.248.225.254| 68.248.225.254]] ([[User talk: 68.248.225.254|talk]]) 01:57, 1 July 2010</span><!-- Template:UnsignedIP2 -->

:There is one and only one way of losing weight and it applies to everyone, regardless of their starting weight: you consume fewer calories than you burn. That is it (well, you could get liposuction, I guess). You can do that by eating less, exercising more or a bit of both. The important thing is to make sure you still get enough nutrition despite reduced calories - you still need plenty of protein, fibre, vitamins and minerals and a little fat. If you are interested in losing weight yourself then a nutritionist will be able to work out a diet for you that will have all the nutrients you need while having a certain number of calories based on the amount of exercise you will be doing. We can't give specific dietary advice here - there are too many factors involved. --[[User:Tango|Tango]] ([[User talk:Tango|talk]]) 02:31, 1 July 2010 (UTC)
::The real trick is finding a method to ensure you actually ''do'' eat less than you expend. The diet mentioned above might work for that for that or another person, it might not. I've been fat since high school and tried several diets and exercise regimes and found the following: Exercise 3 times a week works a little for me, and at least makes me feel better. Low calorie diets don't work for me (I can't handle being always hungry). Low fat diets don't work for me (I can easily get or stay fat on bread alone!). Low [[carbohydrate]] diets (specifically strictly following the [[Atkins diet]]) worked (120kg to 98kg over 3 months) for me on my third attempt - it took me that long to learn that the diet doesn't work if I cheat or if I don't eat often enough. But everyone is different.
::Every day changes that I've seen help people: Exercising more - cycling or walking to work or school; not snacking; eating 5 times a day rather than 3; eating only and exactly 3 times a day; not eating high fat food; not eating high calorie food; not eating processed food; cutting out a particular food (crisps, chocolate, bread, etc). --[[User:Psud|Psud]] ([[User talk:Psud|talk]]) 08:53, 1 July 2010 (UTC)

:The general view is that ''diets don't work'' lifestyle changes do. There seem to be three main areas, actual nutrition, exercise and the psychology of nutrition and exercise.
:The main thrust of most of the work I've seen has been that the most enduring weight loss is slow, over several months.
:As with Tangos comments, it's a question of eating a balanced, healthy, diet and adding in some exercise. The exercise needn't be formal, but increasing exertion in daily life on a regular basis.
:One of the main concerns about the psychology is that published diets are seen as some kind of magic bullet, rather than a stimulus to actually change behaviours. Hence the type of club based weight loss system where there is both mutual support, and tacit peer pressure.
:As with Psud there are little changes, smaller more regular meals, although some of those are little more than formalised ''snacks''. Snacking on fruit, nuts etc rather than processed products etc.
:[[User:ALR|ALR]] ([[User talk:ALR|talk]]) 09:47, 1 July 2010 (UTC)
::Ah, yes, thank you for making that point - I forgot to. Your new diet needs to be something you can happily stick to for the rest of your life, or you will almost certainly put the weight back on again. --[[User:Tango|Tango]] ([[User talk:Tango|talk]]) 15:59, 1 July 2010 (UTC)

:I've recently started to think about a simple rule for health and weight loss: never eat processed foods. Only eat food in the same form it was alive, apart from cutting, and avoid meat also. Following this rule you would avoid the salt, fats, and sugars that routinely tempt us to eat too much. Something I do is to eat a lot of vegetables, as this fills you up, and fruits also. [[Special:Contributions/92.28.244.45|92.28.244.45]] ([[User talk:92.28.244.45|talk]]) 10:10, 1 July 2010 (UTC)
::While there is merit to the above, you need to be careful that you are still getting B vitamins and protein if you are going to cut out meat completely. While I agree that vegie diets are easier to reduce calories, you do need to be aware of the nutrition provided by meat and ensure that you are including it. [[User:Googlemeister|Googlemeister]] ([[User talk:Googlemeister|talk]]) 13:32, 1 July 2010 (UTC)
:::You are probably thinking of Vit B12, which is abundant in fish and eggs. People also greatly overestimate the amount and quality of protein that adults need (according to these pages), and excess protein is now thought to be bad for you. [[Sardine]]s are extremely rich in Vitamin B12, more than anything else you would eat I think. I suggested avoiding meat because of the saturated fat it contains, particularly with modern farming methods. The B vitamin [[folic acid]] is mostly obtained from vegetables I believe, as well as other B vitamins, although I am not an expert on individual B vitamins. If you are eating a diverse diet, then eating some [[red meat]] is going to be more harmful than good for you. [[Special:Contributions/92.28.247.183|92.28.247.183]] ([[User talk:92.28.247.183|talk]]) 19:22, 2 July 2010 (UTC)
I agree with ALR that diets don't work. I've read that this is because the logic of "eating less than you burn" does not apply. As long as you're not on a diet that severly restricts calory intake (say eating less than 1500 Kcal per day), the body will regulate its metabolic rate such that the fat cells stay filled to some fixed level. This also means that if you do lose weight as a result of sticking to a diet of, say, 1500 Kcal, you will eventually return to approximately your old weight if you start to eat a normal amount of kcals again, even if that is still less than what you were used to eating.

What does seem to work is eating healthier, sleeping better and getting more exercise. It could be that if you do this, the body will decide to keep the fat cells filled at a lower level. [[User:Count Iblis|Count Iblis]] ([[User talk:Count Iblis|talk]]) 15:39, 1 July 2010 (UTC)
:You make a good point. I forgot to say that you need to consume only slightly fewer calories than your burn - your intake should stay at this new level for the rest of your life. Rapid weight loss doesn't work, you will almost certainly put the weight back on again. Your weight is a key factor in determining how many calories you burn, so as you lose weight the calories you are burning will reduce until you reach a new equilibrium and then you'll stay at your new weight. If you reduce your calories to 1500kcal/day then you won't reach an equilibrium before you starve, which is why you'll end up increasing your intake again. For a typical lifestyle, most people need around 2000-2500kcal/day. If you are overweight, chances are you are consuming more than that (and burning more than that due to your weight - most overweight people aren't constantly gaining weight). You shouldn't try to reduce your intake to less than 2000-2500, you should reduce it ''to'' 2000-2500. --[[User:Tango|Tango]] ([[User talk:Tango|talk]]) 15:59, 1 July 2010 (UTC)
::See [[somatotype and constitutional psychology]]. For some people it will be more difficult to "lose weight". ~<font color="blue">[[User:AstroHurricane001/A|A]][[User:AstroHurricane001|H]][[User:AstroHurricane001/D|1]]</font><sup>([[User:AstroHurricane001/T|T]][[Special:Contributions/AstroHurricane001|C]][[User:AstroHurricane001/U|U]])</sup> 18:03, 2 July 2010 (UTC)

==Suntan==
I was playing tennis but I was wearing sandals and did not think to put sunscreen on my feet. Now they have some weird tanned stripes where the holes elt sun in. How long does it take to get "un-tan"? Is there any way I can speed up the process of getting un-tan, whih I assume is my body metabolizing melanin? THNX [[Special:Contributions/68.248.225.254|68.248.225.254]] ([[User talk:68.248.225.254|talk]]) 01:57, 1 July 2010
:It's going to depend on your exposure to sun and varies from person to person, but it can easily be weeks. It would be quicker to just tan the rest of your feet, but I wouldn't recommend that - intentionally exposing yourself to sunlight is generally not to be recommended. You could try a fake tan, of course. --[[User:Tango|Tango]] ([[User talk:Tango|talk]]) 02:38, 1 July 2010 (UTC)
::"Intentionally exposing yourself to sunlight is generally not to be recommended." Not recommend by who? Vampires? I am going to go out on a limb here and say that moderate exposure to the sun is generally not a major health concern. And I will base that statement off of the 6,000,000,000 or so people on this planet who encounter sunlight on a regular basis without keeling over. Granted if you are going to overdo it that results in a slight increase in your chance of skin cancer, but that doesn't mean you should not go out during the day. In fact, if you are not exposed to sunlight, you can have vitamin D deficiencies, so telling people not to go out in the sun can be detrimental to their health. [[User:Googlemeister|Googlemeister]] ([[User talk:Googlemeister|talk]]) 13:27, 1 July 2010 (UTC)
:::I said "intentionally". That means going outside for the purpose of exposing yourself to sunlight. Going outside for other reasons and just happening to be exposed to sunlight is a different matter entirely - you have to weigh up the benefit from whatever you are doing with the harm from the sunlight, and as long as you don't overdo it it is often worth going outside. The amount of sunlight required to get enough vitamin D is minimal - in fact, with a decent diet, it can be none at all, at most it's about 10 minutes with only your face and arms uncovered on an overcast day at temperate latitudes (for someone with reasonably fair skin). --[[User:Tango|Tango]] ([[User talk:Tango|talk]]) 16:06, 1 July 2010 (UTC)
::The cells in the epidermis (the outer layer of skin) don't "untan" - they just die and eventually slough off. That process takes 27 days after the cell is first formed. In effect, you grow an entire new skin every four weeks! So if an epidermis cell was 'tanned' right after it was formed then the tan might not completely fade for 27 days. I think that's the worst case - but in practice, I doubt the suns' rays are able to tan those deepest layers of skin, so probably only the outer layer of 'older' cells got tanned - and they'll be around for less than 27 days. [[User:SteveBaker|SteveBaker]] ([[User talk:SteveBaker|talk]]) 03:50, 1 July 2010 (UTC)

:Wash your feet a lot - it helps get rid of the surface of the skin. [[Special:Contributions/92.28.244.45|92.28.244.45]] ([[User talk:92.28.244.45|talk]]) 10:16, 1 July 2010 (UTC)

wth? How do you tan that quick? [[User:John Riemann Soong|John Riemann Soong]] ([[User talk:John Riemann Soong|talk]]) 16:02, 1 July 2010 (UTC)
:A few hours of strong sunlight is more than enough for someone with reasonably fair skin to tan. Remember, the OP wasn't necessarily tanned enough for someone to notice them as being significantly tanned, just enough for there to be a noticeable difference between neighbouring tanned and untanned areas of skin. --[[User:Tango|Tango]] ([[User talk:Tango|talk]]) 16:08, 1 July 2010 (UTC)
::Consistently wearing sandals in the summer will give you a "sandal tan". I find it usually fades by the wintertime. ~<font color="blue">[[User:AstroHurricane001/A|A]][[User:AstroHurricane001|H]][[User:AstroHurricane001/D|1]]</font><sup>([[User:AstroHurricane001/T|T]][[Special:Contributions/AstroHurricane001|C]][[User:AstroHurricane001/U|U]])</sup> 18:01, 2 July 2010 (UTC)

== Reptiles + amphibians ==

I currently have a 29 gallon fishtank that I'd like to transform into a aquaterrarium. I'm looking to drop the water level to about a fourth (and maintain the 7 fish I have now), add some newts and then put in 2-3 thick branches and have [[anole]]s in there as well. I'm under the impression that newts possess special toxic slime that they exude upon attack that will protect them from the anoles. Would that be a plausible set up? '''[[User:DRosenbach|<span style="color:#006400">DRosenbach</span>]]''' <sup>([[User_talk:DRosenbach|<span style="color:#006400">Talk</span>]] | [[Special:Contributions/DRosenbach|<span style="color:#006400">Contribs</span>]])</sup> 02:31, 1 July 2010 (UTC)
:Well many have poisonous skin eg [[Rough-skinned newt]] (also [[Newt#Toxicity]]), I suppose this would be easier to answer if you told us the type of newt.
:Even so it's not clear that the anole would attack the newt (relative sizes?), or indeed that it would be aware of a poison, or that the poison actually acts as a deterrent (eg bitter) rather than species [[attrition]].
:Additionally I would wonder if the anoles and newts would be compatible in terms of their ecosystem - eg dry/damp??
:[[Special:Contributions/87.102.17.114|87.102.17.114]] ([[User talk:87.102.17.114|talk]]) 11:47, 1 July 2010 (UTC)
I would say [[fire-bellied newt]]s, but I may get another type -- unsure. '''[[User:DRosenbach|<span style="color:#006400">DRosenbach</span>]]''' <sup>([[User_talk:DRosenbach|<span style="color:#006400">Talk</span>]] | [[Special:Contributions/DRosenbach|<span style="color:#006400">Contribs</span>]])</sup> 03:35, 2 July 2010 (UTC)

== Surface tension ==

On the surface of a liquid, there aren't any molecules above the surface molecules, hence they experience an attractive force towards the interior of the liquid. But what causes the increases force tangential to the surface? [[Special:Contributions/70.52.45.181|70.52.45.181]] ([[User talk:70.52.45.181|talk]]) 05:34, 1 July 2010 (UTC)
Further questions: I'm looking at the following link, http://www3.interscience.wiley.com:8100/legacy/college/cutnell/0471713988/ste/ste.pdf. When considering how to define surface tension, the article makes reference to a C-shaped apparatus. Why is γ = F/2l and not F/l? It says something about there being two surfaces, but I only see one. Second question: In example one, the article considers the surface tension as applying a force outwards. But I thought surface tension was only inwards. I can see why compressing the liquid will produce some outward force against the needle, but why would that equal γL? It seems like those are two different phenomena. Thanks a lot guys! [[Special:Contributions/70.52.45.181|70.52.45.181]] ([[User talk:70.52.45.181|talk]]) 06:36, 1 July 2010 (UTC)
:ok First question - there isn't really an attractive force towards the centre of the liquid (yes I know the pdf says that).. The molecules on the surface are in equilibrium, so the net force is 0. Obviously if you try to pull a molecule from the surface of the liquid then there will be a force resisting that. ''maybe this seems pedantic''
:Following on, still answering the first question: are you familiar with the Energy=force x distance relationship (''or'' Force=dE/dx) - I think it makes surface tension a lot easier to understand... if so you can assume that the molecules at the surface are at a higher energy (since they are surrounded by less molecules .. and molecules gain energy by interacting with those next to them) .. Then assume that this 'surface energy' is proportional to area ie E=kA .. (k is a constant for air/water interface) - if you can work with that then it is fairly easy to show the answer to the second question without "hand waving" - it also proves that water surfaces generally form the shape of lowest surface area (ie a water droplet is spherical).
::''if not a different explanation will be necessary - but it gets a bit fiddly explaning just with text''
:Your third question - is this about a needle in water? (ie Example 1) - again - if you consider the surface area you will see that the surface tension acts in such a way to minimise the surface area (in this case it means <u>flat</u>) - so if the needle pushes the water down, the surface tension will try push the water back up into flatness.
:Alternatively (3rd question) - try drawing the 'force arrows' in the water surface around the needle (as in diagram b page 2) - you will see that some of the force arrows are pointing upwards (more that 180 degrees of force arrows) - giving a net upwards force.[[Special:Contributions/83.100.252.42|83.100.252.42]] ([[User talk:83.100.252.42|talk]]) 11:24, 1 July 2010 (UTC)
::Sorry, if you don't mind I would like things to be kept to forces and such...grasping something in terms of energy and surface area-minimization is fine, but my understanding of this thing from a force perspective leads me askew, and that's something I have trouble living with. Sorry! [[Special:Contributions/65.92.5.151|65.92.5.151]] ([[User talk:65.92.5.151|talk]]) 03:26, 2 July 2010 (UTC) (PS my IP seems to be changing; don't worry it's OP, not an imposter :) ). [[Special:Contributions/65.92.5.151|65.92.5.151]] ([[User talk:65.92.5.151|talk]]) 03:27, 2 July 2010 (UTC)

== Plaiting and fluffiness ==

When my hair has been [[braid|plaited]] for more than a couple of hours, untwisting the plait and brushing my hair makes it go very fluffy. It eventually "calms down" hours later but can be pretty wild directly after this sequence.

My hair is not normally fluffy and it doesn't "fluff" after brushing any other time.

It's very wavy in thick sections when it comes out of the plait and I'm guessing this has something to do with the subsequent fluffiness, but I'm missing the connection or reason. Does anyone know why plaiting produces the effect? I tried searching for an answer, but it seems plaiting is a common ''answer'' to fluffiness and I therefore get a lot of false positives, :) [[User:Maedin|'''<font color="#4B0082">Mae</font><font color="#008080">din\</font>''']]<sup>[[User_talk:Maedin|talk]]</sup> 06:57, 1 July 2010 (UTC)

== Does a penis have other functions besides urination and copulation? ==

Does penis have any other functions besides urination and copulation? For example, helping maintain balance as a man ealks (nude), or being a primary way to get off access heat during a heat wave, or being a sensory organ regarding thr environment, like a nose, or - anything at all? [[Special:Contributions/92.224.206.50|92.224.206.50]] ([[User talk:92.224.206.50|talk]]) 07:21, 1 July 2010 (UTC)
:<small>Re "sensory organ": Two men were peeing off the end of a pier at night when one remarked, "Gee, the water is pretty cold." The other replied, "And deep, too." -- [[Special:Contributions/60.49.38.251|60.49.38.251]] ([[User talk:60.49.38.251|talk]]) 09:58, 1 July 2010 (UTC)</small>
::<small>Writing one's name in the snow. [[User:Cuddlyable3|Cuddlyable3]] ([[User talk:Cuddlyable3|talk]]) 10:36, 1 July 2010 (UTC)</small>
:Masturbation? Which is used by some as "stress reliever"? --[[User:Enric Naval|Enric Naval]] ([[User talk:Enric Naval|talk]]) 10:40, 1 July 2010 (UTC)

::The penis provides the site for visible [[circumcision]] that is performed for ritualistic or religious purposes. Any usefulness for maintaining balance is disproven by the considerable numbers of women and men with tight trousers who do not fall over. [[User:Cuddlyable3|Cuddlyable3]] ([[User talk:Cuddlyable3|talk]]) 10:43, 1 July 2010 (UTC)

:::As far as balance goes, if it's long enough, you can use it as a [http://www.urbandictionary.com/define.php?term=tripod tripod]. --[[User:Enric Naval|Enric Naval]] ([[User talk:Enric Naval|talk]]) 10:56, 1 July 2010 (UTC)

:Its dysfunctions and complications can be useful indicators of health problems; hypertension, for example, or a hormonal imbalance. More indicative, of course, if the onset of said side effect is sudden and/or the age isn't elderly. Though clearly not its primary application, it's still a gauge that females don't necessarily have (or at least not as obviously). [[User:Maedin|'''<font color="#4B0082">Mae</font><font color="#008080">din\</font>''']]<sup>[[User_talk:Maedin|talk]]</sup> 12:33, 1 July 2010 (UTC)

:<small>It functions as a differential analyzer of the attributes "surprise" and "panic". Surprise is the first time it won't the second time and panic is the second time it won't the first time. [[User:Cuddlyable3|Cuddlyable3]] ([[User talk:Cuddlyable3|talk]]) 13:13, 1 July 2010 (UTC) </small>

::It's also a good [[MacGuffin]] for troll-like questions. --[[User:Tagishsimon|Tagishsimon]] [[User_talk:Tagishsimon|(talk)]] 09:45, 2 July 2010 (UTC)

== Credibility of hypotheses ==

What is it about a particular scientific hypothesis that makes it likely to reflect reality, or unlikely to reflect reality?--[[Special:Contributions/220.253.100.166|220.253.100.166]] ([[User talk:220.253.100.166|talk]]) 08:49, 1 July 2010 (UTC)

: Experiment :) --[[User:Dr Dima|Dr Dima]] ([[User talk:Dr Dima|talk]]) 10:12, 1 July 2010 (UTC)
::See the article [[Falsifiability]]. [[User:Cuddlyable3|Cuddlyable3]] ([[User talk:Cuddlyable3|talk]]) 10:34, 1 July 2010 (UTC)

:A hypothesis is just an idea - until it's tested in some way, we don't know whether it's true or false. However, some hypotheses are more likely to be true than others. For example: You might hypothesize that I'll type the letter "e" more than once in the next sentence. Sadly, that turns out to be wrong. It was a good hypothesis - but until it was tested, we didn't know whether it was true or false. In general, a hypothesis that requires a violation of the known laws of physics/chemistry/whatever is much less likely to be false than one that doesn't. Overturning some piece of well-established science is much less likely than proving something that's already known to be possible. If you hypothesize that T.Rex was able to run at 20mph, that's not an impossible thing - and by measuring fossil bones and looking for T.Rex footprints - you might be able to prove it...but if you hypothesize that the jet of gasses from a pulsar shoot out at twice the speed of light - then we're very nearly certain that this is false - just from general physics principles. [[User:SteveBaker|SteveBaker]] ([[User talk:SteveBaker|talk]]) 19:09, 1 July 2010 (UTC)

: Only an omniscient being could answer this question, since the rest of us have no sure knowledge of the underlying reality. The hypotheses are the only scientific guide we humans have to reality. [[Occam's razor]] is helpful in keeping things simple. [[User:William Avery|William Avery]] ([[User talk:William Avery|talk]]) 19:54, 1 July 2010 (UTC)

== Career in nanotechnology? ==

[I'm asking this question on behalf of a friend] I'm almost done my undergraduate studies in physics and I have started, for the first time really, to consider my career options. I've always enjoyed physics, and I would absolutely love to continue doing research in it, but I have been cautioned about the dangers of entering a overly theoretical discipline of physics. But there are plenty of fields with "real-word applications" that are certainly very interesting. In particular, something like condensed matter physics or theoretical chemistry seem worthwhile to pursue. However, a few weeks ago an old teacher of mine suggested that I look into nanotech, nanoelectronics in particular. It seems fascinating, and the technology and lab equipment are very impressive. My sole caveat is this: will I lose the physics I enjoy if I go into this? I want to work with the equations and study the mechanics of things...will that be jeopordized in something like nanotech? Thanks. [[Special:Contributions/70.52.45.181|70.52.45.181]] ([[User talk:70.52.45.181|talk]]) 10:52, 1 July 2010 (UTC)
:Nanoelectronics involves or can involve <u>a lot</u> of condensed matter physics, and some theoretical chemistry (of the right sort) is also applicable - so your friend can carry on as normal :)
:[[Special:Contributions/83.100.252.42|83.100.252.42]] ([[User talk:83.100.252.42|talk]]) 11:31, 1 July 2010 (UTC)
::Nanotech is a wide range of different things - from the very practical stuff like improved water filtration for use in the third world - to the highly theoretical stuff like [[Molecular assembler]]s. See [[List of nanotechnology applications]]. These days, saying that you want to work in Nanotechnology is like saying you want to do Math...it's used all over the place in a vast range of industries. I think this means that you should be able to find a niche somewhere between the so-practical-it's-just-engineering to the so-unlikely-that-research-funding-is-unobtainable. Somewhere in that spectrum, you should be able to find a job that pays well '''and''' lets you do the science you love. In case you didn't already check it out, we have lots of articles on the subject, including [[Nanoelectronics]] itself - and also [[Molecular electronics]], [[Molecular logic gate]], [[Molecular wires]], [[Nanocircuitry]], [[Nanowires]], [[Nanolithography]]. [[NEMS]], [[Nanosensor]], [[Nanoionics]], [[Nanophotonics]] and [[Nanomechanics]]. [[User:SteveBaker|SteveBaker]] ([[User talk:SteveBaker|talk]]) 18:57, 1 July 2010 (UTC)
Thanks to both of you for the informative and detailed responses. [[Special:Contributions/65.92.5.151|65.92.5.151]] ([[User talk:65.92.5.151|talk]]) 03:23, 2 July 2010 (UTC)

== Wasps ==

A wasp (either a [[yellowjacket]] or [[common wasp]], I think) somehow got into my bed this morning, with predictably hilarious consequences. While I treated the stings with [[hydrocortisone]] cream, they were still extremely painful, even after the swelling had gone down. So, my question: Bees use [[apitoxin]] and wasps apparently use "[[Venom|a chemically different venom designed to paralyze prey, so it can be stored alive in the food chambers of their young]]". What is the mechanism which causes the prolonged pain in bee and wasp stings? ''Apitoxin'' says that bee venom contains proteins which cause inflammation, but that some of the components are anti-inflammatory agents. Why this apparent contradiction? --<span style="font-weight:bold;">[[User:Kateshortforbob|<span style="color:#B96A9A;">Kateshortforbob]] <sub>[[User_talk:Kateshortforbob|talk]]</sub></span></span> 11:10, 1 July 2010 (UTC)
:I'm no doctor but from the sound of the article, apitoxin is multi-faceted. Some ingredients are anti-inflammatory while others are inflammatory, but not all work at the same time or on the same tissue. I suspect this dichotomy is part of why it hurts so much, the venom does different things at different stages ensuring that the wound will continue to cause pain and elude the body's self defense mechanisms. --[[Special:Contributions/144.191.148.3|144.191.148.3]] ([[User talk:144.191.148.3|talk]]) 18:52, 1 July 2010 (UTC)

== A hydrophobic cup? ==

What if a drinking cup has its inner surface lined with a hydrophobic material and water is poured into it? [[Special:Contributions/67.243.7.245|67.243.7.245]] ([[User talk:67.243.7.245|talk]]) 12:34, 1 July 2010 (UTC)
:You've never drunk out of a [[polystyrene]] cup? [[User:Physchim62|Physchim62]] [[User talk:Physchim62|(talk)]] 12:41, 1 July 2010 (UTC)

:Cup holds water then. This happens a lot - paper cups with water proof coatings for instance. Was there some particular detail you where interested in? [[User:Sf5xeplus|Sf5xeplus]] ([[User talk:Sf5xeplus|talk]]) 12:54, 1 July 2010 (UTC)
:[[Paper cup]] describes all the hellish details in full measure.[[User:Sf5xeplus|Sf5xeplus]] ([[User talk:Sf5xeplus|talk]]) 12:55, 1 July 2010 (UTC)
::But water remains in a PS cup. It's not completely hydrophobic then? [[Special:Contributions/67.243.7.245|67.243.7.245]] ([[User talk:67.243.7.245|talk]]) 20:53, 1 July 2010 (UTC)
:::It is completely hydrophobic. Hydrophobic materials do not counteract the force of gravity; they merely are not miscible with water. That's all hydrophobic means. It just means that the material does not dissolve in or mix with water. They do not ''repel'' each other with a force in the way that, say, two north poles of a magnet will. The simply don't mix. When oil (a hydrophobic substance) and water are shaken, the seperate, but not from any force more complex than gravity; the less dense oil floats to the top because it is less dense, and thus lighter per unit volume, than the water. They aren't "forced apart" by any force. Likewise, water placed in a hydrophobic cup will simply sit in the cup and not soak through it. If you made a cup of hydrophilic material, like say salt, the water would dissolve the cup, making it not very useful as a cup. --[[User:Jayron32|<font style="color:#000099">Jayron</font>]]'''''[[User talk:Jayron32|<font style="color:#009900">32</font>]]''''' 05:54, 2 July 2010 (UTC)
::You will get a convex [[Meniscus]]. Conversely, in a hydrophilic cup the meniscus will be concave. [[User:Ariel.|Ariel.]] ([[User talk:Ariel.|talk]]) 14:50, 1 July 2010 (UTC)

the cup shivers uncontrollably and cowers away from the water, if you persist in pouring water onto it, it "shuts down", curling into the fetal position and crying silently. If it is a prolonged experience in which the cup cannot get away, it can be permanently deformed psychology, perhaps hardly speaking a word for the rest of its life (or until treated). medications can help, but "water therapy", the naive idea that you should fight fire with fire, and just immerse the cup completely until it overcomes its fear, has disastrous results and is totally discredited; don't even think about it. [[Special:Contributions/92.230.234.237|92.230.234.237]] ([[User talk:92.230.234.237|talk]]) 13:08, 1 July 2010 (UTC)
:<small>be VERY careful around [[Rabies|hydrophobic]] cups, in case they bite you.... [[User:Physchim62|Physchim62]] [[User talk:Physchim62|(talk)]] 16:15, 1 July 2010 (UTC)</small>

== Help identifying this fruit ==

[[File:Unknown Fruit.jpg|thumb|right|300px|Unknown fruit.]]
I took this picture of a fruit on a tree in the botanic gardens in Oxford in February and have absolutely no idea what it is! Can anyone help? The fruit itself was around 4-5 cm diameter. Unfortunately as it was in the botanic gardens I have no idea of where in the world it is native to and the tree itself was fairly nondescript; ~7-10 metres tall with a trunk diameter of around 40-50 cm. The time of the year also meant there were no leaves for reference although the fruit was happy to stay on the tree over the winter - it was absolutely covered in these fruits. - '''[[User:Zephyris|Zephyris]]''''' <sub>[[User talk:Zephyris|Talk]]</sub>'' 13:21, 1 July 2010 (UTC)
:Umm... not to be glib but if you were at the [[Oxford Botanical Gardens]] then why did not you just ask a botanist? [[Special:Contributions/76.199.154.122|76.199.154.122]] ([[User talk:76.199.154.122|talk]]) 17:46, 1 July 2010 (UTC)
::Well I took a picture of what I thought was the species label for the tree... Turned out to be the species label for the miniature daffodils under the tree! - '''[[User:Zephyris|Zephyris]]''''' <sub>[[User talk:Zephyris|Talk]]</sub>'' 19:30, 1 July 2010 (UTC)
:Do you remember if the tree had thorns? If it did, this might be a [[Bael]]. [[User:Googlemeister|Googlemeister]] ([[User talk:Googlemeister|talk]]) 18:26, 1 July 2010 (UTC)
[[File:UnknownFruit2.jpg|thumb|right]]
::I don't think it did, see the additional image. - '''[[User:Zephyris|Zephyris]]''''' <sub>[[User talk:Zephyris|Talk]]</sub>'' 19:30, 1 July 2010 (UTC)
:Here's the number: 01865 286 690 you can ask them what's the tree above the daffodils. [[Special:Contributions/71.161.46.51|71.161.46.51]] ([[User talk:71.161.46.51|talk]]) 21:03, 1 July 2010 (UTC)
::You might also find their [http://www.botanic-garden.ox.ac.uk/ website] useful. It includes an interactive map (which regrettably does not identify individual plants) and an e-mail address for queries. [[Special:Contributions/87.81.230.195|87.81.230.195]] ([[User talk:87.81.230.195|talk]]) 05:43, 2 July 2010 (UTC)
:::They haven't been much help have they? My money is on the [[Dove tree|Dove-Tree]] or Ghost-Tree, ''Davidia Involucrata''[http://www.bambouseraie.com/Home/10/Redaction/Image/bambouseraie_davidia_involucrata.jpg.limit.800x600.jpg]. "Fruit: 3x2.8cm, obovoid (ie egg-shaped with the big end at the opposite end to the stalk), much ribbed, deep green and slightly glossy, ripening dark purple. Pedicel (ie stalk) 10-14cm, much swollen at the fruit end." (''A Field guide to the Trees of Britain & Northern Europe'', Alan Mitchell, Collins 1974). The English names come from the spectacular large white oval bracts in late May. "Also regrettably known as the 'Handkerchief Tree'" says Mr Mitchell. Discovered in W. China in 1904; the remarkable story of it's introduction to Western science can be found here[http://www.lmi.org.uk/medical_society/16/16Martin.pdf], pages 40-41. A photo of the Oxford Botanical Dove Tree in full foliage is here[http://www.flickr.com/photos/38592047@N08/3544300816/]. [[User:Alansplodge|Alansplodge]] ([[User talk:Alansplodge|talk]]) 18:48, 2 July 2010 (UTC)
::::The dove-tree looks very plausible! I think I would put my money on that... - '''[[User:Zephyris|Zephyris]]''''' <sub>[[User talk:Zephyris|Talk]]</sub>'' 22:20, 2 July 2010 (UTC)
:::::Found another picture[http://www.ukmaburbanforum.co.uk/docunents/presentations/streettrees/streetTrees_sophie_huxley.pdf]; scroll down to page 14. Is this the one? [[User:Alansplodge|Alansplodge]] ([[User talk:Alansplodge|talk]]) 22:44, 2 July 2010 (UTC)

:Could it be a variety of [[walnut]]? Walnut shells are hidden inside a covering that looks like a fruit when growing. [[Special:Contributions/92.28.247.183|92.28.247.183]] ([[User talk:92.28.247.183|talk]]) 20:03, 2 July 2010 (UTC)
::I thought of that but walnut trees have big chunky twigs[http://farm3.static.flickr.com/2373/1993188043_47a3625983.jpg], the nuts have short stalks and no ribs. [[User:Alansplodge|Alansplodge]] ([[User talk:Alansplodge|talk]]) 22:39, 2 July 2010 (UTC)

== Would the car explode? ==

If something happened and a car's intake valve didn't shut all the way (or the valve suddenly developed a serious fissure while the car was in use) and the compression stroke came back up and the spark plug went off, is there anything stopping the combustion flame from streaking back through the lines to the fuel tank and causing a catastrophe?[[Special:Contributions/20.137.18.50|20.137.18.50]] ([[User talk:20.137.18.50|talk]]) 14:04, 1 July 2010 (UTC)
:The fuel line is full of liquid fuel. You need air in order to support combustion, so it would be hard to propagate the flame back very far prior to where air is mixed with the fuel vapors. [[User:DMacks|DMacks]] ([[User talk:DMacks|talk]]) 14:09, 1 July 2010 (UTC)
::A [[Carburetor]] if there is one, also makes a barrier since the fuel must go through a narrow nozzle.[[Special:Contributions/87.102.17.114|87.102.17.114]] ([[User talk:87.102.17.114|talk]]) 14:26, 1 July 2010 (UTC)
:::If a valve gets stuck open, you don't get compression and the fuel doesn't ignite properly. Aside from the fact of there being no air in the fuel lines, there isn't likely to be any combustion either. What happens is that the car sputters along, sounding really rough and developing very little power, because it's only running on three cylinders (assuming it's a four cylinder car that is!). Gasoline really doesn't burn very easily unless it's under hot or under pressure and in vapor form mixed with air. [[User:SteveBaker|SteveBaker]] ([[User talk:SteveBaker|talk]]) 18:39, 1 July 2010 (UTC)
:A [[backfire]] in the intake is not unheard of on poorly maintained cars for a variety of reasons (such as [[ignition timing|timing]], like you mentioned), and while it can easily damage intake manifold parts like rubber seals or sensors it is not likely to cause the fuel lines to catch fire for the aforementioned reasons. --[[Special:Contributions/144.191.148.3|144.191.148.3]] ([[User talk:144.191.148.3|talk]]) 18:42, 1 July 2010 (UTC)
::Yep. Remember - the car is a four-stroke machine. On the first down-stroke, gas/air is sucked into the cylinder. On the up-stroke, it's compressed...only the valve isn't closed so this just pushes most of the gas/air mixture back out again. Then the spark goes off - but there's hardly any gas/air mixture to ignite - and it's not hot and under pressure like it should be - so it probably won't ignite. The lack of a fuel burn means that there is nothing to push the piston back down again - aside from the inertia of the car and flywheel...but it'll go back down - sucking in whatever fuel and air is left in the cylinder. Then the exhaust valve opens and the piston goes back up - pushing most of whatever was in there out to the exhaust. But not all of the exhaust gasses will go out - lots of them will be pushed back to the carb. [[User:SteveBaker|SteveBaker]] ([[User talk:SteveBaker|talk]]) 18:45, 1 July 2010 (UTC)

== How much energy required to reach infinite speed? ==

I can't remember the equation for how much energy is required to reach a given speed. What is it again? And what do you get when you feed infinity through it? Presumably no answer, since reaching c takes infinite energy.--[[Special:Contributions/92.251.129.172|92.251.129.172]] ([[User talk:92.251.129.172|talk]]) 15:04, 1 July 2010 (UTC)
:E=1/2 mv<sup>2</sup> where E=kinetic energy, m = mass and v= velocity. So, yes, at v= infinity, E = infinity. This is the ''non-relativistic'' definition of energy and velocity, so I am sure there are adjustments to be made for speeds close to light speed, however these adjustments actually make it worse; you hit infinite energy at the speed of light (3.00 x 10<sup>8</sup> meters per second). So, you cannot ever reach the speed of light. It's a pain in the ass for science fiction authors, but the speed of light is a hard limit. --[[User:Jayron32|<font style="color:#000099">Jayron</font>]]'''''[[User talk:Jayron32|<font style="color:#009900">32</font>]]''''' 15:13, 1 July 2010 (UTC)
::I thought they usually invented some warp drive or hyperdrive that somehow let them past lightspeed?--[[Special:Contributions/92.251.129.172|92.251.129.172]] ([[User talk:92.251.129.172|talk]]) 15:25, 1 July 2010 (UTC)
::::: Such things are utterly impossible given the laws of physics as we know them. But science fiction is '''''fiction''''' (meaning "it ain't all true"!)...and they are free to invent anything they want in order to make the plot work...and hand-waving away the speed of light limit is something you pretty much have to start off doing if you want people to go to other stars. A few science fiction authors don't do that...but they are certainly in the minority. [[User:SteveBaker|SteveBaker]] ([[User talk:SteveBaker|talk]]) 18:27, 1 July 2010 (UTC)
::::::Hence "invented".--[[Special:Contributions/92.251.158.103|92.251.158.103]] ([[User talk:92.251.158.103|talk]]) 19:59, 1 July 2010 (UTC)
:::Yes. The one [[E.E. Doc Smith]] invented for [[Lensmen]] just got rid of the pesky m in the above equation, so you only have to overcome the resistance of the interstellar medium. Unfortunately, they answer still is infinite (and the original question is physically meaningless - you cannot go faster than the speed of light without violating causality, no matter what your trick is). --[[User:Stephan Schulz|Stephan Schulz]] ([[User talk:Stephan Schulz|talk]]) 15:29, 1 July 2010 (UTC)
::::Well, there's a hidden assumption there that your "trick" is frame-independent. It's imaginable that there is some as-yet-undiscovered physical process that is ''not'' frame-independent, and that defines a preferred frame of reference which we have not yet been able to detect. Then you might be able to use that process to send information at arbitrary speeds ''in that preferred frame'', but unlike in the case of the [[tachyonic antitelephone]], you might not be able to "close the loop" to send information back to where it started from (which would allow the [[grandfather paradox]]).
::::In this scenario (for which, certainly, I am not claiming there is any evidence at all; this is a thought experiment), there would still be some observer, in motion relative to the preferred frame, who could observe an effect occurring before its cause in his time coordinate. But so what? That's just a coordinate. As far as I can see, there is no danger of paradox unless you can create a causal loop.
::::Another possible way out is, keep frame invariance, but throw out isotropy &mdash; the magic FTL drive works only when you're headed towards cosmic right; the usual speed limit applies to cosmic left. --[[User:Trovatore|Trovatore]] ([[User talk:Trovatore|talk]]) 03:44, 2 July 2010 (UTC)
:The relativistic formula for kinetic energy is:

::<math>E_k = \frac{m c^2}{\sqrt{1 - (v/c)^2}} - m c^2 </math>.

:Formally, setting ''v'' to infinity gives you a <u>negative</u> kinetic energy of -''mc''<sup>2</sup> ! But this is meaningless, because ''E''<sub>''k''</sub> tends to infinity as ''v'' approaches ''c'', so you can't go through the "light barrier". [[User:Gandalf61|Gandalf61]] ([[User talk:Gandalf61|talk]]) 15:30, 1 July 2010 (UTC)
::<small> so we can create a perpetual motion machine! All we need to do is get something to infinite velocity, and it will generate tons of free energy! Or maybe we need to use negative energy to get something going that fast. [[User:Googlemeister|Googlemeister]] ([[User talk:Googlemeister|talk]]) 15:43, 1 July 2010 (UTC) </small>

:::No - none of those things work. The answer isn't infinity - it's the square root of some negative quantity. If (v &gt; c) then (v/c)<sup>2</sup> is greater than one - so 1-(v/c)<sup>2</sup> is negative. If you take the square root of a negative number on your pocket calculator, it probably says "E" or "Error". You get a [[complex number]] - and those can't exist as actual quantities in the physical world - whenever you find a complex number as the final answer to a calculation, you know you've screwed up somewhere. That's '''WHY''' this equation says you can't travel faster than light. If the answer was merely infinity, we could possibly imagine some kind of meaning to the answer...but a complex number means "your math is broken somewhere" - and the reason it's broken is because you put a disallowed number into the equation. [[User:SteveBaker|SteveBaker]] ([[User talk:SteveBaker|talk]]) 18:27, 1 July 2010 (UTC)
::::I understood the answer would be meaningless I was jsut curious about what it would be.--[[Special:Contributions/92.251.129.172|92.251.129.172]] ([[User talk:92.251.129.172|talk]]) 18:41, 1 July 2010 (UTC)
::::As v approaches infinity, the imaginary part decreases without limit. Thus, the energy would be real if it's moving at infinite velocity. Of course, from any other point of reference, it's going at a finite, but faster than light, velocity, and it comes out imaginary, so it doesn't really help.
You can question why you would even want to move your rest mass. Arguably "you" are an algorithm that your brain is computing and that can be sent using photons from one machine to another machine at lightspeed. So, travel at lightspeed is possible. [[User:Count Iblis|Count Iblis]] ([[User talk:Count Iblis|talk]]) 15:47, 1 July 2010 (UTC)
:Well what they mean is ''matter'' cannot move at lightspeed. Although you could claim that since we are moving away from a certain galaxy at 0.6 c, and that same galaxy is moving the other way at 0.6 c, we are moving apart faster than the speed of light. ALthough that still isn't going the speed of light.-[[Special:Contributions/92.251.129.172|92.251.129.172]] ([[User talk:92.251.129.172|talk]]) 15:53, 1 July 2010 (UTC)
::You could ''claim'' that, but you'd simply be wrong, because [[relativistic velocities]] (i.e. ones that are a substantial fraction of the speed of light) ''don't'' obey simple arithmetic where 0.6 + 0.6 = 1.2; they obey more complex arithmetic where the answer can never be greater than 1.0: this seems counter-intuitive to us because our everyday logic is based on everyday experience which never includes relativistic velocities, but it has been extensively proved by observations and experiments. [[Special:Contributions/87.81.230.195|87.81.230.195]] ([[User talk:87.81.230.195|talk]]) 18:09, 1 July 2010 (UTC)
:::As I understand it, though, there is a sense in which a galaxy beyond the [[event horizon]] can be said to be moving away from us faster than the speed of light. It gets tricky to state exactly what this means. That general sort of question belongs to general rather than special relativity, and the math is hairy, and there's no longer always as clear a choice of coordinate systems as you might expect. --[[User:Trovatore|Trovatore]] ([[User talk:Trovatore|talk]]) 01:41, 2 July 2010 (UTC)
::::I think you mean a galaxy outside of our [[light cone]]. The event horizon is something different entirely. --[[User:Jayron32|<font style="color:#000099">Jayron</font>]]'''''[[User talk:Jayron32|<font style="color:#009900">32</font>]]''''' 01:53, 2 July 2010 (UTC)
:::::What I mean is outside the observable universe in a certain sense. I have never entirely gotten straight what is the accepted terminology on this, but our [[observable universe]] article appears to be talking about all objects that could potentially have been affected by an event in the past, by which we could also have been affected. I mean the time-reversed notion &mdash; all objects that could potentially influence a future event that we could also influence. The language at [[particle horizon]] suggests that ''event horizon'' is used in this sense. --[[User:Trovatore|Trovatore]] ([[User talk:Trovatore|talk]]) 01:57, 2 July 2010 (UTC)


:::::There is a connection with light cones, but no, "outside our light cone" is not the point exactly. The point is that our forward-facing light cone, and the forward-facing light cone of the galaxy in question, have empty intersection. --[[User:Trovatore|Trovatore]] ([[User talk:Trovatore|talk]]) 01:59, 2 July 2010 (UTC)
::::They're not moving, or at least not that fast. It's just that the intervening space is expanding. — [[User:DanielLC|DanielLC]] 07:42, 2 July 2010 (UTC)
:::::Well, I was careful to say "there is a sense" in which it's moving away from us faster than light. --[[User:Trovatore|Trovatore]] ([[User talk:Trovatore|talk]]) 07:49, 2 July 2010 (UTC)

== question about new planet ==

Time is talking about a new planet that someone got a photo of that is 500ly away. This planet is described as orbiting it's star at 330 AU, and has a surface temp of 2700K. To me this sounds more like a red dwarf star then a planet. Could someone familiar with the methodolgy give me a rough idea of how they can differentiate between a large planet and a very small star in orbit around another star (like Proxima Centauri)? [[User:Googlemeister|Googlemeister]] ([[User talk:Googlemeister|talk]]) 15:41, 1 July 2010 (UTC)

:Make life easy for us and give us a [http://www.time.com/time/health/article/0,8599,2000787,00.html?xid=rss-topstories link to the article], please!
:[http://news.nationalgeographic.com/news/2010/06/100630-science-space-planets-first-picture-confirmed/ This other article] (from ''National Geographic'') has a bit more detail about the planet. Its mass has been calculated at roughly eight times the mass of [[Jupiter]]. That's certainly big, but it's not 'star' big. Our own article on stars [[star#Mass|discusses the minimum stellar mass]], which is something like ''eighty'' times the mass of Jupiter. The boundary is governed by the minimum mass required to sustain stable nuclear fusion in the stellar core; the gravitational attraction holding the star together has to be high enough to balance the pressure generated by core hydrogen fusion. There is a gray area as you go to objects with masses lower than that, but bigger than [[gas giant]]s: the [[brown dwarf]]s. The intro to that article discusses the definitions (and the challenges associated with assigning objects to one category or the other). In general, the boundary between large gas giants and small brown dwarfs is taken to be at around 13 Jupiter masses.
:Finally, the high surface temperature of the planet in the story is a bit of an anomaly. It's not being heated (significantly) by internal fusion, nor is its heat drawn from the parent star. Instead, this is a very ''young'' planet (just a few million years), and its very high temperature comes from gravitational collapse. (The gravitational potential energy freed up as matter fell in to form the planet appears as heat.) The young Earth went through a similar period as it was forming billions of years ago, it took millions of years before the crust cooled and solidified. [[User:TenOfAllTrades|TenOfAllTrades]]([[User_talk:TenOfAllTrades|talk]]) 16:00, 1 July 2010 (UTC)
::I do kind of have to agree, though, that this is not what I think of as a referent for the word ''planet''. Really I think the IAU got the definition wrong in a number of ways &mdash; rather than distinguish between "planets" and "dwarf planets" on the basis of this fairly silly [[clearing the neighborhood]] concept, it would have been more revelatory to draw a line between the "real" planets (that is, the rocky planets), and the gas giants, which are just another sort of cat entirely. --[[User:Trovatore|Trovatore]] ([[User talk:Trovatore|talk]]) 02:09, 2 July 2010 (UTC)
:::Trovatore and his pet peeve. [[User:Dauto|Dauto]] ([[User talk:Dauto|talk]]) 04:07, 2 July 2010 (UTC)
:[[1RXS J160929.1-210524]], btw. --[[User:TotoBaggins|Sean]] 16:20, 1 July 2010 (UTC)

== Trench binocular ==

[[File:Scherenfernrohr (RaBoe).jpg|thumb|left]] [[File:British trench periscope Cape Helles 1915.jpg|thumb]]
The german [[:de:Scherenfernrohr]] has no interwikis - but I think this instrument is well known. See the picture on the right. Can you help? --[[User:Eingangskontrolle|Eingangskontrolle]] ([[User talk:Eingangskontrolle|talk]]) 16:56, 1 July 2010 (UTC)

:If you are asking what the instrument is, it is a [[periscope]].--[[User:Shantavira|Shantavira]]|[[User talk:Shantavira|<sup>feed me</sup>]] 17:08, 1 July 2010 (UTC)

:[[Periscope]] seems closest, or else [[binoculars]]. The first talks about use in [[trench warfare]] but does not mention the design that has separate periscope for each eye of the binocular. The second, in [[binoculars#Military]], specifically mentions this design and application, but is only a small part of a larger and wider-ranging article mostly about binoculars not the key periscope idea. [[User:DMacks|DMacks]] ([[User talk:DMacks|talk]]) 17:11, 1 July 2010 (UTC)
::...and it's not even mentioned in [[trench warfare]]. [[User:DMacks|DMacks]] ([[User talk:DMacks|talk]]) 17:17, 1 July 2010 (UTC)
: There doesn't seem to be a specific article in the EN Wikipedia. They're mentioned in passing, as "trench binoculars", in the [[binoculars]] article. Searching Google, there doesn't seem to be a general English language term for them, beyond "trench binoculars" or "rabbit ear scope". -- [[User:Finlay McWalter|Finlay McWalter]] • [[User talk:Finlay McWalter|Talk]] 17:13, 1 July 2010 (UTC)

:: Can someone who can read German more fluently than I check if [[:de:Scherenfernrohr]] and its cited ref seem [[WP:N|notable]] enough to make a [[Trench binoculars]] article? Although [[:File:British trench periscope Cape Helles 1915.jpg]] looks like a simple (monocular) design not binoculars...hard to see clearly, so maybe better to do [[trench periscope]] instead of specifically binocular. [[User:DMacks|DMacks]] ([[User talk:DMacks|talk]]) 17:15, 1 July 2010 (UTC)
:::::The link is available in english http://home.arcor.de/thuernagel/sf14-e.htm same person has a whole set of articles on "military optics" (or 'tactical optics') ''III Taktische Optiken'' http://home.arcor.de/thuernagel/katalog.htm . There's a mixture of reliable info and some small amount personal reflection/speculation in their (which is all probably completely right) website.
:::::Also known as "donkey ears" in the UK.
:::::Maybe a redirect to a new extended section in periscope? or a complete new article on 'tactical optics'?
:::::I can't see enough there as it is.. though the article would probably survive on it's own as a stub on the assumption that there is more info out there, and the obvious fact that such items are of social-historical significance.[[Special:Contributions/87.102.17.114|87.102.17.114]] ([[User talk:87.102.17.114|talk]]) 18:03, 1 July 2010 (UTC)
:::: It's my understanding that they're a bit more than just binoculars with a periscopic lightpath. I ''believe'' (but can't find a reliable source to adequately support, otherwise I'd have started on the article) that the rabbit-eared variety (like [http://news.webshots.com/photo/2322909020047818006SnUUtG this]) are intended for field artillery spotters. My understanding is that the two lenses independently targettable; the spotter picks his target and aligns both lenses on it - he can then read off the angle from a little gauge, look up the corresponding distance in a little trig table, and report that to the gunners. It's obviously not as accurate as a proper survey with two theodolites separated by a measured baseline, but it's going to be better (and quicker) than just guessing and having to walk trial shots up and down. -- [[User:Finlay McWalter|Finlay McWalter]] • [[User talk:Finlay McWalter|Talk]] 17:41, 1 July 2010 (UTC)
::::::I think it's just for observers - the wide position gives better stereoscopic view (ie less foreshortening) (Is this correct?)
::::::You're thinking it's a [[rangefinder]] - the person says ''Scherenfernrohre are the predecessors of the stereoscopical rangefinders'' - the same site has info on these - they can look very similar - but are more meaty - eg http://home.arcor.de/thuernagel/em61.htm .. they could use the thing as a rough rangefinder though,as the article says.[[Special:Contributions/87.102.17.114|87.102.17.114]] ([[User talk:87.102.17.114|talk]]) 18:03, 1 July 2010 (UTC)
::::::Some info with pictures through in english here http://www.paulstiger1.co.uk/WWII-Optics-Collection.htm and here http://www.fieldgear.org/optics.htm [[Special:Contributions/87.102.17.114|87.102.17.114]] ([[User talk:87.102.17.114|talk]]) 18:16, 1 July 2010 (UTC)
:::::::<small>The internet eh http://www37.atwiki.jp/strike_witches/m/plugin/ref/?guid=on&serial=921&w=500 ...</small> [[Special:Contributions/87.102.17.114|87.102.17.114]] ([[User talk:87.102.17.114|talk]]) 18:24, 1 July 2010 (UTC)
::::::If anyone is interested in writing the article on these particular type of thing then [[:de:Artillerietruppe_(Wehrmacht)]] is useful - it says that they were used by forward observers for artillery.
::::::Also searching "scissors telescope" shows that they were also used attached to tanks (with reliable sources, not forums).[[Special:Contributions/87.102.17.114|87.102.17.114]] ([[User talk:87.102.17.114|talk]]) 19:04, 1 July 2010 (UTC)
::::::It's a http://en.wiktionary.org/wiki/telestereoscope too.[[Special:Contributions/87.102.17.114|87.102.17.114]] ([[User talk:87.102.17.114|talk]]) 19:04, 1 July 2010 (UTC)

== Do the positions of stars change much relative to each other? ==

Will Alpha Centauri always be "about" the same distance from Earth? Suppose one picked any two stars at opposite sides of the galaxy. Does their position change much relative to each other? <span style="font-size: smaller;" class="autosigned">—Preceding [[Wikipedia:Signatures|unsigned]] comment added by [[Special:Contributions/92.251.129.172|92.251.129.172]] ([[User talk:92.251.129.172|talk]]) 18:28, 1 July 2010 (UTC)</span><!-- Template:UnsignedIP --> <!--Autosigned by SineBot-->

: The Wikipedia articles for many stars, such as [[Alpha Centauri]] gives their [[proper motion]] in the infobox at the right. -- [[User:Finlay McWalter|Finlay McWalter]] • [[User talk:Finlay McWalter|Talk]] 18:32, 1 July 2010 (UTC)
::So the sky will be very different in a million years time?--[[Special:Contributions/92.251.129.172|92.251.129.172]] ([[User talk:92.251.129.172|talk]]) 18:34, 1 July 2010 (UTC)
:::Yes; [http://www.astronomy.ohio-state.edu/~pogge/Ast162/Movies/proper.html this page] has movies of the Big Dipper over 200,000 years, and it changes substantially in that time. -- [[User:Coneslayer|Coneslayer]] ([[User talk:Coneslayer|talk]]) 18:49, 1 July 2010 (UTC)
::::See [[radial velocity]] for a sense of how the distances of stars change over time. Stars have even swung very close to the solar system in the past, and will likely do so again in the future&mdash;see [[Gliese 710]] (negative radial velocity) for example. ~<font color="blue">[[User:AstroHurricane001/A|A]][[User:AstroHurricane001|H]][[User:AstroHurricane001/D|1]]</font><sup>([[User:AstroHurricane001/T|T]][[Special:Contributions/AstroHurricane001|C]][[User:AstroHurricane001/U|U]])</sup> 17:42, 2 July 2010 (UTC)

== Tori Vienneau ==

was the Tori Vienneau murder case a suicide ?


http://www.youtube.com/watch?v=LM6bBVd0f7s <small><span class="autosigned">—Preceding [[Wikipedia:Signatures|unsigned]] comment added by [[User:Alexsmith44|Alexsmith44]] ([[User talk:Alexsmith44|talk]] • [[Special:Contributions/Alexsmith44|contribs]]) 18:30, 1 July 2010 (UTC)</span></small><!-- Template:Unsigned --> <!--Autosigned by SineBot-->
:Why are you asking us here? We're scientists, not [[columbo (TV series)|Columbo]]. [[Special:Contributions/87.102.17.114|87.102.17.114]] ([[User talk:87.102.17.114|talk]]) 18:35, 1 July 2010 (UTC)


law <small><span class="autosigned">—Preceding [[Wikipedia:Signatures|unsigned]] comment added by [[User:Alexsmith44|Alexsmith44]] ([[User talk:Alexsmith44|talk]] • [[Special:Contributions/Alexsmith44|contribs]]) 21:08, 1 July 2010 (UTC)</span></small><!-- Template:Unsigned --> <!--Autosigned by SineBot-->

== World building. ==

I'm sure questions like this have been asked many many times here, but I think mine is a little different.

Let's say I'm making a fictional planet and I want to make it as scientifically realistic as possible. I think I can handle most of the biological and geological topics, but mostly what I wanted to know about was meteorological and climatological. Things like ocean currents, jet streams, air masses and weather patterns, etc. What would the reference desk suggest as guidelines for creating the weather patterns on a fictional planet?

Also, I think I can handle most of the geology, but what confuses me most is plate tectonics. I can grasp the basic concepts of rifting and subduction and so on, but I don't think I know enough to create a fictional tectonic system on a fictional planet. This is important because drawing the maps of my world requires being consistent with tectonics.

[[Special:Contributions/63.245.168.34|63.245.168.34]] ([[User talk:63.245.168.34|talk]]) 19:42, 1 July 2010 (UTC)

:Make it pretty similar to Earth and you should be fine. If you stray from that, then really it's anybody's guess. We've only studied one planet in any great detail, so we don't know what else is possible. --[[User:Tango|Tango]] ([[User talk:Tango|talk]]) 19:52, 1 July 2010 (UTC)

::Are you sure about that? We know that weather patterns are defined by some pretty basic physical laws like the Coriolis Effect and so on, and we're familiar enough with these rules and how they effect the atmosphere and the oceans that we can predict the weather with reasonable certainty. Also, don't you think that making it too similar to Earth would be very unlikely? [[Special:Contributions/63.245.168.34|63.245.168.34]] ([[User talk:63.245.168.34|talk]]) 20:05, 1 July 2010 (UTC)

:::Not necessarily, perhaps scientifically, a planet can only be inhabitable if it is similar to earth. I mean how likely is it for life to develop on a planet with almost 0 tectonic activity? Of course if the book is not about humanss (or similar to human) colonizing a planet, but about a chlorine breathing insect race, then of course all bets are off. [[User:Googlemeister|Googlemeister]] ([[User talk:Googlemeister|talk]]) 20:57, 1 July 2010 (UTC)

:There is a whole book on this, created just for you: ''[http://www.amazon.com/World-Building-Science-Fiction-Writing/dp/158297134X/ref=pd_sim_b_1 World-Building: A writer's guide to constructing star systems and life-supporting planets]''. There are other books in the series with titles like ''Aliens and Alien Societies''. (I haven't read ''World-Building'' but have read ''Aliens''. The latter covers a lot of bases and made me consider aspects I hadn't thought about.) [[User:Comet Tuttle|Comet Tuttle]] ([[User talk:Comet Tuttle|talk]]) 21:16, 1 July 2010 (UTC)

:This is actually very difficult to deal with in a comprehensive way -- just consider how much the Earth's climate has varied over the past billion years, mainly as a consequence of the continents moving around. It is widely believed, for example, that the Earth was in a [[snowball Earth|snowball]] state from 800-600 million years ago, with the oceans frozen over nearly down to the equator, due to all of the continental landmass being concentrated near the equator. In short, you can get anything from rainforests at the poles to glaciers at sea level at the equator, just by shifting the land around. [[User:Looie496|Looie496]] ([[User talk:Looie496|talk]]) 22:41, 1 July 2010 (UTC)

::That's mostly what I'm curious about. How does the shape of the land affect the climate?

Also, as for the planet being inhabited or inhabitable, in the context of my fictional universe, it's a terraformed planet with a young, engineered biosphere. Only inhabited by species that were brought to it by humans. There are no aliens in my fiction.

[[Special:Contributions/63.245.168.34|63.245.168.34]] ([[User talk:63.245.168.34|talk]]) 00:12, 2 July 2010 (UTC)
:Land has profound effect on climate. Generally speaking, the direction of prevailing winds over landmasses determines things like where deserts and forests will appear. If prevailing winds have a long [[Fetch (geography)|Fetch]] over water, then the land downwind tends to be much rainier than if the winds have a long fetch over land. Also, there are effects like [[rain shadow]], which makes land upwind of a mountain range rainier than land downwind of the mountain range; which is why Seattle is much rainier than say Spokane. --[[User:Jayron32|<font style="color:#000099">Jayron</font>]]'''''[[User talk:Jayron32|<font style="color:#009900">32</font>]]''''' 01:06, 2 July 2010 (UTC)
::Also, land tends to heat up (and cool down) far quicker than water: you're better off swimming in the Mediterranean in October than in April, even if the land temperatures are similar. [[User:Physchim62|Physchim62]] [[User talk:Physchim62|(talk)]] 01:58, 2 July 2010 (UTC)
:::From observations on [[exoplanet]]s, astronomers can deduce a thermal "cimate" map of some planets, as they are hotter in some areas and cooler in others. However land and ocean have not yet been confirmed on any exoplanets, not even [[Super-Earth]]s, so the hypothetical planet should be like Earth in the sense that it has mountains and oceans. As for the effects of land patterns on the overall climate of regions, [[Wikipedia:Reference desk/Archives/Science/2008 March 13#climate modelling simulation map-based continental land-influenced scenario.3F|here]] is a question I posted here way back, when I was proposing a way to model the climate that is probably much less complicated than [[supercomputer]] simulations but do not involve mathematical formulae and calculations. As I never got an answer, I'm not sure how accurate my method is, but it should be a good general approximation of the climate of a planet given its land masses, but its functioning is reduced when less is known about the planet. Regular weather patterns can be inferred from the location of quasi-stationary highs and lows, but there will always be great variations. We know the general [[climate of Mars]], yet we still don't have the ability to predict global dust storms that often occur on the Red Planet or what exactly triggers them. Also, if the [[climate change]]s on your planet, the effects of that change are likely going to be harder to predict than just the climate map in a static climate. ~<font color="blue">[[User:AstroHurricane001/A|A]][[User:AstroHurricane001|H]][[User:AstroHurricane001/D|1]]</font><sup>([[User:AstroHurricane001/T|T]][[Special:Contributions/AstroHurricane001|C]][[User:AstroHurricane001/U|U]])</sup> 17:24, 2 July 2010 (UTC)

== Micro rockets ==

Suppose we want to send a miniscule payload from the surface of the Moon to the Space station. Then it seems to me that at least in theory, the rocket could be made very small. So, what is the minimum mass of a rocket that is able to let a payload of mass m (assume m < 1 microgram) escape from the Moon's gravity, as a function of m? [[User:Count Iblis|Count Iblis]] ([[User talk:Count Iblis|talk]]) 22:56, 1 July 2010 (UTC)

:What kind of fuel do you plan to use? By mass, do you include everything involved or just the rocket (projectile) that goes up into the air? I ask because different fuels have different masses. Also, assisted blastoff reduces fuel required. Further, the length of thrust is important. Do you want an explosion or a controlled thrust? -- [[User:Kainaw|<font color='#ff0000'>k</font><font color='#cc0033'>a</font><font color='#990066'>i</font><font color='#660099'>n</font><font color='#3300cc'>a</font><font color='#0000ff'>w</font>]][[User talk:Kainaw|&trade;]] 23:11, 1 July 2010 (UTC)
:For a payload that small, the payload is irrelevant. What matters is how light an engine you can make. The smallest rocket able to lift a payload of a microgram will be essentially the same size as one able to lift a payload of a few hundred grams. That said, I can't think of anything with a mass of less than a microgram that I would want to send from the Moon to a space station, certainly not on its own - just put it in with something else that is going. --[[User:Tango|Tango]] ([[User talk:Tango|talk]]) 23:15, 1 July 2010 (UTC)

::I see, so it is not possible to make microscopic engines? [[User:Count Iblis|Count Iblis]] ([[User talk:Count Iblis|talk]]) 00:59, 2 July 2010 (UTC)
:::I wouldn't say it's impossible, but I don't know of anyone ever doing so. There just isn't any reason to try. --[[User:Tango|Tango]] ([[User talk:Tango|talk]]) 01:19, 2 July 2010 (UTC)

:Just fire it from a rifle? (I don't know how the space station would catch the bullet!) [[User:Dbfirs|''<font face="verdana"><font color="blue">D</font><font color="#00ccff">b</font><font color="#44ffcc">f</font><font color="66ff66">i</font><font color="44ee44">r</font><font color="44aa44">s</font></font>'']] 07:13, 2 July 2010 (UTC)

::See [[Space gun]]. It's a lot harder to get things into space than people on here seem to realize. --[[User:Mr.98|Mr.98]] ([[User talk:Mr.98|talk]]) 11:37, 2 July 2010 (UTC)

:::This question is about launching from the Moon into the Moon's orbit. Gravity is 5/6 that on Earth and orbital distance is about 1/4 that of Earth. So, there are two things making it much easier. Gravity is much less. Orbit is much closer. Further, there is almost no atmospheric drag. So, I do not see why shooting a small projectile into orbit would be a big deal. -- [[User:Kainaw|<font color='#ff0000'>k</font><font color='#cc0033'>a</font><font color='#990066'>i</font><font color='#660099'>n</font><font color='#3300cc'>a</font><font color='#0000ff'>w</font>]][[User talk:Kainaw|&trade;]] 12:24, 2 July 2010 (UTC)

:I've read that there's a limit to how small you can make a rocket. Unfortunately, they didn't actually say what the limit was. That said, getting it off the moon is much less difficult then getting it off the Earth. — [[User:DanielLC|DanielLC]] 07:32, 2 July 2010 (UTC)

::Bottle rockets are pretty small... [[User:Googlemeister|Googlemeister]] ([[User talk:Googlemeister|talk]]) 18:17, 2 July 2010 (UTC)

== Why does the ocean appear so high? ==

I took this picture of the Atlantic Ocean from an elevated train.[http://i46.tinypic.com/2zs7983.jpg] Am I unfamiliar with nature or does the ocean appear elevated especially towards the horizon? [[Special:Contributions/67.243.7.245|67.243.7.245]] ([[User talk:67.243.7.245|talk]]) 23:31, 1 July 2010 (UTC)

:We do not know how the camera was oriented. The technical term for this parameter is the [[Horizontal coordinate system|elevation]] of the camera - the angle it makes with respect to the horizon - and not to be confused with its [[elevation]] above sea-level. Obviously, by angling a camera, you can make the horizon appear to be at any vertical position inside the image. In addition, there may be an [[optical illusion]] of distorted perspective if the train's window has a weird aspect ratio or orientation, or if the train was actually tilted when you photographed the scene. The train was high above the ground, (the other kind of elevation), then you may see a lot more land in the foreground than usual, contributing to the unusual perspective. [[User:Nimur|Nimur]] ([[User talk:Nimur|talk]]) 23:51, 1 July 2010 (UTC)

::Well, that's all valid, but also it looks like the foreground may be sloping downward a bit, which is a frequent cause of illusions of this sort. [[User:Looie496|Looie496]] ([[User talk:Looie496|talk]]) 01:22, 2 July 2010 (UTC)

::Actually, we ''do'' know how the camera was oriented -- if you're looking toward a horizon where the sea seems to form a sharp boundary against the sky (rather than fading into haze), a line from you to that horizon must be horizontal. (Horizon. That's why we call it "horizontal".) But as Looie says, the foreground is sloping, so this may contribute to the shoreline looking lower than you expect. --Anonymous, 04:14 UTC, July 2, 2010.

:I think the reason is rather perception (or the lack thereof) of depth (in the sense of distance). On the structured foreground one has a pretty good feel for how the distance increases as one looks from the bottom of the picture to the land-sea boundary. The unstructured sea does not offer any depth indicator (increasing haziness doesn't help on this fairly clear day) so one does not perceive any distance difference between the land-sea boundary and the horizon. That's why the sea appears like a vertical wall, the entire blue surface at the same distance. I hope I understood the question correctly. --[[User:Wrongfilter|Wrongfilter]] ([[User talk:Wrongfilter|talk]]) 15:25, 2 July 2010 (UTC)

::I've noticed the same effect while standing in an airport close to sea level&mdash;the ocean in the distance appeared somewhat distant as well as a bit elevated. Other than perspective could some kind of [[mirage]] effect explain the phenomenon? ~<font color="blue">[[User:AstroHurricane001/A|A]][[User:AstroHurricane001|H]][[User:AstroHurricane001/D|1]]</font><sup>([[User:AstroHurricane001/T|T]][[Special:Contributions/AstroHurricane001|C]][[User:AstroHurricane001/U|U]])</sup> 17:10, 2 July 2010 (UTC)

= July 2 =

== Phantom question (nothing to do with the paranormal) ==

Just a couple of days ago I've read a funny story on the Internet about an [[F-4 Phantom]] taking off from a combination civilian/military airfield. Supposedly, the Phantom pilot was in a hurry to take off but the tower told him to wait because of heavy civilian traffic; then, after some back-and-forth between the tower and the pilot, the controller said, "OK, if you can reach 14,000 feet within half the runway length, then you are cleared for takeoff; otherwise, continue to hold." Allegedly, what happened then was that the Phantom taxied into position at the approach end of the runway, engaged full afterburner, lifted off within half the runway length, and then climbed vertically to 14,000 feet, thus abiding by the tower's conditional clearance. (Here's the link to the site in question: http://www.businessballs.com/airtrafficcontrollersfunnyquotes.htm ) My question is, does the Phantom have a sufficient thrust/weight ratio to do that? I've never flown the Phantom, so I can't be sure that this is possible. Thanks in advance! [[Special:Contributions/67.170.215.166|67.170.215.166]] ([[User talk:67.170.215.166|talk]]) 01:48, 2 July 2010 (UTC)
:Based on the numbers in our article, the plane could climb vertically if it was empty, but the thrust/weight ratio was only 0.86 in a fully loaded configuration. [[User:Looie496|Looie496]] ([[User talk:Looie496|talk]]) 03:23, 2 July 2010 (UTC)
::So in other words, the plane must've had no bombs, no missiles, and only a light load of fuel on board. (Which means that the pilot (1) wasn't flying far, and (2) must've been a real hotshot to climb in afterburner with such a light fuel load.) Thanks for the info, and clear skies to you! [[Special:Contributions/67.170.215.166|67.170.215.166]] ([[User talk:67.170.215.166|talk]]) 03:49, 2 July 2010 (UTC)
:::Although not, I think, literally possible, such a feat would be much closer to attainable by an [[English Electric Lightning]]. [[Special:Contributions/87.81.230.195|87.81.230.195]] ([[User talk:87.81.230.195|talk]]) 05:56, 2 July 2010 (UTC)
::::Chatting back and forth with ATC, especially during heavy traffic would be seriously unprofessional and violate FAA regs. I hope this story is not based on fact. [[User:Googlemeister|Googlemeister]] ([[User talk:Googlemeister|talk]]) 18:16, 2 July 2010 (UTC)

== Wormholes ==

Does a wormhole exert its own gravitational pull assuming they exist? Based off diagrams I have seen of wormholes they seem to have a gravitational well similar to that of a black hole, so wouldn't they have an event horizon around each of the mouths and trap any traveller going through them inside? Or does the negative energy density required inorder to keep a wormhole open cancel out the gravitational fields. I know that this is assuming that traversable wormholes exist and they might not, but thanks for the help anyway. --[[Special:Contributions/74.67.89.61|74.67.89.61]] ([[User talk:74.67.89.61|talk]]) 01:56, 2 July 2010 (UTC)
:Wormholes do not exist. They are hypothetical conceptualizations. Even within the realm of the hypothetical, they introduce irresolvable contradictions. Wormholes are an interesting, mind-bending game for geometrically inclined mathematicians. But we have never seen them; we have never seen any evidence of them; and our best efforts to consistently explain them would require bizarre and unrealistic physics. It would serve the world a lot better if the pop-science physics books would focus on ''actual'', ''existing'' strange physics - like the solution to the generalized [[double pendulum]]. Have you ever seen one of these [http://www.youtube.com/watch?v=2JzMJNMYbRw crazy contraptions]? I can't comprehend the popular fascination with fictional physics, when there is so much unexplained in ''actual'' physics. [[User:Nimur|Nimur]] ([[User talk:Nimur|talk]]) 02:22, 2 July 2010 (UTC)

:: There is no currently known property that forbids the existence of wormholes (and lets be clear here, I am not talking about traversable wormholes, I am talking about any type of wormhole, stable or not, microscopic or macroscopic) although there are a lot of strange implications. We currently do not know enough to say whether they exist or not for certain. When black holes were found in general relativity, many scientists believe they were impossible even though there was no principle that made them impossible, and look where we are today, finding that black holes are actually common throughout the universe. Wormholes are in a similar state today of what black holes used to be in, being allowed by general relativity with nothing preventing them, but no evidence for them and strange impications. I am not saying that they exist or don't, I am simply saying we do not know enough to give a definite answer, which is why I said ASSUMING THAT THEY EXIST do they exert a gravitational pull, i am simply curious as to whether someone knows the answer to this. <span style="font-size: smaller;" class="autosigned">—Preceding [[Wikipedia:Signatures|unsigned]] comment added by [[Special:Contributions/74.67.89.61|74.67.89.61]] ([[User talk:74.67.89.61|talk]]) 13:59, 2 July 2010 (UTC)</span><!-- Template:UnsignedIP --> <!--Autosigned by SineBot-->

:The problem is, we can't just assume they exist. We have to make some assumptions about how they exist - we have to change something in the laws of physics, and there are all kinds of ways we could do that (some more plausible than others). Depending on how we change physics, we'll get different answers to your question. If we talk about a specific theory of wormholes, for example the wormhole described by Matt Visser which is mentioned in [[Wormhole#Traversable wormholes]], then the question is answerable. Unfortunately, I don't know the answer... I can't find the paper mentioned. --[[User:Tango|Tango]] ([[User talk:Tango|talk]]) 02:28, 2 July 2010 (UTC)
::Forgive me for [[WP:SOAP|soap-boxing]], but somebody needs to just come out and say it. Wormholes do not exist. Before anybody tries to contemplate the gravitational behavior of a hypothetical wormhole, they should be required to thoroughly, quantitatively, and correctly describe the behavior of [[double pendulum|this simple contraption]], ''which is only under the influence of regular, earth gravity''. Once you have mastered the mathematical techniques necessary for the generalized description of a system in ordinary geometries, you will have the foundations for the mathematical tools to play with general solutions in arbitrary geometries. Unless you have these techniques completely mastered, any description of wormholes is just gonna be a lot of handwaving and nothing more. That's why it is so frustrating to see so much meaningless and frivolous writing on the subject of wormholes, hidden behind the excuse of advanced science/magic and totally devoid of actual meaning. [[User:Nimur|Nimur]] ([[User talk:Nimur|talk]]) 02:34, 2 July 2010 (UTC)
:::In order to actually try to be encouraging, rather than discouraging, here is a list of topics you will need to learn thoroughly in order to understand geometries and topologies for wormholes:
:::*[[Algebraic geometry]], specifically the representation of space as a set of abstract and mappable coordinates
:::*[[Multivariate calculus]], especially as related to coordinate transforms and the ugly/beautiful details associated with the careful study the mathematics of [[continuity (topology)|continuity]]
:::*[[Geometric topology]]
:::*Advanced [[linear algebra]], [[tensor algebra]]
:::*Any of the various reincarnations of [[mechanics]] in terms of representations of energy - like [[Lagrangian mechanics]]
:::*And of course, the [[General relativity|relativistic theory of gravity]]
:::This list is not complete, but by the time you master those concepts, you'll already know what else you need to learn in order to study wormholes. Spend a lot of time on the mathematical foundations. These subjects are very difficult, but they are not inaccessible. They are ''requisite'' - these are the ''languages'' which are best suited to discussions of the geometry of space. "Simple english" just doesn't have the precision and unambiguity necessary to properly describe these sorts of systems. [[User:Nimur|Nimur]] ([[User talk:Nimur|talk]]) 02:45, 2 July 2010 (UTC)
::::I hold an [[MMath]] degree in which I studied (and passed) modules in all the fields you mention. I am qualified to discuss the subject of wormholes. We do not know if wormholes can exist or not, but from a mathematical point of view (which is the point of view I take, since I am a mathematician by training) the changes to our physical theories are minimal (basically, you just have to forget the [[weak energy condition]]). Just because we cannot demonstrate that the weak energy condition can be violated does not mean that we cannot investigate the consequences of such a violation. --[[User:Tango|Tango]] ([[User talk:Tango|talk]]) 03:07, 2 July 2010 (UTC)
::I have found the paper! It is [http://arxiv.org/PS_cache/arxiv/pdf/0809/0809.0907v1.pdf here]. It describes a solution in which a traveller would feel no forces on travelling through. In the words of the paper, if you send a beam of light into the wormhole "the throat of the wormhole acts as a “perfect mirror”, except that the “reflected” light is shunted into the other universe." (The paper describes wormholes between two universes, but the idea should work for wormholes within one universe too - at least, I can't see any problems with it.) Away from the throat of the wormhole, the universes are just flat [[Minkowski space]], so there would be no gravitational attraction (or repulsion, for that matter). --[[User:Tango|Tango]] ([[User talk:Tango|talk]]) 03:07, 2 July 2010 (UTC)
:::And that paper links to [http://scitation.aip.org/getabs/servlet/GetabsServlet?prog=normal&id=AJPIAS000056000005000395000001&idtype=cvips&gifs=yes&ref=no ''Wormholes in spacetime and their use for interstellar travel: A tool for teaching general relativity'' (1988)]. ([http://www.physics.louisville.edu/wkomp/teaching/spring2006/589/final/wormholes.pdf PDF available]). That paper lays out some conditions for wormholes: "It turns out that there are very simple, exact solutions of the Einstein field equations, which describe wormholes that have none of the above problems. ... The traversible wormhole solutions to Einstein's field equations are so simple that they can be used as a tool for teaching beginning relativity students how to solve the Einstein equations, how to interpret physically solutions they have obtained, and how to explore the properties of solutions." This is what the OP was really asking, I think. Thanks Tango for the reference above. [[User:Nimur|Nimur]] ([[User talk:Nimur|talk]]) 14:37, 2 July 2010 (UTC)
::::I see no reason to believe the OP was interested in the pedagogical properties of wormholes. Those properties are very important (my first rigorous exposure to wormholes was purely as a convenient example of a non-trivial solution to the Einstein equations, although that was the non-traversable Schwarschild wormhole, so not much use to the OP), but I don't think the OP cares about them. --[[User:Tango|Tango]] ([[User talk:Tango|talk]]) 15:11, 2 July 2010 (UTC)
:::::To repeat what I've said in the past about these traversable wormhole "solutions": a solution of general relativity is a solution of the Einstein field equation G<sub>μν</sub>/8πG = T<sub>μν</sub>, and any spacetime geometry whatsoever "solves" that equation if you don't constrain T<sub>μν</sub>. All you have to do is work out G<sub>μν</sub>/8πG for your geometry and then declare T<sub>μν</sub> to be equal to it by fiat. Thus to say that a traversable wormhole geometry is a "solution" of general relativity is to say nothing at all, unless you have some independent constraints on T<sub>μν</sub> (i.e., some non-gravitational laws of physics). Even the simplest physical constraints on T<sub>μν</sub> rule out all of these wormhole solutions. They are therefore not solutions in any meaningful sense. The authors of these papers do acknowledge this, but what they don't mention is how easy it is to write down wormhole and warp-drive and time-travel geometries with whatever crazy properties you want, in the absence of any physical constraints. There's nothing interesting or inventive about the particular geometries they wrote down. -- [[User:BenRG|BenRG]] ([[User talk:BenRG|talk]]) 22:51, 2 July 2010 (UTC)
::::::I would say a "solution" of the Einstein field equation is an explicit description of both the spacetime geometry and the matter/energy distribution that produces it. Just saying that the stress-momentum-energy tensor equals the appropriate value doesn't really count. --[[User:Tango|Tango]] ([[User talk:Tango|talk]]) 23:07, 2 July 2010 (UTC)
:::::::But it has been pointed out that these solutions require "exotic matter" and other un-physical assumptions, in some distribution, to produce the wormhole geometry. The paper Tango linked at least did not require exotic matter to be present in the throat of the wormhole (hence the arbitrary label of "traversable"), but exotic/unphysical matter was required elsewhere in some distribution to produce the system and its geometry. Such exotic matter does not exist. (Rather, we have never observed it, nor do we have any reason to believe it exists, but [[Flying spaghetti monster|it would be really neat if it existed]] and it is very hard to prove non-existence, so lots of people latch on to the idea). [[User:Nimur|Nimur]] ([[User talk:Nimur|talk]]) 00:05, 3 July 2010 (UTC)
::::::::Actually, the solution did involve exotic matter at the throat, just not covering the whole throat so a traveller could avoid it. Also, "traversable" usually refers to the wormhole existing long enough to travel through rather than to whether there is exotic matter in the way. While there is far from any conclusive evidence that exotic matter can exist, the [[Casimir effect]] does point in the right direction. --[[User:Tango|Tango]] ([[User talk:Tango|talk]]) 00:49, 3 July 2010 (UTC)

== MP3 player (well, really amplifier) power consumption ==

I recently got a new iPod Nano (5G). It seems to have somewhat lower volume than my old (2G) one. It's fine for music, but very many spoken word programs (such as the excellent [[Open Yale Courses]]) are recorded with a much lower volume than music. As a consequence, I often listen to them at 100% volume, and even then some are quite quite for outdoor listening. I'm wondering what that does for battery life. Does the power consumption depend only or primarily on the volume level (on the principle that the signal is maximally amplified, even if there is not much of a signal), or does it depend primarily on the amount of actual sound energy coming out of the headphones? The first seems intuitive - if I crank up the volume, I use more energy. Physically, the second is more plausible - after all, where would the energy go otherwise? Does a amplifier amplifying a flat line get hot? If the second is true, how much of an effect is it? Will my mp3 player have a significantly longer battery life playing e.g. the [[Symphony No. 6 (Beethoven)|Pastorale]] than playing [[Painkiller (Judas Priest song)|Painkiller]] (and yes, I have both on my player ;-). --[[User:Stephan Schulz|Stephan Schulz]] ([[User talk:Stephan Schulz|talk]]) 09:30, 2 July 2010 (UTC)
:As a general rule, the power consumption of a noramlly designed amplifier is approximately proportional to the volume it produces, rather than the setting of the volume control. --[[User:Phil Holmes|Phil Holmes]] ([[User talk:Phil Holmes|talk]]) 13:11, 2 July 2010 (UTC)
::Thanks! So I should always use an endless loop of [[4′33″]] for battery benchmarking ;-). --[[User:Stephan Schulz|Stephan Schulz]] ([[User talk:Stephan Schulz|talk]]) 13:19, 2 July 2010 (UTC)
:::iPod Nano uses a [[Class D amplifier]], and great pains are taken to reduce its quiescent current. (That means that "0 micro-amps" would be spent on audio power amplification if you played 4'33" through it - but of course, there are parasitics and losses, but generally very small). Realistically, you will benefit more in terms of battery life by turning the LCD screen / backlight off - this draws significantly more current than the audio system. Unfortunately the iPod nano internals are proprietary and their [http://www.ifixit.com/Teardown/iPod-nano-5th-Generation-Teardown/1157/3 amplifier and audio system is integrated into the "anonymously marked" IC packages] with Apple part-numbers only; it is impossible to get the exact specs. [[User:Nimur|Nimur]] ([[User talk:Nimur|talk]]) 15:04, 2 July 2010 (UTC)

::Just as an aside, I believe you can set individual mp3 files to play with different volumes in iTunes — File > Get Info > Options > Volume Adjustment. You could boost the predictably quiet files by 100% before putting them on the iPod. (I'm reasonably sure those settings are transferrable to the iPod, but I haven't tried it.) --[[User:Mr.98|Mr.98]] ([[User talk:Mr.98|talk]]) 19:11, 2 July 2010 (UTC)

== is there such a thing as a "semi-permanent" or reversible tattoo? ==

so I really like the idea of getting a tattoo, but who knows if later I need to sell out to the man as most people in corporations do after a while. If I ever need to have Forbes and Fortune's cocks ramming down my throat, or, who knows, maybe even Wall Street's directly, they will not be moved by tattoos on me. So, my question is whether there exists a tattoo that is "permanent" in the sense that it doesn't "just" go away, it is a real tattoo and looks like one, but semipermanent or reversible in the sense that I can get rid of it for a fee later? For example, I can imagine that a certain class of dyes have a specific "antidote", a chemical that will cause them to break up and disappear into the body, despite the fact that they do not by themselves. Is there such a thing? Or any other way to get a semipermanent or reversible tattoo? Thank you. [[Special:Contributions/92.230.232.169|92.230.232.169]] ([[User talk:92.230.232.169|talk]]) 09:41, 2 July 2010 (UTC)
:As far as I know, best-practice modern tattoos will respond well to laser treatment (laser light tuned to the dye is used to destroy the colour molecules, your immune systems takes care of the rest). There is a certain risk, though, both that the process will be incomplete, but also of allergic reactions. And it's expensive... --[[User:Stephan Schulz|Stephan Schulz]] ([[User talk:Stephan Schulz|talk]]) 09:46, 2 July 2010 (UTC)
:: so why dont they develop special dyes that respond to, say, a certain frequency of microwave radiation, or a certain "antidote" chemical, or anything else... <span style="font-size: smaller;" class="autosigned">—Preceding [[Wikipedia:Signatures|unsigned]] comment added by [[Special:Contributions/92.229.13.177|92.229.13.177]] ([[User talk:92.229.13.177|talk]]) 10:40, 2 July 2010 (UTC)</span><!-- Template:UnsignedIP --> <!--Autosigned by SineBot-->
:::Our article [[Temporary tattoo#Temporary variants of permanent tattoos]] mentions [[InfinitInk]] which is supposed to be easier to remove with laser treatment. While are I'm sure inks which are easier to remove which could be developed, I'm not convinced your first idea would be much easier or cheaper then with lasers, particularly if you don't want the rough area you were tattoed to be cooked and also don't wont your tattoo to fade a lot over time due to natural exposure to EM radiation from the many forms of transmission common nowadays. Your second idea is more promosing but developing something safe and effective is probably not easier or cheap particularly since the antidote will need to penetrate the skin [[User:Nil Einne|Nil Einne]] ([[User talk:Nil Einne|talk]]) 14:16, 2 July 2010 (UTC)

::::There's also the far-off possibility of nano-tattoos, which could be reprogrammed as you saw fit. (Kind of goes against the point of a tattoo, if you ask me. They're not just an image, they're an image of commitment!) IMO, if you are worried about not looking corporate enough, get it in an area that can be easily hidden, like the shoulder or upper arm, which can be displayed if and when you want it pretty easily, but is also easily hidden under your suit. And please watch your language on here—it's rude to swear for no purpose, and makes you sound neither edgy nor interesting. --[[User:Mr.98|Mr.98]] ([[User talk:Mr.98|talk]]) 14:33, 2 July 2010 (UTC)
::::: I acknowledge your sentiment, though I wonder if it isn't prompted by your regularly ingesting more corporate c**k than is your preference. <small>see? self-censored.</small>[[Special:Contributions/92.224.207.197|92.224.207.197]] ([[User talk:92.224.207.197|talk]]) 17:49, 2 July 2010 (UTC)

:[[Henna]]? [[Woad]]? [[Special:Contributions/92.28.247.183|92.28.247.183]] ([[User talk:92.28.247.183|talk]]) 21:17, 2 July 2010 (UTC)

== [[Copper(I) sulfate]] ==

I reduced copper(II) sulfate (blue) with ascorbic acid solution (colorless) to get a yellow solution. Is it copper(I) sulfate? It reacted with hydrogen peroxide to form a green solution again (it was green because of the additional acidity of the ascorbic acid). --[[User:Chemicalinterest|Chemicali
nterest]] ([[User talk:Chemicalinterest|talk]]) 14:16, 2 July 2010 (UTC)
:Sounds more like a [http://www.informaworld.com/smpp/content~db=all~content=a713704884 copper ascorbate complex] to me. [[User:Physchim62|Physchim62]] [[User talk:Physchim62|(talk)]] 14:43, 2 July 2010 (UTC)
::I disagree (that example is in anhydrous conditions) - in aqueous solution copper is reduced [http://www.springerlink.com/content/u7hu36m76514k151/] [http://www.sciencedirect.com/science?_ob=ArticleURL&_udi=B6TX9-4KFMK9H-2&_user=10&_coverDate=02/28/2007&_rdoc=1&_fmt=high&_orig=search&_sort=d&_docanchor=&view=c&_acct=C000050221&_version=1&_urlVersion=0&_userid=10&md5=03cb104ac196da4f180316ff25d348eb]
::Possibly you made '''nano-copper''' in the first step [http://www.ics-ir.org/jics/archive/v7/S1/article/pdf/JICS-7-S1-Article-10.pdf]. It's also possible that the some of the product at the first step was fine [[Copper(I) oxide]] (this paper [http://cat.inist.fr/?aModele=afficheN&cpsidt=18990436])
::Additionally 'Cuprous(I) hydroxide' is yellow and is obtained in alkaline conditions, though this seems very unlikely in acid conditions - see also [[Benedict's reagent]] and [[Fehling's reagent]]
::This paper [http://www.ysxbcn.com/icnfm2007/part2B/s1181.pdf] is available online, and covers a lot of stuff relating to the first reaction. <span style="font-size: smaller;" class="autosigned">—Preceding [[Wikipedia:Signatures|unsigned]] comment added by [[Special:Contributions/77.86.124.131|77.86.124.131]] ([[User talk:77.86.124.131|talk]]) 15:56, 2 July 2010 (UTC)</span><!-- Template:UnsignedIP --> <!--Autosigned by SineBot-->
::The second step (green solution) sounds like it might be a copper(II) complex - which are often green with oxy-ligands - I'm not sure what happens to the ascorbic acid through all this - but I guess it will be a complex of whatever the ascorbic acid is oxidised to.[[User:Sf5xeplus|Sf5xeplus]] ([[User talk:Sf5xeplus|talk]]) 14:55, 2 July 2010 (UTC)

Well Cu(I) isn't very stable in water. I'm going with the idea of nanocopper -- you reduced Cu(II) to Cu(I) only to have it disproportionate into Cu(II) and Cu(0). (And the Cu(II) gets reduced again into Cu(I)...) And colloidal copper is much different than bulk copper....because of different surface energies and the [[Mie scattering]] it will take on a different colour...welcome to nanochemistry! [[User:John Riemann Soong|John Riemann Soong]] ([[User talk:John Riemann Soong|talk]]) 15:54, 2 July 2010 (UTC)

Also did you add Vitamin C in excess? Or? [[User:John Riemann Soong|John Riemann Soong]] ([[User talk:John Riemann Soong|talk]]) 15:57, 2 July 2010 (UTC)

:Neither were in clear excess. When CuCl<sub>2</sub> is reacted with ascorbic acid, white CuCl is formed. I was wondering whether there is a similarity in the sulfate reaction. --[[User:Chemicalinterest|Chemicalinterest]] ([[User talk:Chemicalinterest|talk]]) 20:17, 2 July 2010 (UTC)
::I will try [[tin(II) chloride]] as a reducing agent and see if there is a difference. --[[User:Chemicalinterest|Chemicalinterest]] ([[User talk:Chemicalinterest|talk]]) 20:21, 2 July 2010 (UTC)

:: The issue is that Cu(I) sulfate is probably very soluble in water whereas CuCl precipitates in excess and is thus most of the Cu(I) is shielded from disproportionation. Try reducing Cu(II) chloride in the presence of concentrated HCl.... also reduce Cu(II) sulfate with Vitamin C in clear excess, eliminate the chance of any remaining Cu(II).
:: Btw, do you have any [[sodium borohydride]]? [[User:John Riemann Soong|John Riemann Soong]] ([[User talk:John Riemann Soong|talk]]) 21:04, 2 July 2010 (UTC)
:: (I ask because NaBH4 is a little more environmentally friendly than Tin(II) and Tin(IV)...what do you do with your waste?) [[User:John Riemann Soong|John Riemann Soong]] ([[User talk:John Riemann Soong|talk]]) 21:08, 2 July 2010 (UTC)
:::<small>Tin is ok unless you're a [[mollusc]] ... In the overall scheme of things using NaBH4 is just as damaging (production of) as a little solder+HCl, but I appreciate your point.[[Special:Contributions/77.86.124.131|77.86.124.131]] ([[User talk:77.86.124.131|talk]]) 21:25, 2 July 2010 (UTC)</small>
:::: <small>well the basic products of NaBH4 decomposition can be neutralised with acetic acid or vinegar... then dumped down the sink. You can't do that with Tin(IV). [[User:John Riemann Soong|John Riemann Soong]] ([[User talk:John Riemann Soong|talk]]) 21:27, 2 July 2010 (UTC)</small>
:::::<small>Tin in the enviroment (in the quantities here) is unlikely to be a problem .. the main problem from tin is poisonous by products from its production ie smelting</small> <span style="font-size: smaller;" class="autosigned">—Preceding [[Wikipedia:Signatures|unsigned]] comment added by [[Special:Contributions/77.86.124.131|77.86.124.131]] ([[User talk:77.86.124.131|talk]]) 21:34, 2 July 2010 (UTC)</span><!-- Template:UnsignedIP --> <!--Autosigned by SineBot-->
::: Do you have a centrifuge? If you want to do the waste management yourself, you can recover the colloidal copper by [[centrifuge]]. Sodium ions, sulfate ions, phosphate ions and chloride ions don't stress the environment that much. [[User:John Riemann Soong|John Riemann Soong]] ([[User talk:John Riemann Soong|talk]]) 21:31, 2 July 2010 (UTC)
:::: I would use tin(II) chloride in quantities in order of .2 grams; that shouldn't hurt the environment. --[[User:Chemicalinterest|Chemicalinterest]] ([[User talk:Chemicalinterest|talk]]) 00:21, 3 July 2010 (UTC)
:More clues: When reacted with an excess of [[ascorbic acid]], it forms a yellow solution. The yellow solution becomes darker as ammonia is slowly added. The color is similar to a dilute solution of [[iron(III) chloride]] in acidic conditions. Wisps of brown precipitate are formed. When much ammonia is added, it creates a very dark brown precipitate that lightens within 1 second to a yellow brown precipitate. The yellow brown precipitate reacts with small amounts of hydrochloric acid to form a colorless precipitate (CuCl), which dissolves in excess HCl to form a clear solution. The precipitate also reacts with hydrogen peroxide to form a greenish solution (Cu(H<sub>2</sub>O)<sup>2+</sup>). Hope this helps. --[[User:Chemicalinterest|Chemicalinterest]] ([[User talk:Chemicalinterest|talk]]) 00:43, 3 July 2010 (UTC)

== Concentration of NaOH ==

100 mL of 0.125 M NaOH was mixed with 64 mL of H<sub>2</sub>O. Calculate the final concentration.

Is it incorrect to use the dilution equation in the following way?

(100 mL) (0.125 M) = (64 mL) (C)

Concentration = C = 0.20 M H<sub>2</sub>O.

--[[User:478jjjz|478jjjz]] ([[User talk:478jjjz|talk]]) 17:04, 2 July 2010 (UTC)
:Yes that's wrong.
:The correct formula is :
Initial volume
-------------- x Initial concentration = Final concentration
Final volume
:The final volume is 164mL. [[Special:Contributions/77.86.124.131|77.86.124.131]] ([[User talk:77.86.124.131|talk]]) 17:16, 2 July 2010 (UTC)

So, then the final Concentration is

[(100mL)/(164 mL)] * 0.125 M = 0.0762 M

--[[User:478jjjz|478jjjz]] ([[User talk:478jjjz|talk]]) 17:44, 2 July 2010 (UTC)

== HCl and NaOH ==

<font color=green>25.0 mL of 0.5 M NaOH was reacted with 45 mL of 0.12 M HCl. Calculate the final concentration of NaOH.</font>
<br />

(0.0250 L of NaOH)*(0.5 mol NaOH) = 0.0<u>1</u>25 mol NaOH.

<br />

(0.045 L) * (0.12 M HCl) = 0.0054 mol HCl

<br />
Since NaOH and HCl react in a 1:1 ratio, then HCl is the limiting reagent. The product is 0.0054 mol NaOH.

How do I proceed further?--[[User:478jjjz|478jjjz]] ([[User talk:478jjjz|talk]]) 18:03, 2 July 2010 (UTC)
:You're starting materials indicate 0.0125 mol NaOH and 0.0054 mol HCl are availible to react. Since there is a 1:1 ratio, answer these questions:
:# If 0.0054 mol of HCl react, and the SAME NUMBER OF MOLES of NaOH react as well, then how many moles of NaOH ''actually'' react?
:# If you started with 0.0125 mol of NaOH, and you TAKE AWAY the number of moles of NaOH that reacted (#1 above) how many moles of NaOH are left over?
:# If you mixed two solutions, one of which contained 0.0250 L and the other which contained 0.045 L, what is the TOTAL VOLUME of the two solutions after mixing?
:# Now, you have some NaOH left (answer to #2) and it is in the combined volume (answer to #3). What is the final concentration, given that the concentration of the NaOH left is the LEFTOVER MOLES OF NaOH DIVIDED BY THE COMBINED LITERS OF SOLUTION.
:Every problem you solve that looks very similar to this is solved roughly the same way. --[[User:Jayron32|<font style="color:#000099">Jayron</font>]]'''''[[User talk:Jayron32|<font style="color:#009900">32</font>]]''''' 18:14, 2 July 2010 (UTC)

#0.0054 mol NaOH react
#0.0071 mol NaOH left over
#0.070 L = total volume
# (0.0071 mol NaOH)/(0.070 L) = 0.10 M NaOH.--[[User:478jjjz|478jjjz]] ([[User talk:478jjjz|talk]]) 18:29, 2 July 2010 (UTC)

:Solution concentration equals amount of stuff divided by solution volume--say that like a hundred times or so. That is the formula that always answers any dilution question, because you will always have or can figure out any two of those variables and then use that formula to solve the third. The secret to avoiding mistakes is to always write units with your numbers so you know exactly what the value means (i.e., whether it's concentration, volume, etc.). So when you first say "(0.5 mol NaOH)" BZZT, that's a kind of careless mistake that can lead to all sorts of confusion and more serious understanding mistakes later ("L * mol" is...um...L*mol not mol--that's a pretty fundamental fact). It's not just nit-picky--if you are working through towards an answer and the units are not the right type for the question ("how many moles?" and you have an answer in liters) you immediately know you made a mistake. Say you have figured out "0.0054 mol NaOH", that's an amount of stuff. You can easily figure out the total volume in which it is dissolved (the volume of a mixure of same-solvent solutions is just the sum of the volume of each part). And now you have a formula you've said like a hundred times or so that uses those two to figure out concentration. [[User:DMacks|DMacks]] ([[User talk:DMacks|talk]]) 18:49, 2 July 2010 (UTC)

== Titration ==

<font color="purple">250.0 mL of an unknown HCl solution was titrated with 14.75 mL of 0.0762 M NaOH to the phenolphthalein end point. Calculate the molarity of the HCl solution.</font>--[[User:478jjjz|478jjjz]] ([[User talk:478jjjz|talk]]) 18:41, 2 July 2010 (UTC)

The reaction is

HCl + NaOH ==> NaCl + H<sub>2</sub>O


Total volume= 250.0 mL + 14.75 mL = 264.75 mL = 0.26475 L

:Every concentration problem is the same, just different numbers. [[User:DMacks|DMacks]] ([[User talk:DMacks|talk]]) 18:49, 2 July 2010 (UTC)
We have 0.0012 mol NaOH --[[User:478jjjz|478jjjz]] ([[User talk:478jjjz|talk]]) 18:47, 2 July 2010 (UTC)

::DMacks, I beg to differ with you. I don't know how many moles of HCl I started with. This is the source of my confusion.--[[User:478jjjz|478jjjz]] ([[User talk:478jjjz|talk]]) 19:00, 2 July 2010 (UTC)
:::If you have the number of moles of NaOH and wish to find the number of moles of HCl, look at the equation. It will tell you the ratio between the two. - [[User:Jarry1250|Jarry1250]]&nbsp;<sup>[''[[Special:Contributions/Jarry1250|Humorous]]? [[User_talk:Jarry1250|Discuss]].'']</sup> 19:05, 2 July 2010 (UTC)
::::0.0012 mol HCl reacted with 0.0012 mol NaOH. I don't know how many moles of HCl I started with; how do I find the moles of leftover HCl?
--[[User:478jjjz|478jjjz]] ([[User talk:478jjjz|talk]]) 19:11, 2 July 2010 (UTC)
:::::What does the "phenolphthalein end point" mean? [[User:DMacks|DMacks]] ([[User talk:DMacks|talk]]) 19:26, 2 July 2010 (UTC)
::::::It means to the point where all the acid (HCl) and base (NaOH) have completely reacted.--[[User:478jjjz|478jjjz]] ([[User talk:478jjjz|talk]]) 19:30, 2 July 2010 (UTC)
:::::::With none left over? [[User:DMacks|DMacks]] ([[User talk:DMacks|talk]]) 19:32, 2 July 2010 (UTC)
::::::::Since I don't know the starting moles of HCl, I don't know how much of it is left over. I want to know if this problem from one of my chemistry quizzes is even solvable.--[[User:478jjjz|478jjjz]] ([[User talk:478jjjz|talk]]) 19:34, 2 July 2010 (UTC)
:::::::::Maybe the context of my question wasn't clear. Again, for the definition of the phenolphthalein endpoint of a titration, you say the acid and base "have completely reacted". How many moles of base do you need in order that X moles of acid gets "all reacted"? Does the definition here mean you have added just an arbitrary small amount, exactly as much of one as the other, or a huge excess amount? How much un-neutralized acid (or base) is present according to the definition of your endpoint? [[User:DMacks|DMacks]] ([[User talk:DMacks|talk]]) 19:42, 2 July 2010 (UTC)

*0.0012 mol NaOH has reacted with X amount of HCl. I don't know the initial concentration of HCl which I can multiply by the volume to get the # of moles of HCl. If I have, let's say, 0.000005 moles of HCl at the start of the reaction, then HCl would be the limiting reagent & none would be left over.--[[User:478jjjz|478jjjz]] ([[User talk:478jjjz|talk]]) 19:51, 2 July 2010 (UTC)
:If you have 0.000005 moles of HCl at the start of the reaction, how many moles of NaOH would you add to get that amount of HCl all reacted? [[User:DMacks|DMacks]] ([[User talk:DMacks|talk]]) 19:53, 2 July 2010 (UTC)

They would both be in a beaker. 0.0012 mol NaOH reacts with with all HCl and (0.0012 -0.000005) mol NaOH is left over in the basic solution in the beaker due to the excess NaOH.--[[User:478jjjz|478jjjz]] ([[User talk:478jjjz|talk]]) 19:56, 2 July 2010 (UTC)
:There's your problem: the very definition of your endpoint is when the solution reaches a very specific pH (the color changes), which tells you exactly how much H+ is present at that point. The casual wording of the reaction description might be throwing you: if you have >0 of compound X present, it is not "all reacted", it's excess unreacted. All reacted means all ''of it'' is reacted, not just "as much as can". If I have 10 H+ and add 5 OH-, my OH- is all reacted, but my H+ is ''not'' all reacted. The first drop of base you add to the acid completely reacts, but you (hopefully) recognize that this is not the endpoint of the titration (would be silly because every titration would be "1 drop"). You keep adding base until (for example) neutrality (until the H+ is "all reacted"). [[User:DMacks|DMacks]] ([[User talk:DMacks|talk]]) 20:05, 2 July 2010 (UTC)
*The <font color="purple">purple words above</font> have been copied verbatim. I have concluded that this problem isn't solvable. I have to speak to the instructor and let him know that it is preposterous to put unsolvable problems on quizzes to make me get a low grade. (If anyone can somehow solve this problem, then please do!)--[[User:478jjjz|478jjjz]] ([[User talk:478jjjz|talk]]) 20:11, 2 July 2010 (UTC)
::The problem is perfectly solvable. In fact, it is a routine calculation in analytical chemistry! Just just need to figure out the amount of HCl in the initial solution, then divide by the volume of the initial solution to find the concentration. [[User:Physchim62|Physchim62]] [[User talk:Physchim62|(talk)]] 20:16, 2 July 2010 (UTC)
::(ec) To be honest, if I were the teacher, I would tell you to go back and relearn what "endpoint" means in the context of a titration experiment. Again, per the technical definition of phenolphthalien endpoint, you know ''exactly'' the concentration of unreacted acid in the solution at that point (i.e., excess, compared to the amount of base you added). [[User:DMacks|DMacks]] ([[User talk:DMacks|talk]]) 20:17, 2 July 2010 (UTC)
:This problem was in a lab quiz. The #s in the above purple problem have nothing to do with the experiment that I had performed during the prior class. As it stands, I re-affirm that the purple problem is unsolvable.--[[User:478jjjz|478jjjz]] ([[User talk:478jjjz|talk]]) 20:22, 2 July 2010 (UTC)
::The first step in pretty much any titration problem is the calculate the amount of reagent you have added from the burette. Now here, you've added 14.75&nbsp;mL of a 0.0762&nbsp;M solution: amount is concentration times volume, so you have added ''n''(NaOH)&nbsp;= 0.01475*0.0762&nbsp;= 0.001124&nbsp;mol. You know the equation, so you know that that hydroxide has reacted with 0.001124&nbsp;mol of HCl. So how much HCl is left over? Take a quick look at our article on [[phenolphthalein]], and you will see that it changes colour at pH&nbsp;8.2… so when the phenolphthalein changes colour, there's ''no'' hydrochloric acid left! So your initial solution of HCl contained 0.001124&nbsp;mol HCl in 250&nbsp;mL, in other words it was a 0.001124/0.25&nbsp;= 0.00450&nbsp;M solution. [[User:Physchim62|Physchim62]] [[User talk:Physchim62|(talk)]] 20:41, 2 July 2010 (UTC)

::Let's start again
::14.75 mL of 0.0762 M NaOH was used - that's 14.75 x 0.0762 = 1.12395mmol of NaOH (A)
::It was titrated to an end point with an acid-base indicator in the reaction NaOH + HCl >>> NaCl + H<sub>2</sub>O , that's as 1:1 reaction so 1.12395mmol of HCl must have been used (B)
::Molar concentration is related to number of moles and volume by the equation:
Number of moles
Concentration = ------------------ (C)
Volume
::You've been asked to find the molality (concentration) of the HCl solution, so that's 1.12395mmol ÷ 250ml (D)
::Be careful with units, since both the quanities in D are 'milli' in this case the answer is is mol/litre , which is the same of molality, no further work needed.
::If you didn't get any part please ask about it. I've labelled each step A,B,C etc .[[Special:Contributions/77.86.124.131|77.86.124.131]] ([[User talk:77.86.124.131|talk]]) 20:33, 2 July 2010 (UTC)
:::Your methodology differs from the section above this problem. Therein, the leftover moles that hadn't reacted were used to find the concentration. On the other hand, in this problem, the number of moles that ''have reacted'' have been used to find the concentration.--[[User:478jjjz|478jjjz]] ([[User talk:478jjjz|talk]]) 20:38, 2 July 2010 (UTC)

*The problem should have said " Calculate the <u>initial</u> molarity of the HCl solution." User:Physchim62's explanation has made me believe that 0.001124/0.25&nbsp;= 0.00450&nbsp;M HCl is indeed correct for the <u>initial</u> concentration.
::The point is that you start out with 250&nbsp;mL '''''of''''' an HCl solution: the implication is that it is a sample of a larger volume of solution that you have sitting in a bottle somewhere. [[User:Physchim62|Physchim62]] [[User talk:Physchim62|(talk)]] 20:58, 2 July 2010 (UTC)

I have finally reconciles the approach of this problem and the one above it. In both cases, we have used the unreacted substance to find concentration. In this one, we found the unreacted amount prior to the reaction to get the concentration. In the previous problem, we found the unreacted amount after the reaction to get the concentration.--[[User:478jjjz|478jjjz]] ([[User talk:478jjjz|talk]]) 20:50, 2 July 2010 (UTC)
:The other point with a titration is that you use an indicator (here, phenolphthalein) so that you know exactly when the last of the reactant has been used up. In effect, you already know the final concentrations – they're both zero. [[User:Physchim62|Physchim62]] [[User talk:Physchim62|(talk)]] 20:58, 2 July 2010 (UTC)
::I thank you and others who helped me with this problem.--[[User:478jjjz|478jjjz]] ([[User talk:478jjjz|talk]]) 21:12, 2 July 2010 (UTC)

== Temperature of the ocean ==

What IS the temperature of the water 1000s of meters down in the worlds oceans? <span style="font-size: smaller;" class="autosigned">—Preceding [[Wikipedia:Signatures|unsigned]] comment added by [[Special:Contributions/86.182.34.28|86.182.34.28]] ([[User talk:86.182.34.28|talk]]) 19:20, 2 July 2010 (UTC)</span><!-- Template:UnsignedIP --> <!--Autosigned by SineBot-->
:Pretty close to freezing. 35-40 deg F. Volcanic vents the exception. [[User:Googlemeister|Googlemeister]] ([[User talk:Googlemeister|talk]]) 19:25, 2 July 2010 (UTC)
:[[Abyssal plain#Terrain features]] has lots of interesting information on this subject. --[[User:Tango|Tango]] ([[User talk:Tango|talk]]) 20:05, 2 July 2010 (UTC)

::<s>[[Density_of_water#Density_of_water_and_ice|Four degrees celsius]], because that is the temperature at which water is most dense.</s> <small>See below, and [[Density_of_water#Density_of_water_and_ice|the article]] - sorry, my error.</small> [[Thermocline]] has good information for you. Because of [[Salt fingering|salinity-based]] and thermal-based [[upwelling]], there may be local variations, but in the very deep ocean, there is a reasonably static temperature profile (even if there is mass transfer of the water). [[User:Nimur|Nimur]] ([[User talk:Nimur|talk]]) 20:30, 2 July 2010 (UTC)

:::As I learned after giving the same answer here some time ago, the four degree value holds true for fresh water but not for salt water. The link you pointed to explains that the freezing point of seawater is about minus two Celsius, and the density increases right down to the freezing point. [[User:Looie496|Looie496]] ([[User talk:Looie496|talk]]) 00:20, 3 July 2010 (UTC)

== Audio amplifier power output ==

Why is the output of a domestic [[audio amplifier]] not inversely proportional to the [[loudspeaker]] [[impedance]]. I was taught that P=V^2/R but amplifiers dont seem to obey this law when you look at their specifications. Why not?--[[User:Bellwelder|Bellwelder]] ([[User talk:Bellwelder|talk]]) 20:12, 2 July 2010 (UTC)
:Mostly because the amplifier itself has impedance as well - this makes your equation:

P=V<sup>2</sup>/(R<sub>speaker</sub>+R<sub>amplifier</sub>)

:The other additional factor in old fashioned amplifiers is the power supply capacity of the power supply ie the transformer - there's a limit to how much power a transformer can supply related to its inductance, but I don't know the equation.
:[[Special:Contributions/77.86.124.131|77.86.124.131]] ([[User talk:77.86.124.131|talk]]) 20:41, 2 July 2010 (UTC)
::You will get maximum power transfer from the amplifier to the loadspeaker when their [[Impedence matching|impedences are matched]], not when the speaker impedence is minimum: this is known as the [[maximum power theorem]]. [[User:Physchim62|Physchim62]] [[User talk:Physchim62|(talk)]] 21:47, 2 July 2010 (UTC)

== natural skin oil ==

What oils most closely mimic natural skin oil: for example, olive oil, peanut oil, canola oil, mineral oil, vaseline, coconut oil, palm oil, etc.? <small><span class="autosigned">—Preceding [[Wikipedia:Signatures|unsigned]] comment added by [[User:Alexsmith44|Alexsmith44]] ([[User talk:Alexsmith44|talk]] • [[Special:Contributions/Alexsmith44|contribs]]) 20:34, 2 July 2010 (UTC)</span></small><!-- Template:Unsigned --> <!--Autosigned by SineBot-->
:Human skin oil is called [[Sebum]] , there's a simplfied analysis at [[Sebaceous_gland#Composition]] - of those components most are not found in vegetable oils, so a comparison between them isn't that viable. (Mineral oil and [[Vaseline]] are pure petroleum product and not like skin oil at all).[[Special:Contributions/77.86.124.131|77.86.124.131]] ([[User talk:77.86.124.131|talk]]) 20:47, 2 July 2010 (UTC)
:Very roughly it's similar to 40% vegetable oil + 20% beeswax + 10% vaseline + 10% soap (though it's not salty like soap) .. hope that helps.[[Special:Contributions/77.86.124.131|77.86.124.131]] ([[User talk:77.86.124.131|talk]]) 20:59, 2 July 2010 (UTC)

== Name these two plants? ==

In a park in southern England. http://img186.imagevenue.com/img.php?image=04455_DSCF0003_122_354lo.JPG The one on the right foreground had grape-like berries earlier in the year, on the now dried fish-bone-shaped parts. Thanks [[Special:Contributions/92.28.247.183|92.28.247.183]] ([[User talk:92.28.247.183|talk]]) 21:04, 2 July 2010 (UTC)
:<small>Just to give the same warning as before, the image link gives a popup to a NSFW site</small>[[Special:Contributions/77.86.124.131|77.86.124.131]] ([[User talk:77.86.124.131|talk]]) 21:18, 2 July 2010 (UTC)
: Back left is I think some kind of fig ([[ficus]]), but there's an awful lot of them to choose from. Perhaps the one in front is [[mahonia]]? [[Special:Contributions/213.122.27.137|213.122.27.137]] ([[User talk:213.122.27.137|talk]]) 22:42, 2 July 2010 (UTC)

== Annual [[architectural plants]] ==

I bought some annual seeds to sow in my garden in southern England and was dispointed to discover that they were only 2 or 3 inches high when flowering. Are there any big garden plants that will grow as an annual, particularly in semi-shade? 'Architectural plant' means a large plant at least three or four feet high. Thanks. [[Special:Contributions/92.28.247.183|92.28.247.183]] ([[User talk:92.28.247.183|talk]]) 21:12, 2 July 2010 (UTC)
:[[Rhododendron]]? I think you mean a small [[shrub]] of which there are many - was there anything else you wanted in the plant that could help narrow the search?[[Special:Contributions/77.86.124.131|77.86.124.131]] ([[User talk:77.86.124.131|talk]]) 21:20, 2 July 2010 (UTC)

:Some varieties of Hollyhock (''Althaea'' aka ''[[Alcea]]'') would certainly qualify: they are often used as the 'backdrop' plant (in front of which shorter flowers are placed) in the traditional English [[Cottage garden]]. Foxgloves (''[[Digitalis]]'') would fit the size criterion, but are perennial or biennial.
:Leafing quickly through [[D. G. Hessayon|Dr D. G. Hessayon]]'s ''The Bedding Plant Expert'', other candidate annuals with some varieties in the 3-4 feet range include: Love-lies-bleeding (''[[Amaranthus]]''); African Daisy (''[[Arctotis]]''); Spider Flower (''[[Cleome]]''); [[Datura]]; Sunflower (''[[Helianthus]]''); Burning Bush (''[[Kochia]]''); Larkspur (''[[Delphinium]]''); Annual Mallow (''[[Lavatera]]''); Nasturtium (''[[Tropaeolum]])''; Sweet Pea (''[[Lathyrus]]''); Ornamental Maize (''[[Zea]]''). [[Special:Contributions/87.81.230.195|87.81.230.195]] ([[User talk:87.81.230.195|talk]]) 22:41, 2 July 2010 (UTC)

= July 3 =

== Enthalpy-entropy chart ==

[[File:HS-Wasserdampf engl.png|right|300px]]
Hi. On this [[enthalpy-entropy chart]], can someone please tell me what 'x' represents (ie x = 10%, x = 20%, x = 30%...).

The person who actually created the graph seems to have 'retired' from Wikipedia, and another user asked for a caption to be added to it.

Thanks in anticipation, <small><span style="border:1px solid;background:#00008B">[[User:Chzz|'''<span style="background:#00008B;color:white">&nbsp;Chzz&nbsp;</span>''']][[User talk:Chzz|<span style="color:#00008B;background-color:yellow;">&nbsp;►&nbsp;</span>]]</span></small> 01:32, 3 July 2010 (UTC)
:It's either [[Percent humidity]], i.e. the ratio of actual vapor pressure to vapor pressure at the [[dew point]], or some sort of meaure of proximity to the [[critical point]]. --[[User:Jayron32|<font style="color:#000099">Jayron</font>]]'''''[[User talk:Jayron32|<font style="color:#009900">32</font>]]''''' 01:38, 3 July 2010 (UTC)

== Medroxyprogesterone versus medroxyprogesterone acetate ==

Hola,

I'm trying to figure out the difference between [[medroxyprogesterone]] versus [[medroxyprogesterone acetate]]. Anyone know? Anyone have any sources that can be used to distinguish the two? [[User:WLU|WLU]] <small>[[User talk:WLU|(t)]] [[Special:Contributions/WLU|(c)]] Wikipedia's rules:</small>[[WP:SIMPLE|<sup><span style='color:#FFA500'>simple</span></sup>]]/[[WP:POL|<sub><span style='color:#008080'>complex</span></sub>]] 01:57, 3 July 2010 (UTC)
:Other than the obvious difference in chemical structure, what are you looking for? It appears that the 17-acetate is the only one used medically - both of the references in the MP article actually relate to MPA. [http://www.merck.com/mmpe/lexicomp/medroxyprogesterone.html The Merck Manual entry] indicates that the former is "supplied as the acetate". The [http://www.nlm.nih.gov/medlineplus/druginfo/meds/a682470.html article in MedlinePlus] uses the terms interchangeably. Many references discuss plasma levels of MPA, so my initial impression that the acetate is readily hydrolyzed is probably false. -- [[User:Scray|Scray]] ([[User talk:Scray|talk]]) 02:59, 3 July 2010 (UTC)

Latest revision as of 01:12, 8 June 2024

Welcome to the science section
of the Wikipedia reference desk.
Select a section:
Want a faster answer?

Main page: Help searching Wikipedia

   

How can I get my question answered?

  • Select the section of the desk that best fits the general topic of your question (see the navigation column to the right).
  • Post your question to only one section, providing a short header that gives the topic of your question.
  • Type '~~~~' (that is, four tilde characters) at the end – this signs and dates your contribution so we know who wrote what and when.
  • Don't post personal contact information – it will be removed. Any answers will be provided here.
  • Please be as specific as possible, and include all relevant context – the usefulness of answers may depend on the context.
  • Note:
    • We don't answer (and may remove) questions that require medical diagnosis or legal advice.
    • We don't answer requests for opinions, predictions or debate.
    • We don't do your homework for you, though we'll help you past the stuck point.
    • We don't conduct original research or provide a free source of ideas, but we'll help you find information you need.



How do I answer a question?

Main page: Wikipedia:Reference desk/Guidelines

  • The best answers address the question directly, and back up facts with wikilinks and links to sources. Do not edit others' comments and do not give any medical or legal advice.
See also:

May 26[edit]

Why is there no monosmium octafluoride or moniridium nonafluoride or PtF10?[edit]

There's crazy ways to make new forms like electrocuting extremely hot plasmas and forcing noble gases to bond or crushing between microdiamonds yet no one's figured out how to make OsF8 or IrF9 but OsO4 could be made with Paleolithic tech (poisonous though). Sagittarian Milky Way (talk) 16:58, 26 May 2024 (UTC)[reply]

The reason will be that there is not enough space to pack that many fluorine atoms around the metal, at a close enough distance to have strong enough bonds. The bond in the hexafluoride would be much stronger, and to add more fluorine, it would have to push out those other fluorine atoms to squeeze new ones in. The energy gain by this would have to exceed that of breaking an F-F bond in F2. Perhaps atomic fluorine could assist in a rare gas matrix, but in warmer conditions F2 would be produced. Perhaps a diamond anvil press could do something, but I have not heard of its use for these. Graeme Bartlett (talk) 21:35, 26 May 2024 (UTC)[reply]
Under high pressure it should be possible. OsF8 and IrF8 are predicted to be stabilised by 300 GPa, as the 6p orbitals on the metal lower and we have ligand-to-metal charge transfer from F 2p to M 6p. Also TcF7 and CdF3 should stabilise (the former interesting because ReF7 exists at normal conditions and this is one case where Tc and Re differ; the latter interesting for breaking the oxidation state barrier for stable group 12 elements). Double sharp (talk) 05:43, 27 May 2024 (UTC)[reply]
Re Paleolithic OsO4: an interesting thought, but even the easy metals like Au seem to have been only recognised later per Element discovery. If we want to push it as early as possible, then I think you'd need an early discovery of Pt (accomplished by pre-Columbian South Americans), together with enough alchemical knowledge to get aqua regia to separate out an Os-Ir fraction. Double sharp (talk) 06:44, 27 May 2024 (UTC)[reply]
Well from osmium, not completely de novo. Sagittarian Milky Way (talk) 14:27, 28 May 2024 (UTC)[reply]

@Graeme Bartlett: That said, is there a similarly simple explanation for why Rh, Ir, and Pt can reach the VI oxidation state only in fluorides and not oxides? And why is PdVI so elusive despite this oxidation state being well-established for all other PGMs? Double sharp (talk) 07:04, 27 May 2024 (UTC)[reply]

I will add there is no PdV either. Higher oxidation states get less stable as you move right in the periodic table from W to Pt or Mo to Pd. Graeme Bartlett (talk) 23:36, 28 May 2024 (UTC)[reply]
Thanks. Reading around, maybe there is no simple answer exactly for why the threshold of impossibility is precisely where it is, so I shall have to be satisfied by this. :) Double sharp (talk) 09:48, 29 May 2024 (UTC)[reply]
Google Bard tells me it is about electronegativity of fluorine and polarizability of oxygen. Not to be trusted though. High oxidation compounds all seem to be isolated molecules, and not network solids, which would be topologically possible with oxygen. So it seems that more single bonds is more stable than a few double bonds, but up to the limit of about 6 due to not being able to pack in the atoms around the metal. Graeme Bartlett (talk) 11:46, 29 May 2024 (UTC)[reply]
This certainly makes sense. And in the 3d series, as expected the limiting coordination number seems even less in some cases: we have CrO3 but only CrF5, and Mn2O7 but only MnF4. Double sharp (talk) 15:24, 29 May 2024 (UTC)[reply]
For discussion of Pd(V) and Pd(VI) inorganic and synthesis of a Pd(V) cluster, see doi:10.1073/pnas.0700450104. DMacks (talk) 17:19, 29 May 2024 (UTC)[reply]
Graeme Bartlett, we don't ever use the mono- prefix with the first word of a binary compound's name. CO2 is carbon dioxide, not monocarbon dioxide. Georgia guy (talk) 10:15, 29 May 2024 (UTC)[reply]
But not applicable to Sagittarian Milky Way who likes "mono"; and there is monosodium phosphate or monosodium glutamate for non-binary compounds. Graeme Bartlett (talk) 11:46, 29 May 2024 (UTC)[reply]

For actual Ir(IX), there is IrO4+, though this is not "bottlable" stuff. Possibly Pt(X) will be possible too in PtO42+, but that one has not yet been experimentally confirmed. Double sharp (talk) 04:19, 31 May 2024 (UTC)[reply]

Two universes in same space[edit]

My heading may be mediocre considering Nasa's simplified definition "The universe is everything. It includes all of space, and all the matter and energy that space contains". So

  • can there exist two (or more) 'universes' in the same space-time domain?
    • If yes, can we say the space compromises the universe and not vice-versa?
    • If no, does that mean all of space is part of 'universe', thus even if mass in form of galaxy, planets etc. is limited in mass and volume, still all of that infinite vacuum that surrounds it is part of 'our universe'?
  • If there is something far away from our 'observable universe' which did not come out of big bang (considering big bang happened for sure), would that still be considered part of our universe?

You may answer any part you like. Thanks, ExclusiveEditor Notify Me! 20:04, 26 May 2024 (UTC)[reply]

  • Your "if no" option means that empty space is part of the Universe. Far away stuff would be part of the universe. Graeme Bartlett (talk) 21:39, 26 May 2024 (UTC)[reply]
  • You talk of an infinite vacuum that surrounds the universe. That would be absolutely empty space. But everything that exists, even absolutely empty space, is by definition part of the universe. So, either there's some sort of boundary, beyond which there is nothing, not even absolutely empty space - which means there is no "beyond"; or, there is no boundary and the universe is infinitely limitless. Take your pick. -- Jack of Oz [pleasantries] 22:39, 26 May 2024 (UTC)[reply]
    Or it is a finite closed manifold. —Tamfang (talk) 21:25, 30 May 2024 (UTC)[reply]
The universe, by definition, is everything. Or did you say that already? ←Baseball Bugs What's up, Doc? carrots→ 00:28, 27 May 2024 (UTC)[reply]
The universe has (potentially) been demoted. See Multiverse (not a long song). Clarityfiend (talk) 06:49, 27 May 2024 (UTC)[reply]
Not really "potentially" at this point, just "hypothetically". ←Baseball Bugs What's up, Doc? carrots→ 08:01, 27 May 2024 (UTC)[reply]
Fairly sure that "absolutely empty space" is, very much by definition, not a "thing" and hence not part of "everything" 2A01:E0A:CBA:BC60:CF2:682A:5D96:ED9D (talk) 07:50, 28 May 2024 (UTC)[reply]
Mathematicians might disagree with you. Is the empty set, for example, a "thing"? I'd say so. I'll concede that there is some controversy on the point. --Trovatore (talk) 07:58, 28 May 2024 (UTC)[reply]
Not sure that's right. What about scalar fields? Sean.hoyland (talk) 08:02, 28 May 2024 (UTC)[reply]

Q. Imagine big bang really happened, so by definition that the entire space (our space time domain to be exact) is part of 'our universe', and so that will mean something far away which did not come out of our big bang is still part of 'our universe'; thus meaning that big bang did not necessarily give birth to 'our universe' but just a part of 'our universe'. Also that means 'expansion of universe' would just mean expansion of 'our part of our universe' which is near us and came thru our big bang (and not all universe)? ExclusiveEditor Notify Me! 07:45, 27 May 2024 (UTC)[reply]

Where is the question in that? Our definition of Universe includes the far away stuff. The definition does not depend on how it was created. You may also be interested in the idea of an inter-penetrating alternate universe that uses the same space. For example a Dark matter world, or perhaps a Counter-Earth, six months out of phase. I can imagine it as a clathrate. Graeme Bartlett (talk) 07:53, 27 May 2024 (UTC)[reply]
The question is more of terminologies and what words means, than physics itself. But clarifying this helps as to know if the definitions are standardized or still vague. ExclusiveEditor Notify Me! 07:50, 27 May 2024 (UTC)[reply]
What evidence, if any, supports your premise? ←Baseball Bugs What's up, Doc? carrots→ 07:52, 27 May 2024 (UTC)[reply]
Like many words, people do not always use the term universe to mean the same thing. Usually you can understand from the context which sense is meant. When people say "the universe", they usually mean the totality of space and everything in it that we know to exist or can reasonably expect to exist based on what we know. So the spatial extension of the universe is the totality of space; there is no room for another universe.
Mathematically, you can have several spaces that are in no way connected to each other, so one can imagine there are other universes that occupy other spaces that we have no way of knowing about: we cannot reach them, and voyagers or information from these other universes cannot reach us. Therefore the hypothesis that other universes exist is unfalsifiable and falls outside the realm of science. It is a plot device in science fiction, but then there are portals connecting the universes, so from a definitional point of view one might say they are all part of a universe that is somehow compartmentalized.
Another sense of the term universe, given by the Merriam–Webster dictionary as sense 1c(3), is: "an aggregate of stars comparable to the Milky Way galaxy".[1] That is a complicated way of saying "a galaxy". Used in this sense, there are maybe a trillion such universes in the observable universe. They share the space and may collide.  --Lambiam 09:46, 27 May 2024 (UTC)[reply]
That sounds like a throwback to when the Milky Way was thought to be "everything". I'm also reminded of a time when I had a short discussion with a fundamentalist Christian who claimed that God was "outside of the universe." I said that the universe, by definition, is everything. So if God exists, then He must be part of the universe. The guy conceded. ←Baseball Bugs What's up, Doc? carrots→ 21:11, 27 May 2024 (UTC)[reply]
I agree with that guy. If you accept the notion of an omnipotent God, then He must be able to create stuff that is not a part of Himself. Just because we "define" the Universe to conform with our limited conceptual ability does not alter that. -- Jack of Oz [pleasantries] 21:53, 27 May 2024 (UTC)[reply]
God being omnipotent, It is also able to create Itself.  --Lambiam 09:17, 28 May 2024 (UTC)[reply]

Perhaps he conceded too easily. It's a fairly standard theological view that God exists beyond space and time, and while you personally may use "the universe" to mean "everything that exists", I think it's probably more usual to imagine the universe as being restricted to that which exists in spacetime.
That said, and this is of course a digression, the view you report does not strike me as exactly orthodox Christian theology, which usually emphasizes God's dual role as both immanent and transcendent. If you view God as transcendent only and not at all as immanent, then it seems like you might be a deist or something. --Trovatore (talk) 22:01, 27 May 2024 (UTC) [reply]
You seem to be picturing the 'Big Bang' as an explosion of matter/energy into a pre-existing infinite space, which still surrounds the (expanding) 'universe' of matter and energy that we are in (so that there could be other 'Big Bang bubbles' somewhere else in that infinite space): this is a fundamental misunderstanding of current astronomical theories. According to mainstream astronomy, The 'Big Bang' was a (rapid) expansion of space and time themselves, more properly understood as the single entity Spacetime, and of the energy (some of which became matter) within them/it. There is, geometrically, nothing 'outside' Spacetime because there is no outside. This is, of course, hard to visualise and grasp. {The poster formerly known as 87.81.230.195} 94.2.67.173 (talk) 21:15, 27 May 2024 (UTC)[reply]
As said by Hank Green (and expanded by Elle Cordova) "there was no up, there was no down, there was no side to side". Search youtube for "big bang poem". --User:Khajidha (talk) (contributions) 11:46, 29 May 2024 (UTC)[reply]


May 29[edit]

Elderly digestion[edit]

Older people have more distended intestines due to loss of muscle tone. It also means food is pushed along more slowly along the digestive tract. Does this mean that per ounce of ingested food, elderly people will extract more nutrients?

Of course, if this causes them to eat less, it may not mean more calories absorbed. Imagine Reason (talk) 09:24, 29 May 2024 (UTC)[reply]

I don't know if any of the premises of your argument are correct. But consider this; for about half a billion years our ancestors have had a digestive system. Extracting every usable bit of nutrients from our food has been under strong selection the whole time, because starvation kills and food is limited. Why then would a malfunctioning elderly system do better? Abductive (reasoning) 18:54, 29 May 2024 (UTC)[reply]
There is an evolutionary pressure to extract every usable bit of nutrients from our food. There's also an evolutionary pressure to keep the power-to-weight ratio of the digestive system high. Wasting some nutrients to keep the digestive system light may be benificial. I'm not suggesting I disagree with your conclusion. PiusImpavidus (talk) 19:54, 29 May 2024 (UTC)[reply]
The human digestive system is far from perfect in extracting calories. Imagine Reason (talk) 21:52, 31 May 2024 (UTC)[reply]

Elvish astronomy[edit]

Considering Legolas' canonical feats, what would be his naked-eye limiting magnitude, assuming seeing conditions in which the average human would get 6.0?

(The main reason for this question is that in Morgoth's Ring an Elvish name for Neptune is given. Though I suppose they might've been using palantíri as telescopes.) Double sharp (talk) 09:36, 29 May 2024 (UTC)[reply]

Perhaps elves have eagle eyes; Tolkein is silent on the issue. Alansplodge (talk) 11:15, 29 May 2024 (UTC)[reply]
He does kind of imply at least that the naked-eye limiting magnitude is greater for Elves than for Men. In The Nature of Middle-Earth, "Dark and Light" it is written: The Quendian imagination of the shape of Arda and of the visible Heaven (Menel) above it, was due to the acute minds of a people endowed with sight far keener than the human norm. Though this is for the Round World version, in which the Sun and Moon already exist from the beginning. As for quantitative figures, I guess we're stuck with the data point of Legolas counting 105 horsemen from a distance of 24 km.
Actually, it occurs to me that (perhaps more interestingly than limiting magnitudes), Elves really ought to be able to resolve the Galilean moons of Jupiter. This provides an alternate solution to the longitude problem, at least if you take the Round World versions. :) Double sharp (talk) 15:20, 29 May 2024 (UTC)[reply]
The Galilean moons would be a cinch for an elf. I had a (human) friend who could do this (he was tested on it several times). Not being particularly interested in astronomy, he only found out in his adult years that this was not usual for most people. {The poster formerly known as 87.81.230.195} 94.2.67.173 (talk) 19:42, 29 May 2024 (UTC)[reply]
Hmm, probably also the crescent of Venus should be resolvable for them. This perhaps has implications on the shapes of the Silmarils. (Although in the Round World Version, Venus already exists beforehand, and its identification with Eärendil is said to be mythologising. From the same essay I quoted: Certain stars (no doubt those we call planets) and among them especially Venus, which they called Elmō (and later mythologically Eärendil), they early observed were “wayward” and altered their places with regard to the “farstars” (fixed stars). These they called companions of the Sun and thought them quite small heavenly bodies – derived from the Sun.) Double sharp (talk) 02:31, 30 May 2024 (UTC)[reply]
The limiting magnitude can be increased with a sharper view. The sharper the view, the smaller the area of the detector (retina) on which the light falls. The background light is fixed per unit of surface area of the detector, so with the signal on a smaller area, less background competes in this area, increasing signal-to-noise. A sharper view can also help to take a faint object out of the glare of a nearby bright object; relevant to see the Galilean moons.
The other way to increase the limiting magnitude is by increasing sensitivity. No matter how sharp your eyes are, you need a couple of photons before you can see anything. The more photons you detect, the lower the relative Poisson noise. The sensitivity can be increased by (A) a better detector, detecting a larger fraction of the incoming photons; (B) larger aperture, i.e. a bigger pupil; (C) increasing integration time. Many nocturnal animals (and elves may be somewhat nocturnal) have better, more sensitive detectors, although at a price. The tapetum lucidum found in many animals reduces resolution somewhat; some species sacrificed colour vision for better low-light vision. Bigger eyes help to see better, but although Tolkien often writes that Elven eyes are keen and fair, he never writes (AFAIK) that they're big. Maybe elves can at will increase the integration time of their eyes. For humans it's fixed at several centiseconds, but if elves can boost it to a full second (they would largely loose the ability to detect motion), seeing Neptune shouldn't be too hard. Still takes a lot of dedication and patience to find out which of those tens of thousands of faint stars slowly moves, but patience is something you should have if you live forever. Ents might disagree. PiusImpavidus (talk) 20:55, 29 May 2024 (UTC)[reply]
Why would Ents disagree? They have loads of patience (though it is unclear if they are immortal or just extremely long-lived). Clarityfiend (talk) 22:23, 29 May 2024 (UTC)[reply]
Ents might disagree with the statement that elves have patience. PiusImpavidus (talk) 11:07, 30 May 2024 (UTC)[reply]
Thanks for the details! I guess I'm personally more inclined now towards explaining the Neptune thing by an Elvish invention of the telescope.
Or maybe the Valar told them where to look, noting that Neptune's magnitude is actually brighter than the most extreme reports of naked-eye viewing of stars. It would be a lot easier to find Neptune if you already know where it is, than to find which of those myriad faint stars is slowly moving. In NoME, Elvish Reincarnation implies that the Eldar were informed about isotopes by the Valar, so this isn't unreasonable in-universe. (Though finding that passage makes me amused by the thought of seeing Galadriel's NMR spectra.) Double sharp (talk) 04:23, 30 May 2024 (UTC)[reply]
In canon Men are sometimes mistaken for Elves, so there must not be any gross difference in eye size. —Tamfang (talk) 21:37, 30 May 2024 (UTC)[reply]
Assuming a comparable physiology, an Elven retina may pack more sensitive photoreceptor cells, while the lens may have better optical qualities.  --Lambiam 06:08, 31 May 2024 (UTC)[reply]

I wouldn't take it for granted that the Solar System outside of Middle Earth is the same as the real one. According to The Silmarillion Earth (or "Arda") was explicitly created by God and smaller deities with their omnipotent powers, and so were the peoples of that world (Elves, Men, Dwarves, and surely Hobbits and Ents, too). And the part that would drive mad the astronomers reading that book, the Sun and the Moon were also created by those beings... ages after the creation of Arda, and after life existed on that world. Cambalachero (talk) 13:00, 31 May 2024 (UTC)[reply]

Indeed. But Tolkien planned in his later years to make an altered Round World Version in which the Sun and Moon exist from the beginning of Arda. In that case the Two Trees simply preserve their light as it was before they were tainted by Melkor. As I quoted above, these late rewrites imply that the planets in Tolkien's world are the same ones that we have. :) Double sharp (talk) 11:05, 1 June 2024 (UTC)[reply]
Hmm, curious where you heard about these supposed intended revisions to create a lore more consistent with actual astral bodies. I'm a little skeptical without seeing a source on this, just because it doesn't seem to add up with other well-established details. Bear in mind that every element of the legendarium that bears light on the creation and cosmology of Eä was published after J.R.R. Tolkien's death. Christopher Tolkien finished The Silmarillion by trying to faithfully patch together the content (and fill in gaps himself) using an express desire to have the final work reflect what his father, the original author, would have intended. Nobody would have been in a better position than the younger Tolkien (who made the completion of the legendarium a substantial part of his own life's work and had the fullest access to the existing materials) to know what the elder intended in this respect, and he would have had every opportunity to cause the final work to reflect it, as he did with countless other details.
Further, thematically it just doesn't seem to fit: the trees are so fundamental to the cosmonogy of the legendarium, as well as its evolution and eschatology. The trees are part of the more gnostically "pure" version of the world after it was sung into existence out of the higher pleroma by the music of Ainur. The fall of the trees, though instigated by the machinations of Melkor and Ungoliant, are thematically (and arguably psuedo-naturally) the result of an inevitable and unarrestable trend of the world trending away from the direct influence of the Valar and towards a world more defined by physicality and all the ills that come with it.
All of this being the result of Melkor's discordant notes, which Eru permitted to remain a part of the song of creation. First the Valar leave Middle Earth and retire to Valinor, and as time wears on, begin making less and less in the way of even indirect influence over it. Next, the Trees are destroyed and the First Age begins. Later the world is reshapped such that Valinor is not even entirely on the same physical plane as the rest of Arda, and reachable only via the Straight Path. The elves diminish and go into the West, returning ultimately to Valinor, the magic and grace of their realms failing and converting them into mundane lands. Meanwhile the light of the trees persists in the silmarils and the phial of Galadriel, bitter sweet echoes of a purer but irretrievable age. Magic fades and Illuvatar's second (and less ethereal) children, humans, inherit the world.
You see what I mean? The idea of the world's inclination to a state defined increasingly more by a base, more purely physical state and away from the direct influence of the spiritual animus that gave birth to it is baked into the narrative and the lore, from start to finish. And the trees are the ultimate symbol of the starting point (or at least the start after the music was finished and the Ainur descended into Arda). Besides, the sun and the moon predating the round world just doesn't make much sense: a flat Arda wouldn't be able to rotate on is axis. Mind you, not to imply that there's in way to get the legendarium's cosmology to work with actual astrophyics. Which Tolkien very well knew: all indications are that this was a part of the point. But introducing the sun and the moon before the First Age just makes the discontinuities more obvious and intrusive.
All that said, very curious to see where this comes from originally. Certainly there is no shortage of matters that Tolkien went back and forth on over his decades of revisions of the relevant works, nor issues where his son had to make best-guess efforts in choosing among the disparate versions of events. But personally, I tend to doubt that Tolkien seriously considered this particular shake-up. Tonally and in terms of continuity, it just doesn't add up. -sighs a sigh of glutted satisfaction, having sucked out all of the fun the subject matter and wrecked it as surely as a giant spider sucking the magic out a world tree and poisoning it.- SnowRise let's rap 02:51, 3 June 2024 (UTC)[reply]
@Snow Rise: There is a good summary here, though the primary sources (JRRT's texts) are still in Morgoth's Ring (ed. Christopher Tolkien) and The Nature of Middle-earth (ed. Carl Hostetter). A scholarly paper about it is here.
A slightly shorter summary: pre-ROTK, JRRT produced a version of the Ainulindalë with the changes to the cosmology. Melkor seizes part of the Earth to make the Moon for his stronghold, before the Valar cast him out of it and cleanse it. For the time being, it was only an experiment.
But in the late 1950s, JRRT came to believe that the making of the Sun and Moon was too "astronomically absurd" to write in an age when most people believe that the Earth is spherical and is more or less like an island in space. So he came up with a new concept: the "Flat World Versions" are traditions that were handed down by the Númenoreans and then in Arnor and Gondor, that are inescapably blended and confused with Mannish myths. The Elves had their own lore from Valinor that was astronomically and geologically in accord with what we know instead.
JRRT describes the new cosmology across some essays published in the aforementioned collections and in an interview. The Sun and Moon were created together with the Earth, and originally they had the Primeval Light, and what Melkor did instead was corrupt them. Middle-earth was then twilit, because Morgoth darkened the Earth with clouds, such that the stars and moon were invisible and the Sun was only a dim twilight (something like the real Venus without the greenhouse effect, I might add). The significance of the Two Trees is that only there was the Primeval Light preserved, and Varda domed Valinor over to keep Morgoth's corruption out and have it only be lit by the stars. The world was also round from the beginning, but you could not circumnavigate it before the drowning of Númenor because Aman would block the way. The Númenorean Catastrophe removes the inhabitants of Aman from the physical world, though the landmass remains and becomes America after significant geographical upheaval. Thus JRRT writes in these notes Aman and Eressëa would be the memory of the Valar and Elves of the former land.
Crucially, in 1966 The Hobbit was slightly edited in accordance with this new revision: where once read In the Wide World the Wood-elves lingered in the twilight before the raising of the Sun and Moon (completely correct per the Flat World version), there now read In the Wide World the Wood-elves lingered in the twilight of our Sun and Moon. JRRT's late texts published and edited by Christopher in The Peoples of Middle-earth sometimes also imply this: the new description of Fëanor's burning of the ships specifies that it was done "in the night", and that "In the morning the host was mustered", which makes no sense in the old cosmology because the Sun wouldn't yet have first risen; and Thingol's throne room of Menelrond is supposed to be based on the domes of Varda (which don't exist before the changes). The heraldry JRRT drew in the 1960s for the House of Finwë is a "Winged Sun", which also makes more sense in the new cosmology because Finwë otherwise could not have seen a Sun that only rose after he died. So the evidence is clear that the elder Tolkien really seriously intended this change.
As for why it was not adopted by Christopher: Christopher's commentary on these texts indicates that he seems to have thought it a bad idea on the part of his father. I think Christopher made a sensible call, as going on with the intended changes would require much more editorial intervention than leaving things as they were: it does not seem that JRRT ever finished the planned rewrites. But from my perspective as a fan, the whole idea is fascinating and allows the amusing conceit of trying to figure out what was really going on behind the scenes, which is why I asked the initial question about how astronomically plausible the reworking was in one aspect (the visibility of Neptune). Double sharp (talk) 03:45, 3 June 2024 (UTC)[reply]
Fascinating! Just when I thought the paratextual narrative of this particular piece of literature couldn't get more complicated! Personally I like the ultimate approach adopted (even beyond the concern you rightly point out of it requiring more intervention--I still think I prefer the narrative irrespective of whether you hypothesize a scenario in which J.R.R. could make the adjustments himself). The Age of the Trees being treated as literal (or at least facially literal from the perspective of the Silmarillion/legendarium's narrative; all of the books are presented to varying degrees as possibly unreliable ancient historical text, after all) just feels very at home with both the broader cosmogony as well as the thematics of the overall work. But then, I have revisited those books so many times over the years (at least compared to my once, maybe twice and done policy for most literature: soooo many things to read!) that I may have some bias! SnowRise let's rap 16:54, 3 June 2024 (UTC)[reply]
@Snow Rise: I kind of think of JRRT's problem as being that in seeking increased verisimilitude, he'd gone from wanting to write a mythology to wanting to write a science fiction novel. Only, instead of starting a new science fiction novel and leaving the mythology as it was in its almost-complete state, he ended up trying to turn his mythology into a science fiction novel, with predictable results. But I suppose the whole narrative was too dear to his heart for him to come up with another like it, just like the Silmarilli were to Fëanor. Which is understandable: I share your obsession, and how much greater must it have been for the author? :)
With that said: even in this take, the Age of the Trees would still be literal in the sense that there were luminescent trees in Valinor that got killed by a giant spider. They would only not be literal in the sense that the Sun would still be shining (in its form post-tainting by Melkor) over Middle-earth. Only in the northwest (over Beleriand) is there a volcanic winter due to the fumes of Thangorodrim – and from that, we can see where the idea of the Sun first rising later comes from as an in-universe myth. Double sharp (talk) 10:42, 4 June 2024 (UTC)[reply]

May 30[edit]

Volume of honey in a bee nest[edit]

What is the average volume of honey in a bee nest in the wild? I was able to find information on the average volume of a bee nest, but I know that not all of that volume is honey, of course. Thank you! HeyArtemis (talk) 07:49, 30 May 2024 (UTC)[reply]

It varies alot. Honey#Production has a number for Apis mellifera. Sean.hoyland (talk) 08:39, 30 May 2024 (UTC)[reply]

Does light decay?[edit]

Let's say that an object, such as a star, emits a beam of light and it moves across the space. It goes at the speed of light and, unless it reaches an opaque object, it would keep going... for how much time? Forever? Or is there a point when light would simply dimish and disappear? Cambalachero (talk) 19:53, 30 May 2024 (UTC)[reply]

No.
It may react with matter, if it encounters any. A flux of many photons will spread out to a larger volume and so the intensity (number of photons passing through an area) will diminish in accord with the inverse square law. But light passing through a vacuum does not 'decay' or have a limit on its range.
BTW, this theory that light can only travel a few thousand miles before 'running out' is part of flat earther canon for some models, as an explanation of how nighttime happens. But then they're flat earthers. Andy Dingley (talk) 20:11, 30 May 2024 (UTC)[reply]
The idea of tired light used to be somewhat popular but has been entirely discarded by now. --Wrongfilter (talk) 20:22, 30 May 2024 (UTC)[reply]
I'm afraid it is a zombie idea.[2]  --Lambiam 05:58, 31 May 2024 (UTC)[reply]
I guess the apparent reality that things in motion continue in motion forever by default, if unperturbed, seems a bit unnatural when you live in a macroscopic world. Sean.hoyland (talk) 07:32, 31 May 2024 (UTC)[reply]
Correct. If that wasn't the case, we wouldn't be able to see stars that are too far away. We are currently seeing stars so far away that they aren't just stars. They are galaxy or similar objects so far away that they show up as one little blob of light. The limit is not how far light can travel before pooping out. It is how old the stars are. For example, a 100 year old star that is 90 light years away would not be visible becuase the original light hasn't reached us yet (ignoring the complication of direction of relative movement). So, the argument is that the night sky should be flooded with light from all directions. And, it is. It only looks black in a relative sense. 12.116.29.106 (talk) 14:51, 4 June 2024 (UTC)[reply]
I imagine it would be much worse than not being able to see stars. If excitations of quantum fields got tired we would be in big trouble. Sean.hoyland (talk) 15:05, 4 June 2024 (UTC)[reply]

May 31[edit]

Is redshift calculated differently for different spectra?[edit]

I ask because I came across an article, TXS 1545-234, in the course of regular gnoming. The article claims this radio galaxy to be one of the most distant known objects, but its redshift is only around z = 2.754 (which I take to be measured from radio emissions). It does appear in this source (|date= at least 2006), and I'm wondering whence the claim of such great distance paired with such pedestrian redshift. I'm not able to understand our article Redshift. Also, if anyone has any ideas about how to de-orphan the article linked, please do have at. Folly Mox (talk) 11:31, 31 May 2024 (UTC) edited 11:52, 31 May 2024 (UTC)[reply]

It is certainly not one of the most distant galaxies known. The author of the article, @Galaxybeing:, should explain why they think it is. --Wrongfilter (talk) 12:12, 31 May 2024 (UTC)[reply]
They appear to have made a similar claim at MRC 0406-244 (z = 2.44) although thankfully not at another recent creation, QSO J0100-2708 (z = 3.52). Folly Mox (talk) 12:57, 31 May 2024 (UTC)[reply]
The redshift, a dimensionless quantity, is the same for the whole spectrum emitted by an object. Compared to TXS 1545-234, JADES-GS-z13-0 is thought to be more than three times as far away from us.  --Lambiam 12:37, 31 May 2024 (UTC)[reply]
Thanks both for your answers, and confirming my suspicion that the claim was merely incorrect. Folly Mox (talk) 13:00, 31 May 2024 (UTC)[reply]
A redshift of 2.754 is pretty far away, but stating it's one of the most distant known objects is overstating it a bit. Distances to far-away galaxies are normally expressed in redshift, as redshift is directly observable, in contrast to distance, which depends on a model of the expansion of the universe. But when using redshift as a distance measure, one has to keep in mind that it's highly non-linear. Also, distance is a bit of a strange concept when dealing with these cosmologically distant objects. Are we talking about the distance today, or at the time the light was emitted, or the distance travelled by the light? That last at least has some relevance as it translates to the time that the light has travelled and therefore when it was emitted. JADES-GS-z13-0 may be several times farther away today than TXS 1545-234, but most of that is thanks to the expansion of the universe after the light was emitted. When the universe was young and small (although still infinite), it expanded fast in absolute numbers (percentage per year). In light travel distance, the difference isn't so much.
These objects can be studied to learn more about the early universe. For that, knowing the distance to us is not so important; we want to know about the distance (or time) to the Big Bang. At some point, distances (times) to the Big Bang are known more accurately than distances (times) to us. In any case, the redshift tells us immediately that the universe expanded by a factor of five between the times when the light of JADES-GS-z13-0 was emitted and when the light of TXS 1545-234 was emitted. That puts JADES-GS-z13-0 a lot closer to the Big Bang, although only a small fraction farther from us. PiusImpavidus (talk) 11:05, 1 June 2024 (UTC)[reply]
On a tangent but an amusing use (abuse?) of "red shift": chemists use red shift generically to mean "moves to lower energy". Even in the IR part of the spectrum, the term red shift would be used to describe the shift of a band to lower E, say 2000 to 1950 cm-1. This language is of course strange because, formally speaking, a shift toward red for an IR band would mean a shift to higher E. Just sayin'.--Smokefoot (talk) 17:11, 1 June 2024 (UTC)[reply]
Radioastronomers apply the same "abusive" terminology. We see this linguistic phenomenon also in uses of the verb "to dial", as in the advise to "dial 9-1-1 for any emergency" given to users of smartphones with touch screens. (Using the original rotary sense of the verb in connection with the casual parlance of "butt dial" results in the unfortunate mental image of Giuliani twerking.) Other examples are referring to cotton bed sheets as "linens", or (in the US) to stainless-steel knives, forks and spoons as "silverware", and the computer-graphics terminology calling a screen region a "canvas". I think there is a learned term for this phenomenon if the sense of a word getting abstracted from the physical embodiment after which it was originally named.  --Lambiam 05:29, 2 June 2024 (UTC)[reply]
JADES-GS-z13-0
Astronomers talk about redshift if it goes to longer wavelengths, in radio, IR, UV etc., and about blueshift if it goes to shorter wavelengths, in radio, IR, UV etc. Things can also redshift past red. See how red that distant object in this picture is? It's a feature coming from UV, shifted to IR. The Lyman-alpha absorption line is at 121.6 nm, here broadened into a Gunn–Peterson trough, redshifted to 1.6 μm, between the F150W and F200W filters of the camera in JWST. It's how they make a first estimate of the redshift, based on broadband images. A precise number follows later from spectroscopy, but takes far more observing time, so this is only done for the most promising targets. PiusImpavidus (talk) 10:18, 2 June 2024 (UTC)[reply]

Physics problem[edit]

A shop sign is made of a panel that protrudes slightly from the wall on which it is hung, forming an angle of 5° with it. It is 0.74 m tall and has a mass of 8.9 kg. The upper side of the panel is attached to the wall by two cables, one from the right side and one from the left side. Find the tension of the 2 cables 78.211.54.11 (talk) 19:40, 31 May 2024 (UTC)[reply]

Original in Italian, now edited by IP to English.Translates as: "A shop sign consists of a panel that protrudes slightly from the wall on which it hangs, forming an angle of 5° with it. It is 0.74 m high and has a mass of 8.9 kg. The upper end is attached to the wall by two cables, one on the right side and one on the left side. Find the tension in the 2 cables."
IP editor: as it says at the top of this page, we don't answer homework questions and what we do answer should preferably be asked in English. Mike Turnbull (talk) 19:46, 31 May 2024 (UTC)[reply]
Asking homework questions also causes tension. ←Baseball Bugs What's up, Doc? carrots→ 01:15, 1 June 2024 (UTC)[reply]
Doesn't the tension depend on the angle the cables (idealized as straight line segments) make with the panel?  --Lambiam 02:53, 1 June 2024 (UTC)[reply]
Correct. The answer to the question as asked is a curve of tension vs cable length or vertical location. Greglocock (talk) 22:56, 1 June 2024 (UTC)[reply]
Is it like this? Graeme Bartlett (talk) 22:42, 1 June 2024 (UTC)[reply]
or is it like this? Graeme Bartlett (talk) 22:22, 1 June 2024 (UTC)[reply]
like this but not equilateral

In my interpretation of the problem – which is not necessarily the intended one – the lower edge of the panel is attached to the wall and the panel can rotate along that edge, like in the "or is it like this?" diagram, which has an exaggerated thickness for the panel. Unlike that diagram, the panel does not stick out at a 90° angle but is standing almost upright. Also, the cables are attached to the upper edge. So it is more like the situation here to the right, but instead of a bottom 60° angle we have a 5° angle. Not enough info has been given to determine the other angles.  --Lambiam 04:45, 2 June 2024 (UTC)[reply]
P.S. Another way to frame the problem could have been in terms of a drawbridge.  --Lambiam 04:53, 2 June 2024 (UTC)[reply]

The answer is 42 (glorps). I'll leave it to you to figure out the conversion factor to mks. Clarityfiend (talk) 12:06, 3 June 2024 (UTC) [reply]
                |
      		|'.
		|  '.
		|    '.
		|      '.   
		|        '.
		|          ::
		|         ::
		|< 5deg >::
		|       ::
		|      ::
		|     ::.
		|    :: .
		|   ::  .
		|  ::   .
		| ::    .
		|::     V
		|*      8.9kg

Equate turning moments at *
                      clockwise  =  anticlockwise
 8.9 kg x sin(5 deg) x 0.74m / 2 = t/2 x 0.74m
                         where t = tension in each of 2 cables
                                 = 8.9 sin(5 deg)
                                 = 0.775686... kg

Philvoids (talk) 13:16, 3 June 2024 (UTC)[reply]

Shouldn't there be a sine or cosine factor in the anticlockwise term? Imagine the cables being attached to the wall very close to the *. Consider dU/dL, where U is potential energy of the panel and L is the length of the cables.  --Lambiam 15:27, 3 June 2024 (UTC)[reply]
Although not accurately shown in the ASCII sketch my calculation presumes that the two cables are at right angles to the sign, are parallel and are fastened to the wall separately at two points. This minimises the cable lengths and tensions. Philvoids (talk) 10:28, 4 June 2024 (UTC)[reply]
The homework problem did not specify where the cables are attached or that the tension should be minimized, making it unsolvable for the average student. For an old-fashioned drawbridge over a castle moat, as in the animation, a better choice is to place the attachment point of the cables at a distance from the hinge equal to the length of the bridge, making the triangle isosceles. Otherwise, the tension grows without bounds as the bridge near a vertical position.  --Lambiam 15:37, 4 June 2024 (UTC)[reply]
I could not find an ASCII symbol for a hinge but installing one at the base of the sign is a well thought mechanical improvement that can stop the thing flapping about in a wind. This reference desk cannot grant permission to construct a drawbridge with walkway at this location and the shop owner of the wall would likely protest at your plan to cut a hole for passage. Unless he is a herder of sheep or other small animals and himself less than 74cm tall and sees some advantage. The OP doesn't ask for the cable tension when the sign/bridge is vertical but we can say there are two cases: Case #1: The hinge is broken or absent. t = 8.9/2 = 4.45 kg plus distributed weight of the cable itself; Case #2: The hinge takes the weight, t = 8.9 sin (0) so both cable lengths and tensions are zero. In that case it would be simpler just to nail the sign to the wall. Or put out a call to Banksy who can save us the cost of the sign, usually upset someone and greatly increase the value of the wall. Philvoids (talk) 18:39, 4 June 2024 (UTC)[reply]
For all we know, there is already an opening behind the sign for letting the proprietor's homing pigeons in. If they are equipped with an RFID chip, the sign can be let down automatically on their arrival.  --Lambiam 05:39, 5 June 2024 (UTC)[reply]
Pigeon RFIDs can be compromised by malicious Yinpterochiroptera and Yangochiroptera. Members of this notorious "YinYangBat Gang" equipped with ultrasound-to-UHF converters can mount Spoofing attacks on the shared ID frequency. An ad hoc aposematic remedy such as a wall painting of a cat with the sign text "I EAT BATS" does not alone give security. To this end all pigeons must be urged to register a pass-coo that is less obvious than the too common "Coo Coo". Philvoids (talk) 10:28, 5 June 2024 (UTC)[reply]

June 2[edit]

Why females produce androgens[edit]

If human embryons of both sexes start off from a female blueprint and given that females lack the male Y chromosome, how it came that women also produce androgens (even if in small quantity), with related limb and facial hair? 212.180.235.46 (talk) 19:59, 2 June 2024 (UTC)[reply]

The article you linked says that the ovaries also produce androgens. ←Baseball Bugs What's up, Doc? carrots→ 22:33, 2 June 2024 (UTC)[reply]
Biological systems weren't built by any sort of logical designer. They in no way resemble a computer program, a computer, or, for that matter, anything else in the universe. In the case of androgens, the article mentions that androgens are the precursors to estrogens. Males need estrogens too, btw. All these are steroids, which are fundamental to life and are derived from cholesterol. Abductive (reasoning) 23:18, 2 June 2024 (UTC)[reply]
"Biological systems weren't built by any sort of logical designer." Which is why so-called "intelligent design" is just nonsense. The "design" is emphatically not "intelligent". --User:Khajidha (talk) (contributions) 12:07, 3 June 2024 (UTC)[reply]
I'm not sure biology and computing are as far apart as they used to be now that systems essentially create the gigantic opaque function that transforms input to output themselves in response to their environment/what they have seen and the objectives. Generative adversarial networks for example seem a bit closer to biology than systems used to be. Sean.hoyland (talk) 13:06, 3 June 2024 (UTC)[reply]
Not in a billion years. Abductive (reasoning) 17:51, 3 June 2024 (UTC)[reply]
The technology for synthetic biology, still in its infancy, is advancing with large strides. Whether you like it or not, sooner rather than later it will become possible to design and create complete viable and functioning biological organisms.  --Lambiam 06:40, 4 June 2024 (UTC)[reply]
Further, the idea that it is so complicated that nobody understands what it does because it thinks for itself is a farce. No matter what is being used for the computing hardware, be it electronic or biological, the mechanism of operation is very well understood by the engineers who developed it. It just sounds cool to say that it is beyond comprehension. It doesn't sound cool to say that the engineers understand it very well and could trace input through to the output if they wanted to, but simply don't care to do because they have other things to work on. 12.116.29.106 (talk) 14:42, 4 June 2024 (UTC)[reply]
Tracing the input through to the output is IMO not a helpful concept. Not only do we not know why the most advanced chess or go playing programs make certain surprising moves, but it is not even clear what it means to "understand" why they did this. The computing platform performs a calculation with a certain outcome. The engineers can perform the same calculation by hand, or using abaci, and if they make no mistake they may arrive at the same result in a few billion years: 42. But can they say more than that the answer is 42 because this is the consequence of the rules applied to the input? They knew that already. If someone wants to know why it is the consequence, they can tell them to repeat the calculation.  --Lambiam 15:51, 4 June 2024 (UTC)[reply]
IP, LLMs for example involve activations in very high dimensional spaces. Trying to map those activation patterns to things we can understand, like concepts etc., is the whole field of interpretability, and it is in its infancy (and safety critical). Engineers are still far from understanding why input A to a model is transformed into output B. If this is something that interests you, have a look at the work being done in Anthropic's lab. Sean.hoyland (talk) 17:16, 4 June 2024 (UTC)[reply]
Overgrown Markov Chain models are pretty much useless in any technical field. BTDT Greglocock (talk) 04:48, 5 June 2024 (UTC)[reply]
I guess that's one of the reasons why "Attention Is All You Need" turned out to be such a great title for a paper. Sean.hoyland (talk) 08:39, 5 June 2024 (UTC)[reply]

June 5[edit]

Prostheses[edit]

what are the matrials uesed in implantable artificial prostheses, such as artificial hearts and small-diameter blood vessels; in the engineering of living tissues ? -- 196.153.184.240

Heart valves usually made from pyrolytic carbon. https://www.ncbi.nlm.nih.gov/pmc/articles/PMC10034107/#:~:text=Mechanical%20valves%2C%20usually%20made%20from,stroke%20(3%2C%204). 41.23.55.195 (talk) 06:13, 5 June 2024 (UTC)[reply]
Many different materials are used depending on requirements. For details see prosthesis and the many links therefrom.Shantavira|feed me 15:30, 5 June 2024 (UTC)[reply]

June 6[edit]

Health risk of taurine consumption[edit]

Realistically, how worried should people be about the health risks of consuming taurine as a supplement? I only ask because half the literature says they find it helpful and beneficial at some unknown dosage, while the other half says it is potentially carcinogenic and could contribute to colon cancer. As a layperson, I find this very confusing. Some of the literature says it could be simply a matter of dosage, but nobody seems to know what the safe or harmful limits are. Can anyone offer some risk analysis devoid of emotion? Should we avoid anything with taurine in it, or not worry at all about it? Viriditas (talk) 01:27, 6 June 2024 (UTC)[reply]

As far as Wikipedia is concerned, WP:MEDRS applies. That means that the minimum quality level for WP:RS making medical claims are systematic reviews indexed for MEDLINE (there are some exceptions from this indexation, but generally speaking MEDLINE is the gold standard). tgeorgescu (talk) 03:29, 6 June 2024 (UTC)[reply]
Our article states that there is no good clinical evidence that taurine supplements provide any benefit to human health, Why pay for useless supplements? The human body naturally produces a large amount of taurine, far more than one can reasonably take in as a supplement. There is increasing evidence that taurine actually plays a role in preventing cancer.[3] Any carcinogenicity of supplements can only be due to their being fake, or a lack of quality control in their production.  --Lambiam 07:23, 6 June 2024 (UTC)[reply]
The underlying issue is that large doses of taurine are added to energy drinks. Nobody seems to know why. A current study is looking at an association between energy drink consumption and the rise in colon cancer in young adults.[4] Viriditas (talk) 08:23, 6 June 2024 (UTC)[reply]
Even disregarding any potential carcinogenic risks, there are enough studies that show damaging health effects of high consumption levels of energy drinks.[5][6]  --Lambiam 18:28, 6 June 2024 (UTC)[reply]
¼ liter of my favorite energy drink has 80 mg caffeine, while adults usually consume up to 400 mg caffeine per day. And I use the energy drink totally without sugar. tgeorgescu (talk) 18:59, 6 June 2024 (UTC)[reply]
The energy drink thing may be related; most energy drinks contain vitamin B12. There is a concern that (mega)dosing B12 in excess of daily requirements carries with it a slight increase in risk of cancer due to B12 containing cobalt, which is both a heavy metal and has a trace of radioactive cobalt-60. Abductive (reasoning) 20:25, 6 June 2024 (UTC)[reply]
Thank you, that's helpful. I just read the comments by the epidemiologist on that topic, and while my reading might be flawed, they seemed to indicate that dosage and tobacco smoking played a significant role in the risk. Viriditas (talk) 22:02, 6 June 2024 (UTC)[reply]
Real skepticism cuts both ways. Taurine is in human breast milk, evidence of benefit of at least one energy drink exceeding risk. :-) But our article has imho excessive, even dubious doubt in that section on its conditional essentiality or benefit for infants, and thus its common use in another energy drink. Doubt that appears to stem from OR or opinion rather than the source, which says e.g. "Thus the new data provide further support for the view that taurine is a conditionally essential nutrient for the preterm infant" & that ethical considerations seem to prevent further research.John Z (talk) 03:57, 7 June 2024 (UTC)[reply]


June 8[edit]